You are on page 1of 116

Contents

Chapter 1: Basic Concept of Chemistry ....................... 01 – 34

Chapter Assignment ....................................... 16 - 19


Previous Year Questions .................................. 20 - 25
Chapter Test ................................................... 26 - 28
Answers .......................................................... 29 - 29
Hints and Solutions ........................................ 30 - 34

Chapter 2: Redox Reactions ........................................ 35 – 68

Chapter Assignment ....................................... 53 - 57


Previous Year Questions .................................. 58 - 60
Chapter Test ................................................... 61 - 63
Answers .......................................................... 64 - 64
Hints and Solutions ........................................ 65 - 68

Chapter 3: Atomic Structure ...................................... 69 – 114

Chapter Assignment ....................................... 94 - 98


Previous Year Questions ................................ 99 - 107
Chapter Test ............................................... 108 - 110
Answers ...................................................... 111 - 112
Hints and Solutions .................................... 113 - 114

[2022]
Chapter - 1
BASIC CONCEPT OF CHEMISTRY
1.1 Laws of Chemical Combinations
Law of conservation of mass: “In all physical and chemical changes, the total mass of the reactants
is equal to that of the products” or “Matter can neither be created nor destroyed.”

Law of constant composition or definite proportions: “A chemical compound is always found


to be made up of the same elements combined together in the same fixed ratio by weight.”

Law of multiple proportions: “When two elements combine together to form two or more chemical
compounds, then the weights of one of the elements which combine with a fixed weight of the other
bear a simple ratio to one another.”

Law of Reciprocal Proportions: The ratio of the weights of two elements A and B which combine
with a fixed weight of the third element C is either the same or a simple multiple of the ratio of the
weights of A and B which directly combine with each other.

Gay Lussac’s law of gaseous volumes : “When gases react together, they always do so in volumes
which bear a simple ratio to one another and to the volumes of the products, if gaseous, all
measurements are made under the same conditions of temperature and pressure.”

Avogadro’s hypothesis : “Equal volumes of all gases under similar conditions of temperature and
pressure contain equal number of molecules or moles.”

Atom: It is the smallest particle of an element that takes part in a chemical reaction. It may or may
not be capable of free existence.

1
Atomic mass unit (a.m.u) : It is equal to th of the mass of an atom of C-12. It is equal to
12

1.6606 × 10–24 g.

Atomic mass : It is the average relative mass of its atoms as compared with an atom of carbon –
12 isotope taken as 12.

Calculation of average atomic mass : If an element exists in two isotopes having atomic masses

ma  nb
‘a’ and ‘b’ in the ratio m: n, then average atomic mass = .
mn

Gram atomic mass (G.A.M.): The atomic mass of an element expressed in grams is called gram
atomic mass. This amount of the element is called one gram atom. This amount of the element can
also be called one mole atom.

Amity Institute for Competitive Examinations : Phones: 24336143, 24336144, 25573111/12/13/14,95120-2431839/42 [1]
Basic Concept of Chemistry AICE (IIT-JEE)

Method of determining atomic weight

(i) By application of the relation

At. wt. = Eq. wt. × Valency

Knowing the exact eq. wt. and approximate atomic weight, valency can be calculated (which is
a whole number). Knowing exact eq. wt. and valency, exact atomic weight can be calculated.

(ii) Dulong and Petit’s method: According to Dulong and Petit’s law, for solid elements (except
Be, B, C and Si)

At. wt. × Specific heat = 6.4 (approx.)


6.4
 Approx at. wt. = Sp. heat
Approx. At. wt.
= Eq. wt. = approx. Valency..

The approx. valency is converted to the nearest whole number which is called exact valency.

Eq. wt. × exact valency = exact Atomic Weight.

Cp
(iii) Specific heat/molar heat capacity method: For gases,   is 1.66 for monoatomic, 1.40
Cv
for diatomic and 1.30 for triatomic gases. Thus having known the atomicity of the gas from the
Mol. wt.
value of , Atomic wt. of the gaseous element = Atomicity .

Molecular wt. = 2 × Vapour density.

(iv) Vapour Density method: This method can be used for those elements whose chlorides are
volatile so that their vapour densities can be determined. Then

Mol. wt. of the chloride = 2 × V.D.

If x is the valency of the element (M), then the formula of its chloride will be MClx. Hence

Molecular weight of the Chloride MClx = At. wt. of M + x × 35.5


= Eq. wt. of M × Valency of M + x × 35.5
= E × x + x × 35.5
= x (E + 35.5)
2  V.D.
 x (E + 35.5) = 2 × V.D. or x =
E  35.5
Knowing the eq. wt. E of the element, the valency x can be calculated. Then Atomic weight =
eq. wt. × Valency.

(v) Law of Isomorphism: Compounds having similar molecular formulae and identical crystal
structure are called isomorphs. The method is based upon the fact that elements in isomorphous
compounds have same valencies, e.g.,

(a) K2SO4, K2CrO4 and K2SeO4 are isomorphous. Hence valency of S, Cr and Se = 6.

Amity Institute for Competitive Examinations : Phones: 24336143, 24336144, 25573111/12/13/14,95120-2431839/42 [2]
Basic Concept of Chemistry AICE (IIT-JEE)

(b) ZnSO4.7H2O, FeSO4.7H2O, MgSO4.7H2O are isomorphous. Hence valency of Zn, Fe and
Mg = 2.

(c) Alums, M2SO4.M 2(SO4)3.24H2O in which M is monovalent and M is trivalent are


isomorphous.

Knowing the valency, Atomic weight = Eq. wt. × Valency

Molecules: It is the smallest particle of an element or a compound that is capable of free existence.

Molecular mass: Molecular mass of a substance is the average relative mass of its molecules as
compared with an atom of C–12 isotope taken as 12.

Gram molecular mass or molar mass (G.M.M.): The molecular mass of a substance expressed
in gram is called gram molecular mass or molar mass. This amount of the substance is called one
gram molecule. This amount of the substance is called one mole of the substance.

Methods of determining molecular weight

(i) Gram Molecular Volume (G.M.V.) method, 22.4 litres of every gas or vapour at STP weight
equal to molecular weight expressed in grams. This is the principle of Victor Meyer method used
for volatile liquids.

(ii) Vapour density method

Molecular weight = 2 × Vapour Density

Wt. of certain volume of the vapour


where Vapour Density = Wt. of same volume of H under same conditions of temp. and pressure
2

(iii) Diffusion method: According to Graham’s law of diffusion, rates of diffusion of different gases
under similar conditions of temperature and pressure are inversely proportional to the square
root of their densities or molecular weights, i.e.,

r1 d2 2  d2 M2
  
r2 d1 2  d1 M1

Knowing the molecular weight of one gas, that of the other can be calculated.

(iv) Colligative property method: Discussed in the unit of ‘Solutions’.

Moles
The collection of 6.023 × 1023 molecules of an element or ions or compounds constitute one mole of
that element, ion or compound. This number, 6.023 × 1023 is known as Avogadro’s number. This is
called mole-particle relationship.

Mole-Particle Relationship :

e.g.,(i) one mole of Na contains 6.023 × 1023 atoms of Na.

(ii) one mole of oxygen, i.e., one mole of O2 contains 6.023 × 1023 molecules of oxygen.

(iii) one mole of CCl4 carbon tetrachloride contains 6.023 × 1023 molecule of CCl4 (carbon
tetrachloride)

Amity Institute for Competitive Examinations : Phones: 24336143, 24336144, 25573111/12/13/14,95120-2431839/42 [3]
Basic Concept of Chemistry AICE (IIT-JEE)

Mole-Weight Relationship :

(i) one mole of an element weighs equal to gram atomic weight of the element

e.g. one mole of Na weighs 23 gm (Gram atomic wt. of Na = 23)

(ii) one mole of Mg weighs 24 gms (GAM of Mg = 24)

(iii) one mole of substance (molecular state) weighs equal to the gram molecular weight of the
substance.

e.g. one mole of O2 weighs 32 gms (GMM of O2 = 32)


one mole of SO2 weighs 64 gms.

Mole-Volume Relationship:

(i) one mole of every gas at STP occupies 22.4 litres of volume.

e.g. one mole of CO2 at STP occupies 22.4 litres of volume.

(ii) one mole of SO2 at STP occupies 22.4 litres of volume.

Gram Atoms
One gram-atom of an element means collection of 6.023 × 1023 atoms. This concept applies only
to the elements which exists in polyatomic states (e.g., O, Cl, S, P, etc.). It is meaningless for
metals and compounds.

* The number of atoms in w gms of an element whose atomic mass is A is :

w
gm  atom  = mole atom
A

* The number of atoms is given by:


w
No. of atoms =  N0
A
Avogadro’s number: It is the number of atoms present in one gram atom of an element or the
number of molecules present in one gram molecule of the substance. In general, it is the number of
particles present in one mol of the substance. Its value is 6.02 × 1023.

Equivalent weight: The equivalent weight of a substance is the number of parts by weight of the
substance that combine with or displace directly or indirectly 1.008 parts by weight of hydrogen or 8
parts by weight of oxygen or 35.5 parts by weight of chlorine.

Mol. wt. of the acid


Eq. wt. of an acid = Basicity

Mol. wt. of the base


Eq. wt. of a base =
acidity

Gram equivalent weight: The equivalent weight of a substance expressed in grams is called gram
eq. wt. or one gram equivalent.

Amity Institute for Competitive Examinations : Phones: 24336143, 24336144, 25573111/12/13/14,95120-2431839/42 [4]
Basic Concept of Chemistry AICE (IIT-JEE)

Now we can define gram-equivalent (gm eq.) in g gms of a substance whose equivalent weight is
E as follows:

g
No. of eq =
E
* The number of milliequivalents (meq.) in g gms is given by:

g
meq = × 1000
E
Methods of determining equivalent weight:

(i) Hydrogen displacement method: This method is used for metals which react with an acid to
evolve hydrogen gas.

Eq. wt. of the metal is the weight of the metal which displace 1.008 g of H2 or 11200 c.c. of
H2 at STP.

Wt. of metal
Eq. wt. of metal= Wt. of H displaced  1.008
2

Wt. of metal
or = Vol. of H in ml displaced at STP  11200
2

(ii) Oxide formation or reduction of the oxide method: In this method a known weight of the
metal is converted into its oxide directly or indirectly. Knowing the weight of the metal oxide
formed, the weight of oxygen combined can be calculated. Alternatively, a known weight of the
metal oxide may be reduced to metal whose weight is determined.

Wt. of metal
Eq. wt. of metal = Wt. of oxygen combined  8

Wt. of metal
= Vol. of O displaced / combined in ml at STP  5600
2

weight of metal
Eq. wt. of metal = Wt. of chlorine combined  35.5

Wt. of metal
=  11200
Vol. of Cl 2 combined in ml at STP

(iii) Double decomposition method: For a reaction of the type

AB + CD  AD  + BC

(e.q. AgNO3 + NaCl  AgCl  + NaNO3)

Weight of AB taken Eq. wt. of AB Eq. wt. of A  Eq. wt. of B


Weight of AD formed = Eq. wt. of AD = Eq. wt. of A  Eq. wt. of D

Amity Institute for Competitive Examinations : Phones: 24336143, 24336144, 25573111/12/13/14,95120-2431839/42 [5]
Basic Concept of Chemistry AICE (IIT-JEE)

Knowing the equivalent weights of any two radicals out of A, B and D, that of the third can be
calculated.

(iv) Electrolytic method (based on Faraday’s laws of electrolysis)

1 Faraday (96500 coulombs) deposit one gram equivalent of the substance.

Electrochemical equivalent of a substance is the weight of the substance deposited by one


coulomb (1 coulomb = 1 amp × 1 sec).

When the same quantity of electricity flows through solutions of different electrolytes,

Wt. of X deposited Eq. wt. of X


Wt. of Y deposited = Eq. wt. of Y

(v) Neutralization method: This is based on the fact that acids and bases react in equivalent
amounts. Hence equivalent weight of an acid is the weight of the acid which is neutralized by
1000 cc of 1N base solution (which contains 1g eq. of the base) Likewise equivalent weight of
a base can be determined.

(vi) Silver salt method: (for organic acids only) The organic acid is converted into its silver salt
(RCOOAg), a known weight of which is ignited to give residue of Ag which is weighed. Then

Eq. wt. of RCOOAg Wt. of silver salt


Eq. wt. of Ag (108) = Wt. of silver

Eq. wt. of acid (RCOOH) = Eq. wt. of RCOOAg.

(vii)Conversion method: When one compound of a metal is converted into another compound of
the same metal (e.g. metal carbonate  metal oxide), then

Weight of compound I Eq. wt. of metal  Eq. wt. of anion of compound I


Weight of compound II = Eq. wt. of metal  Eq. wt. of anion of compound II

Relationship between eq. wt. atomic wt. and valency of an element:


Atomic weight
Eq. wt. =
Valency

Molecular weight of the substance


Equivalent weight of oxidizing/reducing agent = No . of electrons gained/los t by one molecule

Illustration 1: Fe2(SO4)3 is used in water and sewage treatment to aid the removal of suspended impurities.
Calculate the mass percentage of iron, suphur and oxygen in this compound.

Solution: Step - I: Molecular weight of Fe2(SO4)3 = (56 × 2) + (32 × 3) + (16 × 12) = 400

Number of parts by weight of Fe 56  2


Step - II: % of Fe = Molecular weight of Fe (SO ) ×100 =  100 = 28%
2 4 3 400

Amity Institute for Competitive Examinations : Phones: 24336143, 24336144, 25573111/12/13/14,95120-2431839/42 [6]
Basic Concept of Chemistry AICE (IIT-JEE)

Number of parts by weight of S 32  3


% of S= Molecular weight of Fe (SO ) ×100 =  100 = 24%
2 4 3 400
Number of parts by weight of O 16  12
% of O = Molecular weight of Fe (SO ) ×100 =  100 = 48%
2 4 3 400

Illustration 2 : An inorganic salt gave the following percentage composition: Na = 29.11%, S = 40.51% and
O = 30.38%. Calculate the empirical formula of the salt.
Solution : Calculation of empirical formula.
Element Symb % of At. Mass Relative no. of Simplest atomic Simplest
ol element of atoms ratio whole
element = percentage number
atomic mass atomic
ratio
Sodium Na 29.11 23 29.11 1.266 2
 1.266 1
23 1.266
Sulphur S 40.51 32 40.51 1.266 2
 1.266 1
32 1.266
Oxygen O 30.38 16 30.38 1.898 3
 1.898  1.49  1.5
16 1.266

Thus, the empirical formula is Na2S2O3.

SAMPLE PROBLEMS 1.1 (MCQ)


Problem 1: The sodium salt of methyl orange has 7% sodium. What is the minimum molecular weight of
the compound?
(a) 420 (b) 375
(c) 328.57 (d) 294.46

mass of sodium 23
Solution: (c) % of Na =  100  7   100
molecular mass M
23 100
M  326.6 = 328.57
7
Problem 2: 64g of an organic compound has 24g carbon and 8g hydrogen and the rest is oxygen. The
empirical formula of the compound is
(a) CH4O (b) CH2O
(c) C2H4O (d) None of these

Solution: (a) C H O
Mass 24 8 32
24 8 32
Moles
12 1 16
Ratio 2 8 2
Simplest ratio 1 4 1
Hence empirical formula is CH4O

Amity Institute for Competitive Examinations : Phones: 24336143, 24336144, 25573111/12/13/14,95120-2431839/42 [7]
Basic Concept of Chemistry AICE (IIT-JEE)

Problem 3: How many gram ions of SO24 are present in 1 gram molecule of K2SO4.Al2(SO4)3.24H2O

(a) 2 (b) 3
(c) 1 (d) 4

Solution: (d) 1g molecule is 1 mole

Mole of SO24 4 × 1 = 4 g ion

Problem 4: Common salt obtained from sea-water contains 96% NaCl by mass. The approximate number
of molecules of NaCl present in 10.0g of the common salt is: (Atomic weight Na = 23)
(a) 1021 (b) 1022
(c) 1023 (d) 1024

Solution: (c) Mass of NaCl = 10 × 0.96 = 9.6 g


9.6
Moles of NaCl =
58.5
9.6
No. of molecules =  6.023 1023  1023
58.5
Problem 5: X and Y are two elements which form X2Y3 and X3Y4. If 0.20 mol of X2Y3 weighs 32.0 g
and 0.4 mol X3Y4 weighs 92.8 g, the atomic weights of X and Y are respectively
(a) 16.0 and 56.0 (b) 8.0 and 28.0
(c) 56.0 and 16.0 (d) 28.0 and 8.0

32 92.8
Solution: (c)  0.2;  0.4
2x  3y 3x  4 y
Hence x =56 and y =16

Problem 6: Four 1-1 litre flasks are separately filled with the gases H2, He, O2 and O3 at the same
temperature and pressure. The ratio of total number of atoms of these gases present in
different flask would be:
(a) 1 : 1 : 1 : 1 (b) 1 : 2 : 2 : 3
(c) 2 : 1 : 2 : 3 (d) 3 : 2 : 2 : 3

Solution: (c) No. of atoms = no. of moles × atomicity


H2 He O2 O3
1×2 1×1 1× 2 1×3
2:1:2:3

Problem 7: 16g of an ideal gas SOx occupies 5.6 L at STP. The value of x is
(a) x = 3 (b) x = 2
(c) x = 4 (d) None of these
5.6 1
Solution:
 mole
(b) No. of moles of SOx = 22.4 4
Molar mass of SOx = Wt / Moles = 16 × 4 = 64
x = 2

Amity Institute for Competitive Examinations : Phones: 24336143, 24336144, 25573111/12/13/14,95120-2431839/42 [8]
Basic Concept of Chemistry AICE (IIT-JEE)

Problem 8: The empirical formula of a compound of molecular mass 120 is CH2O. The molecular
formula of the compound is
(a) C2H4O2 (b) C4H8O4
(c) C3H6O2 (d) All of these

molar mass 120


Solution: (b) n   4
Empirical formula mass 30
Molecular formula is C4H8O4

Problem 9: A compound possess 8% sulphur by mass. The least molecular mass is:
(a) 200 (b) 400
(c) 155 (d) 355
Wt.
Solution: (b) % of S = 100
Molar mass
32  100
Molar mass   400
8
Problem 10: 12g of alkaline earth metal gives 14.8 g of its nitride. Atomic weight of metal is
(a) 12 (b) 20
(c) 40 (d) 14.8
Solution: (c) 3M  N 2 
 M3 N 2
12g 2.8g 14.8g
Moles of metal = 3 × moles of N2
12 2.8
 3
Atomic weight 28
120
Atomic weight of metal =  40
3

1.2 Expressing concentration of solutions


Solution is a homogenous mixture of two or more components in which Intermixing particles are of
atomic or molecular dimensions. A solution consists of a dissolved substance known as solute and
the substance in which the solute is dissolved is known as solvent. The concentration of a solution
means the quantity of solute dissolved per unit volume of solution, or per unit quantity of solvent.

quantity of solute
Concentration of solution = quantity of solvent or solution

Note: While discussing various methods for expressing concentration, we have taken solute as B
dissolved in solvent A and WA as grams of solute and WB as grams of solvent.

Mass fraction is the fractional part of a component that is contributed by it to the total mass of
solution.
wB
mass fraction of B = w w
A B

wA
mass fraction of A = w  w
A B

Amity Institute for Competitive Examinations : Phones: 24336143, 24336144, 25573111/12/13/14,95120-2431839/42 [9]
Basic Concept of Chemistry AICE (IIT-JEE)

Mole fraction is the fractional part of the moles that is contributed by each component to the total
number of moles that prepare the solution. In containing nA moles of solvent and nB moles of solute:
nB
mole fraction of B = xB = n  n
A B

nA
mole fraction of A = xA = n  n
A B

Molality (m) is expressed as number of moles of solute dissolved in 1000 gms (1 Kg) of solvent. It
is denoted by m.
nB
m  1000
wA

Molarity (m) is expressed as moles of solute contained in one litre of solution or it is also taken as
millimoles of solute in 1000 cc (ml) of solution. It is denoted by M.

moles of solute millimoles of solute


molarity (M) = litres of solution = millilitre s of solution

nB WB / WB
M  =
Vlt V(lit)

Normality (N) is expressed as the number of gram equivalents (gm eq.) of solute contained in one
litre of solution or it can also be taken as number of milliequivalents (meq) in 1000 cc (ml) of solution.
It is denoted by N.

2meq. of solute milieq. of solute


normality of solution (N) = litres of solution = millilitres of solution

gm eq.
N= V(lit)

Note: The following results should be remembered and used directly. Molarity (M) and Normality
(N) are temperature dependent but molality (m) and mole fraction are Temperature
independent units.

wB
(1) moles = M Vlt = M
B

wB
milimoles = M Vcc = M  1000
B
wB
(2) gm-eq. = N Vlt = E
B

wB
meq. = N Vcc = E  1000
B

10 x d 10 x d
(3) Molarity (M) = M0 Normality (N) =
E
d : density of solution in g/cc x :% age strength of solution (by weight)

Amity Institute for Competitive Examinations : Phones: 24336143, 24336144, 25573111/12/13/14,95120-2431839/42 [ 10 ]


Basic Concept of Chemistry AICE (IIT-JEE)

(4) N = x × M

1000  xB
(5) m = M  x
A A

x : acidity for base or basicity for acid or electron transfer / mole for O.A. and R.A./n-factor.

Strength of a solution is generally expressed as grams of solute in one litre of solution.


wB
strength = litres of solution

strength = Normality × Eq. wt. = Molarity × Mole wt.

Diluting a solution: Whenever a given solution of known concentration, i.e., normality & molarity
(known as standard solution) is diluted by adding more of solvent, the number of millimoles (or
millequivalents) of solutes remain unchanged. The concentration of solution however changes.

In such cases M1 V1 = M2 V2 (M1 & V1 are molarity and volume of original solution)

(M2 & V2 are molarity and volume of diluted solution)

Stoichiometric Calculations:
(Quantitative analysis of a balance chemical equation)

Consider a balanced chemical reaction of equation:

mA + nB  pC + qD

A and B are reactants; C and D are products; m, n, p, q are the stoichiometric co-efficients.

The above balanced reaction is analysed as:

m moles of A react with n moles of B to produce p moles of C and q moles of D.

This can be represented as:

m moles of A  n moles of B  p moles of C  q moles of D

It is also observed that when a substance takes part in a chemical reaction then the amount of the
substance taking part is proportional to its equivalent weight

i.e. 1 equivalent of every substance combines (displaces, double displaces, oxidises, reduces,
neutralizes) are equivalent of another substance. Hence in a chemical reaction of the type

aA + bB — mC + nD
1 eq. of A combines with 1 eq. of B to form 1 eq. of C and 1 eq of D or
neq of A  neq of B  neq of C  neq of D.
In general, there are two types of stoichiometric reactions (which are most common):

Neutralisation Reactions: The analysis of two types of reactions is generally carried out in the
form of mass of reactants (or products) taking part in a given reaction (gravimetric analysis) or in
terms of concentrations of reactants (or products) taking part in a given reaction
(volumetric analysis).

Amity Institute for Competitive Examinations : Phones: 24336143, 24336144, 25573111/12/13/14,95120-2431839/42 [ 11 ]


Basic Concept of Chemistry AICE (IIT-JEE)

Neutralisation: A reaction in which an acid (or a base) completely reacts with a base (or an acid)
to form salt and water. If HA be the acid, BOH be the base and BA be the salt, then neutralisation
reaction can be represented as follows:

HA + BOH  BA + H2O

As we know that an acid may be monobasic (HCl, HNO3, etc.), dibasic (H2SO4, H2C2O4, etc.) or
tribasic (H3PO4, etc.) and similarly a base may be monoacidic (NaOH, NaHCO3, etc.), diacidic
(Ca(OH)2, Na2CO3, etc.) or triacidic (Al(OH)3, etc.), so it is better to define the neutralisation
reaction in the following manner.

A reaction in which 1 gram equivalent (or 1 meq of an acid (or a base) completely reacts with 1 gram
equivalent (or 1 meq) of a base (or an acid) to form 1 gram equivalent (or 1 meq) of
corresponding salt.

Illustration 1: Oxygen is prepared by catalytic decomposition of potassium chlorate (KClO3). Decomposition


of potassium chlorate gives potassium chloride (KCl) and oxygen (O2). How many moles
and how many grams of KClO 3 are required to produce 2.4 mole O 2 ?

(M KClO3 = 122.5g/ mole)

Solution : Decomposition of KClO3 takes place as


2KClO3(s) 2KCl(s) + 3O2(g)
2 mole KClO3 gives 3 mole of O2
 3 mole O2 is formed by 2 mole of KClO3.
2 
 2.4 mole O2 will be formed by   2.4  mole KClO3 = 1.6 mole KClO3.
 3 
Mass of KClO3 = Number of moles × Molar mass = 1.6 × 122.5 = 196 g

Illustration 2: 1 litre mixture of CO and CO2 is taken. This is passed through a tube containing red hot
charcoal. The volume now becomes 1.6 litre. The volumes are measured under the same
conditions. Find the composition of mixture by volume.

Solution : Let there be ‘x’ ml of CO in the mixture. Hence there will be (1000 – x) ml CO2. The
reaction of CO2 with red hot charcoal may be given as,

CO2(g) + C(s) 2CO(g)


1000 – x 2000 –2 x
Total volume of the gas becomes = x + 2(1000 – x)
x + 2000 – 2x = 1600
x = 400 ml
 Volume of CO = 400 ml
and volume of CO2 = 600 ml

Amity Institute for Competitive Examinations : Phones: 24336143, 24336144, 25573111/12/13/14,95120-2431839/42 [ 12 ]


Basic Concept of Chemistry AICE (IIT-JEE)

Illustration 3: A solution of oxalic acid C2H2O4. 2H2O is prepared by dissolving 0.63 g of the acid in 250
cm3 of the solution. Calculate molarity of the solution.

Solution : Molar mass of oxalic acid = 126 g/mol.


250
 250 cm3 or L = 0.25 L of the solution contains 0.63 g oxalic acid.
1000
0.63 1
 Molarity of the solution =  = 0.02 M
126 0.25

SAMPLE PROBLEMS 1.2 (MCQ)

Problem 1: 2KI  I 2  22HNO3 


 2HIO3  2KIO3  22NO2  10H2 O
If 3 mole of KI and 2 moles I2 are reacted with excess of HNO3. Volume of NO2 gas
evolved at NTP is
(a) 739.2 L (b) 1075.2 L
(c) 44.8 L (d) 67.2 L

Solution: (a) KI is limiting reagent


 3 mole of KI will give 33 mole of NO2 according to stoichiometry.

Problem 2: If 240 g of carbon is taken in a container to convert it completely to CO2 but in industry it has
been found that 280g of CO was also formed along with CO2. Find the mole percentage
1
yield of CO2. The reactions occurring are C  O 2 
 CO 2 ; C  O 2 
 CO
2
(a) 25% (b) 50%
(c) 75% (d) 100

Solution: (b) C  O 2 
 CO2  CO
POAC on ‘C’ atom
1 (mole of C) = 1 (mole of CO2) + 1 (mole of CO)
240 280
 mole of CO 2 
12 28
Mole of CO2 = 20 – 10 = 10
10
Mole % CO2 = 100  50%
20

Problem 3: In the reaction 4A  2B  3C 


 A 4 B2C3 what will be the number of moles of product
formed. Starting from 2 moles of A, 1.2 moles of B and 1.44 moles of C
(a) 0.5 (b) 0.6
(c) 0.48 (d) 4.64
Solution: (c) 4A  2B  3C 
 A 4 B2 C 3
Initial mole 2 1.2 1.44 0
Final mole 0 0.48
C is limiting reagent
 moles of A4B2C3 is 0.48

Amity Institute for Competitive Examinations : Phones: 24336143, 24336144, 25573111/12/13/14,95120-2431839/42 [ 13 ]


Basic Concept of Chemistry AICE (IIT-JEE)

Problem 4: How much NaNO3 must be weighed out to make 50 ml of an aqueous solution containing
70 mg of Na+ per mL?
(a) 12.394 g (b) 1.29 g
(c) 10.934 g (d) 12.934 g

Solution: (d) M wt of NaNO3 = 85


70 mg of Na+ are present in 1 mL
50 ml of solution contains 50 × 70 = 3500 Mg = 3.5 g Na+ ion
23 g of Na+ are present in 85g of NaNO3
85
3.5 g Na+ are present in  3.5  12.934 g of NaNO3
23
Problem 5: The strength of 10 –2 M Na 2 CO 3 solution in terms of molality will be
(density of solution = 1.10 g mL–1) (Molecular weight of Na2CO3 = 106 g mol–1)
(a) 9.00 × 10–3 (b) 1.5 × 10–2
(c) 5.1 × 10–4 (d) 11.2 × 10–3

M  1000
Solution: (a) Explanation: m  where ‘m’ is molality, M is molarity
(100  d  M  Molecular weight)

102 1000 10 10
    9.00  103
(1000 1.1  102  106) 1100  1.6 1099.4
[Take 1099.4 = 1100]

Problem 6: A solution containing 0.1 mol of a metal chloride MClx requires 500 ml of 0.8 M AgNO3
solution for complete reaction MCl x  xAgNO3 
 xAgCl  M(NO3 ) x . Then the
value of x is
(a) 1 (b) 2
(c) 4 (d) 3

Solution: (c) MCl x  AgNO3 


 xAgCl  M(NO3 ) x
Mole of MCl x Mole of AgNO3

1 x
1
0.1  (0.5  0.8)
x
0.4
x 4
0.1
Problem 7: The temperature at which molarity of pure water is equal to its molality is:
(a) 273 K (b) 298 K
(c) 277 K (d) None of these

Solution: (c) At 4°C i.e. 277 K density of water = 1 g/ml


1 kg water  1000 ml water = 1 lit
Molarity and molarity remains same

Amity Institute for Competitive Examinations : Phones: 24336143, 24336144, 25573111/12/13/14,95120-2431839/42 [ 14 ]


Basic Concept of Chemistry AICE (IIT-JEE)

Problem 8: What is the molarity of H2SO4 solution that has a density 1.84 g/cc at 35°C and contains
98% by weight?
(a) 4.18 M (b) 8.14 M
(c) 18.4 M (d) 18 M

(%w / w)  density 10 98  1.84  10


Solution: (c) Molarity    18.4 M
Molar mass of solute 96
Problem 9: 5.85g of NaCl is dissolved in 1 L of pure water. The number of ions in 1 mL of this solution
is
(a) 6.02 × 1019 (b) 1.2 × 1022
(c) 1.2 × 1020 (d) 6.02 × 1020

5.85
Solution: (c) Mole of NaCl =  0.1
58.5

0.1
Molarity =  0.1M
1
Moles in 1 ml of solution = MV = 0.1 × 10–3 = 10–4
Number of ions in 1 ml = 2 × 10–4 × 6.02 × 10–23 = 1.204 × 1020

Problem 10: H3PO4 (98g mol–1) is 98% by mass of solution. If the density is 1.8 g/ml, the molarity is
(a) 18 M (b) 36 M
(c) 54 M (d) 0.18 M

10 1.8  98
Solution: (a) Molarity   18 M
98



Amity Institute for Competitive Examinations : Phones: 24336143, 24336144, 25573111/12/13/14,95120-2431839/42 [ 15 ]


Basic Concept of Chemistry AICE (IIT-JEE)

CHAPTER ASSIGNMENT
MULTIPLE CHOICE QUESTIONS (Single Correct)
1. For the reaction A + 2B C, 5 moles of A and 8 moles of B will produce
(a) 5 moles of C (b) 4 moles of C (c) 8 moles of C (d) 13 moles of C

2. 5 volumes of a hydrocarbon on complete combustion consumed 10 volumes of oxygen giving 5


volumes of CO2 at STP. The hydrocarbon is
(a) C2H6 (b) C2H5 (c) CH4 (d) C2H4

3. The volume of oxygen used when x gms of Zn is converted to ZnO is


x2 x 4x
(a)  5.6 litres (b)  5.6 litres (c)  5.6 litres (d) None of these
65 65 65

4. The volume of water which should be added to 300 ml of 0.5 M NaOH solution so as to get a solution
of 0.2 M is:
(a) 550 ml (b) 350 ml (c) 750 ml (d) 450 ml

5. The molality of a solution of conc. HCl containing 36.5% by weight of HCl would be
(a) 16.75 (b) 17.75 (c) 15.75 (d) 14.75

6. A mixture of N2 and H2 to react in a closed container to form NH3. The reaction ceases before
either reactant has been totally consumed. At this stage, 2.0 moles each of N2, H2 and NH3 are
present. The moles of N2 and H2 present originally were respectively,
(a) 4 and 4 moles (b) 3 and 5 moles (c) 3 and 4 moles (d) 4 and 5 moles

7. 5.6 litres of oxygen at NTP is equivalent to


1 1 1
(a) 1 mole (b) mole (c) mole (d) mole
2 4 8
8. The haemoglobin form the red corpuscles of most mammals contains app. 0.33% of iron by weight.
The physical measurement indicates the molecular weight of haemoglobin 67200. The number of
iron atoms in each molecule of haemoglobin is (At. wt. of Fe = 56)
(a) 2 (b) 3 (c) 4 (d) None of these

9. If 0.5 mol of BaCl2 is mixed with 0.2 mol of Na3PO4, the maximum number of mole of Ba3(PO4)2
that can be formed is
(a) 0.7 (b) 0.5 (c) 0.30 (d) 0.10

10. A gas of mixture of 3.0 litres of propane and butane on complete combustion at 25ºC produced 10
litres of CO2. Find out the composition of the gas mixture.
(a) 70%, 30% respectively (b) 66.66%, 33.33% respectively
(c) 30%, 70% respectively (d) None of these

11. One mole of a mixture of CO and CO2 required exactly 20 g of NaOH in solution for complete
conversion of all the CO2 into Na2CO3. How much NaOH would it require for conversion into
Na2CO3 if the mixture (one mole) is completely oxidised to CO2?
(a) 60 g (b) 80 g (c) 40 g (d) 20 g

Amity Institute for Competitive Examinations : Phones: 24336143, 24336144, 25573111/12/13/14,95120-2431839/42 [ 16 ]


Basic Concept of Chemistry AICE (IIT-JEE)

12. The volume ratio of 6 N and 2 N HCl required to prepare 100 mL of 5 N HCl is
(a) 3 : 1 (b) 1 : 3 (c) 4 : 1 (d) 1 : 4
13. Specific volume of a cylindrical virus particle is 6.02 × 10–2 mL/g, having the radius and length 7Å
and 10Å respectively. What is the molecular weight of virus?
(a) 15.4 kg/mol (b) 1.54 × 104 kg /mol
(c) 4.68 × 104 kg /mol (d) 2.08 × 104 kg /mol
14. 2.5 litre of 1 M NaOH solution is mixed with another 3 litre solution of 0.5 M NaOH. The molarity of
resultant solution is
(a) 1.0 M (b) 0.84 M (c) 0.73 M (d) 0.56 M
15. When the same amount of Zn is treated separately with excess of H2SO4 and excess of NaOH, the
ratio of volumes of H2 evolved is
(a) 1 : 1 (b) 1 : 2 (c) 2 : 1 (d) 9 : 4

INTEGER TYPE QUESTIONS


16. 0.98 g of a polybasic acid (mol. wt. 98) requires 30 mL of 0.5 M Ba(OH)2 for complete neutralisation.
The basicity of acid is_____?

17. A solution of H2O2 has normality N/1.7. Its % strength is_____?

18. Number of water molecules attached on Cu2+ ion in CuSO4. 5H2O is_____?

19. ‘n’ factor for potash alum is_____?

20. A gaseous alkane CnH2n + 2 is exploded with oxygen. The volume of oxygen for complete combustion
of alkane to CO2 formed is in the ratio 7 : 4. The value of n is_____?

21. Two acid solution A and B are titrated separately each with 25 mL of 1 N Na2CO3 solution. The
volume of each acid used for titration is 10 mL and 40 mL respectively for the complete neutralisation.
The volume ratio of VB and VA which is mixed to prepare one mL 1 N solution is_____?

22. Number of equivalents of oxygen in 1 M K2Cr2O7 acting as oxidant in presence of acid are_____?

23. An inorganic compound ZnCr2Ox, 9.81 g Zn, 1.8 × 1023 atoms of Cr and 0.6 g atom of O. The
volume of x is_____?

24. The equilibrium molarity of OH– is 0.08 M in a solution of 0.1 M (OH)2. The % of dissociation of
Ca(OH)2 is_____?

25. The normality of 0.5 M H3PO3 solution is_____?

MULTIPLE CHOICE QUESTIONS (More than one Correct)


26. 1 mole of a mixture of CO and CO2 requires exactly 1 litre solution of 1 M NaOH for complete
neutralisation. If CO present in mixture is now converted to CO2 and again the mixture is treated
with NaOH, then after this conversion:
(a) moles of CO2 present initially in mixture =1
(b) 2 litre NaOH solution of 1 M is more required for neutralisation
(c) 2 litre solution of 1/2 M NaOH is required more for neutralisation
(d) 56 g KOH in aqueous solution is required more for neutralisation

Amity Institute for Competitive Examinations : Phones: 24336143, 24336144, 25573111/12/13/14,95120-2431839/42 [ 17 ]


Basic Concept of Chemistry AICE (IIT-JEE)

27. Which one is not correct about VO + Fe2O3 FeO + V2O5?


(a) 2 moles of VO reacts completely with 5 mole of Fe2O3
(b) 1 mole of VO reacts completely with 1.5 mole of Fe2O3
(c) Eq. weight of V2O5 = M/6 and of Fe2O3 is M/2
(d) Eq. weight of VO = M/3 and of FeO is 2M/3

28. 1 mole of H3PO3 reacts with NaOH in solution. Select the correct statements:
(a) 1 mole of NaOH will replace N H+ ion from H3PO3
(b) 2 moles of NaOH will replace 2N H+ ion from H3PO3
(c) 3 moles of NaOH will replace 3N H+ ion from H3PO3
(d) on complete neutralisation of H3PO3, the equivalent weight of H3PO3 = 41

29. 100 mL of 0.8 M NaOH are used to neutralised 100 mL solution obtained by passing 2.70 g SO2Cl2
in water. Select the correct statement:
(a) The solution of SO2Cl2 has 0.2 M H2SO4 and 0.4 M HCl
(b) The volume ratio of NaOH used for H2SO4 and HCl is 1 : 2
(c) The volume ratio of NaOH used for H2SO4 and HCl is 1 : 1
(d) Molarity of SO2Cl2 solution is 0.1 M

30. Which one are correct about the solution that contains 3.42 ppm Al2(SO4)3 and 1.42 ppm Na2SO4?
(a) [Al3+] = [Na+] (b) [SO42–] = [Na+] = [Al3+]
(c) [SO42–] = [Na+] + [Al3+] (d) [SO42–] = [Na+]

31. The pair of species having different percentage (mass) of carbon is


(a) CH3COOH and C6H12O6 (b) CH3COOH and C2H5OH
(c) HCOOCH3 and HCOOH (d) C2H5OH and CH3OCH3

32. 1 mole of Ba(OH)2 will exactly neutralise


(a) 0.5 mole HCl (b) 1 mole H2SO4 (c) 1 mole H3PO4 (d) 2 mole H3PO2

33. A sample of H2O2 solution labelled as 28 volume has density of 265 g/L. Select the correct option
representing concentration of same solution.
W
(a) M H2O2  2.5 (b) %  17
V
(c) mole fraction of H2O2 = 0.2 (d) m H 2O2  13.88

34. If 1 mole of H3PO4 is reacted with 1 mole of X(OH)2 as


 XHPO 4  2H 2O then
H3 PO 4  X(OH) 2 
Molecular weight
(a) The equivalent weight of base is
2
98
(b) The equivalent weight of H3PO4 is
3
(c) The resulting solution is required 1 mole NaOH for complete neutralisation
(d) Minimum 1 mole X(OH)2 is required for complete neutralisation of XHPO4.

35. 1g molecule of V2O5 contains


(a) 5 mole of oxygen atom (b) 2 mole of V atom
(c) 1 mole of oxygen atom (d) 2.5 mole of oxygen atom

Amity Institute for Competitive Examinations : Phones: 24336143, 24336144, 25573111/12/13/14,95120-2431839/42 [ 18 ]


Basic Concept of Chemistry AICE (IIT-JEE)

MATRIX MATCH TYPE QUESTIONS


36. Column I contains the compounds of Ammonia which will react with 224 gm of KOH to evolve
NH3. Column II contains the volume of Ammonia evolved in the reaction.
Match Column I with Column II.
Column I Column II
A. 149 gm of (NH4)3PO4 (p) 44.8 
B. 513 gm of NH4ClO2 (q) 89.6 
C. 166 gm of NH4H2PO2 (r) 22.4 
D. 58 gm of (NH4)2SO3 (s) 67.2 

37. X, Y are two elements which has equivalent weight 40 and 80 respectively. List I contains the
corresponding compounds of X and Y and List II contains the ratio of their equivalent weight of X
and Y of the corresponding compounds.
Column I Column II
A. Sulphite (p) 0.63
B. Nitrate (q) 0.75
C. Carbonate (r) 0.72
D. Bromide (s) 0.67

COMPREHENSION TYPE QUESTIONS


Comprehension-I
Normality is number of gram equivalents dissolved per litre of solution. It changes with change in
temperature. In case of monobasic acid, normality and molarity are equal but in case of dibasic acid, normality
is twice the molarity. In neutralization and redox reactions, number of milliequivalents of reactants as well
as products are always equal

38. On heating a litre of a N/2 HCl solution, 2.750 g of HCl is lost and the volume of solution becomes
750 mL. The normality of resulting solution will be
(a) 0.58 (b) 0.75 (c) 0.057 (d) 5.7

39. The volume of 0.1 M Ca(OH)2 required to neutralize 10 mL of 0.1 N HCl will be
(a) 10 mL (b) 20 mL (c) 5 ml (d) 40 mL

40. Molarity of 0.5 N Na2CO3 is


(a) 0.25 (b) 1.0 (c) 0.5 (d) 0.125

Comprehension - II

The density of 3 M solution of Na2S2O3 is 1.25 g mL–1

41. The % by weight of Na2S2O3 is


(a) 36.24 (b) 37.92 (c) 40.24 (d) 38.34

42. Mole fraction of Na2S2O3 is


(a) 0.015 (b) 0.025 (c) 0.065 (d) 0.035

43. Molalities of Na+ and S2O32– ions are respectively


(a) 7.732, 3.866 (b) 3.866, 7.732 (c) 3.732, 7.866 (d) 7.866, 3.732

Amity Institute for Competitive Examinations : Phones: 24336143, 24336144, 25573111/12/13/14,95120-2431839/42 [ 19 ]


Basic Concept of Chemistry AICE (IIT-JEE)

PREVIOUS YEAR QUESTIONS


1. How many moles of electron weigh one kilogram? [IIT 2002]

1 6.023 1
(a) 6.023 × 1023 (b)  10 31 (c)  10 54 (d)  108
9.108 9.108 9.108  6.023

2. The total number of electrons present in 18 ml of water (density of water is 1 g ml–1) is [IIT 1980]
(a) 6.02 × 1023 (b) 6.02 × 1022 (c) 6.02 × 1024 (d) 6.02 × 1025

3. If 10 21 molecules are removed from 200 mg of CO 2 , then the number of moles of CO 2


left are [IIT 1983]

(a) 2.88 × 10–3 (b) 28.8 × 10–3 (c) 0.288 × 10–3 (d) 1.66 × 10–2

4. The number of gram molecules of oxygen in 6.02 × 1024 CO molecules is [IIT 1990]

(a) 10 gm molecules (b) 5 gm molecules (c) 1 gm molecules (d) 0.5 gm molecules

5. At 100°C and 1 atm, if the density of liquid water is 1.0 g cm–3 and that of water vapour is
0.0006 g cm–3 , then the volume occupied by water molecules in 1 litre of steam at that
temperature is [IIT 2000]

(a) 6 cm3 (b) 60 cm3 (c) 0.6 cm3 (d) 0.06 cm3

6. The triad of nuclei which is isotonic is [IIT 1988]


14 15 17 12 14 19 14 14 17
(a) 6C , 7N , 9F (b) 6C , 7N , 9F (c) 6C , 7N , 9F (d) 6C , 7N , 9F19
14 14

7. 29.2% (w/w) HCl stock solution has a density of 1.25 g mL–1. The molecular weight of HCl is 36.5 g
mol–1. The volume (mL) of stock solution required to prepare a 200 mL solution of 0.4 M HCl is___?
[JEE-Advance 2013]

8. If the value of Avogadro number is 6.023 × 1023 mol–1 and the value of Boltzmann constant is
1.380 × 10–23 J K–1, then the number of significant digits in the calculated value of the universal gas
constant is____? [JEE-Advance 2014]

9. A compound H2X with molar weight of 80 g is dissolved in a solvent having density of 0.4 g ml–1.
Assuming no change in volume upon dissolution, the molality of a 3.2 molar
solution is___? [JEE-Advance 2014]

10. Mole concept is not applicable to [DCE-2002]


(a) volume of gases as NTP (b) volume of liquids at NTP
(c) atoms & molecules (d) ions & particles

11. The dot at the end of a sentence has a mass of one microgram. Assuming the black stuff is carbon,
calculate the number of carbon atoms which make such a dot [DCE-2002]

(a) 2 × 1023 (b) 5 × 1016 (c) 6.0 × 1023 (d) 6 × 107

Amity Institute for Competitive Examinations : Phones: 24336143, 24336144, 25573111/12/13/14,95120-2431839/42 [ 20 ]


Basic Concept of Chemistry AICE (IIT-JEE)

12. A mixture of CH4N2 and O2 is inclosed in a vessel of one litre capacity at 0°C. The ratio the
partial pressure of gases is 1 : 4 : 2. The total pressure of the gaseous mixture is 2660 mm. The
number of molecules of oxygen present in vessel is [DCE-2002]

6.02  1023
(a) (b) 6.02 × 1023 (c) 22.4 × 1022 (d) 1000
22.4

13. An X molal solution of a compound in benzene has mole fraction of solute equal to 0.2. The value
of X is [DCE-2001]
(a) 14 (b) 3.2 (c) 1.4 (d) 2

14. ‘X’ gm of an element gave ‘Y’ gm of oxide. Eq. wt. of the element is [DCE-1999]

X X X Y
(a) 8 (b) (Y – X) × 8 (c) Y  X  8 (d) 8
Y X X

15. If 9.8 gm of hexane burns completely in oxygen, how many moles of CO2 is produced?[DCE-1995]
(a) 6 (b) 0.6 (c) 0.9 (d) 1.2

16. If m1 gm of metal A displaces m2 gm of another metal B from the salt solution and if their equivalent
weights are E1 and E2 respectively, then the equivalent weight can be expressed by [DCE-1995]

m1 m2 m1  m2 m1
(a) E1 = × E2 (b) E1 = × E2 (c) E1 = (d) E1 =  E2
m2 m1 E2 m2

17. What volume of CO2 will be liberated at NTP if 1.2 gm of carbon is burnt in excess of oxygen?
(a) 11.2 litres (b) 22.4 litres
(c) 2.24 litres (d) 1.12 litres [DCE-1994]

18. One mole of CO2 contains [DCE-1994]


(a) 6.02 × 1023 atoms of C (b) 6.02 × 1023 atoms of O
(c) 18.1 × 1023 molecules of CO2 (d) 3 gm atom of CO2

19. The Avogadro Number or a mole represents [DCE-1994]


23 23
(a) 6.02 × 10 ions (b) 6.02 × 10 atoms
(c) 6.02 × 1023 molecules (d) 6.02 × 1023 entities

20. What is the weight of one molecule of a monatomic element X whose atomic
weight is 36? [DCE-94]
(a) 6.0 × 10–23 gm (b) 6.02 × 1023 gm (c) 36 × 1023 gm (d) 36 × 10–23 gm

21. Air contains 20% O2 for the reaction C2H5NH2 + O2  CO2 + NO2 + H2O. How much volume
of air will be required for 1 mol of C2H5NH2? [DCE-2007]
(a) 106.4 L (b) 1064 L (c) 212.8 L (d) 2128 L

22. Oxide of Mn contains 70% of Mn formula of this oxide will be [DCE-2007]


(a) Mn2O 3 (b) MnO (c) Mn3O 2 (d) Mn2 O

Amity Institute for Competitive Examinations : Phones: 24336143, 24336144, 25573111/12/13/14,95120-2431839/42 [ 21 ]


Basic Concept of Chemistry AICE (IIT-JEE)

23. Under certain thermodynamic conditions atomic wt of oxygen is 16.7, it contains two isotopes 16
8O
and 17 17
8 O. Percentage 8 O isotopes is [DCE-2007]
(a) 50 (b) 60 (c) 70 (d) 80

24. 40% by weight solution will contain how much mass of the solute in 1 litre solution, density of the
solution is 1.2 g/ml [DCE-2007]
(a) 480 g (b) 48 g (c) 38 g (d) 380 g

25. Two solutions of a substance (non electrolyte) are mixed in the following manner. 480 ml of 1.5 M
first solution + 520 mL of 1.2 M second solution. What is the molarity of the
final mixture? [AIEEE-2005]
(a) 1.20 M (b) 1.50 M (c) 1.344 M (d) 2.70 M

26. If we consider that 1/6, in place of 1/12, mass of carbon atom is taken to be the relative atomic mass
unit, the mass of one mole of substance will [AIEEE-2005]
(a) decrease twice
(b) increase two fold
(c) remain unchanged
(d) be a function of the molecular mass of the substance

27. 6.02 × 1020 molecules of urea are present in 100 ml of its solution. The concentration of urea
solution is [AIEEE-2004]
(a) 0.02 M (b) 0.01 M (c) 0.001 M (d) 0.1 M
(Avogadro constant, NA = 6.02 × 1023 mol–1)

28. To neutralise completely 20 mL of 0.1 M aqueous solution of phosphorous acid (H3PO3), the volume
of 0.1 M aqueous KOH solution required is [AIEEE-2004]
(a) 40 mL (b) 20 mL (c) 10 mL (d) 60 mL

29. 25 ml of a solution of barium hydroxide on titration with a 0.1 molar solution of hydrochloric acid
volume gave a of 35 ml. The molarity of barium hydroxide solution was [AIEEE-2003]
(a) 0.14 (b) 0.28 (c) 0.35 (d) 0.07

30. For an ideal gas, number of moles per litre in terms of its pressure P, gas constant R and temperature
T is [AIEEE-2002]
(a) PT/R (b) PRT (c) P/RT (d) RT/P

31. In a compound C, H and N atoms are present in 9 : 1 : 35 by weight. Molecular weight of compound
is 108. Molecular formula of compound is [AIEEE-2002]
(a) C2 H6 N 2 (b) C 3 H 4 N (c) C6 H8 N 2 (d) C 9 H 12 N 3

32. With increase of temperature, which of these changes? [AIEEE-2002]


(a) Molality (b) Weight fraction of solute
(c) Fraction of solute present in water (d) Mole fraction

Amity Institute for Competitive Examinations : Phones: 24336143, 24336144, 25573111/12/13/14,95120-2431839/42 [ 22 ]


Basic Concept of Chemistry AICE (IIT-JEE)

33. Number of atoms in 558.5 gram Fe (at wt. of Fe = 55.85 g mol–1) is [AIEEE-2002]
(a) twice that in 60 g carbon (b) 6.023 × 1022
(c) half that in 8 g He (d) 558.5 × 6.023 × 1023

34. How many moles of magnesium phosphate, Mg 3 (PO 4 )2 will contain 0.25 mole of oxygen
atoms? [AIEEE-2006]
–2 –2
(a) 2.5 × 10 (b) 0.02 (c) 3.125 × 10 (d) 1.25 × 10–2

35. In the reaction, 2Al(s) + 6HCl(aq)  2Al3+(aq) + 6Cl–(aq) + 3H2(g), [AIEEE-2007]


(a) 67.2 L H2(g) at STP is produced for every mole Al that reacts
(b) 11.2 L H2(g) at STP is produced for every mole HCl(aq) consumed
(c) 6 L HCl(aq) is consumed for every 3L H2(g) is produced
(d) 33.6 L H2(g) is produced regardless of temperature and pressure for every mole Al that reacts

36. The density (in g ML–1) of a 3.60 M sulphuric acid solution that is 29% H2SO4 (Molar mass =
98 g mol–1) by mass will be [AIEEE-2007]
(a) 1.22 (b) 1.45 (c) 1.64 (d) 1.88

37. A gaseous hydrocarbon gives upon combustion 0.72 g of water and 3.08 g, of CO2. The empirical
formula of the hydrocarbon is [JEE-Mains 2013]

(a) C6H5 (b) C7H8 (c) C2H4 (d) C3 H 4

38. The molarity of a solution obtained by mixing 750 mL of 0.5 (M)HCl with 250 mL of 2(M)
HCl will be: [JEE-Mains 2013]

(a) 1.75 M (b) 0.975 M (c) 0.875 M (d) 1.00 MN

39. At 300 K and 1 atm, 15 mL of a gaseous hydrocarbon requires 375 mL air containing 20% O2 by
volume for complete combustion. After combustion the gases occupy 330 mL. Assuming that the
water formed is in liquid form and the volumes were measured at the same temperature and pressure,
the formula of the hydrocarbon is: [JEE Mains 2016]
(a) C4H10 (b) C3H6 (c) C3H8 (d) C4 H 8

40. In neutral or faintly alkaline solution, 8 moles of permanganate anion quantitatively oxidize thiosulphate
anions to produce X moles of a sulphur containing product. The magnitude
of X is: [JEE Advanced 2016]

41. The most abundant elements by mass in the body of a healthy human adult are: Oxygen (61.4%);
Carbon (22.9%), Hydrogen (10.0%); and Nitrogen (2.6%). The weight which a 75 kg person whould
gain if all 1H atoms are replaced by 2H atoms is [JEE Mains 2017]
(a) 15 kg (b) 37.5 kg (c) 7.5 kg (d) 10 kg

42. 1 gram of a carbonate (M2CO3) on treatment with excess HCl produces 0.01186 mole of CO2. The
molar mass of M2CO3 in g mol–1 is [JEE Mains 2017]
(a) 1186 (b) 84.3 (c) 118.6 (d) 11.86

Amity Institute for Competitive Examinations : Phones: 24336143, 24336144, 25573111/12/13/14,95120-2431839/42 [ 23 ]


Basic Concept of Chemistry AICE (IIT-JEE)

43. The ratio of mass percent of C and H of an organic compound (CXHYOZ) is 6 : 1. If one molecule of
the above compound (CXHYOZ) contains half as much oxygen as required to burn one molecule of
compound C XH Y completely to CO 2 and H 2 O. The empirical formula of compound
CXHYOZ is [JEE Mains 2018]
(a) C2H4O3 (b) C3H6O3 (c) C2H4O (d) C3 H4 O 2

44. The amount of sugar (C 12 H 22 O 11 ) required to prepare 2 L of its 0.1 M aqueous


solution is: [JEE Mains 2019]
(a) 68.4g (b) 17.1g (c) 34.2g (d) 136.8g

45. A 10 mg effervescent tablet containing sodium bicarbonate and oxalic acid releases 0.25 ml of CO2
at T = 298. 15 K and p = 1 bar. If molar volume of CO2 is 25.0 L under such condition, what is the
percentage of sodium bicarbonate in each tablet? [JEE Mains 2019]
–1
[Molar mass of NaHCO3 = 84 g mol ]
(a) 16.8 (b) 8.4 (c) 0.84 (d) 33.6

46. 25 ml of the given HCl solution requires 30 mL of 0.1 M sodium carbonate solution. What is the
volume of this HCl solution required to titrate 30 mL of 0.2 M aqueous
NaOH solution? [JEE Mains 2019]
(a) 25 mL (b) 50 mL (c) 12.5 mL (d) 75 mL

47. Iodine reacts with concentrated HNO3 to yield Y along with other products. The oxidation state of
iodine in Y, is: [JEE Mains 2019]
(a) 5 (b) 3 (c) 1 (d) 7
48. 50 mL of 0.5 M oxalic acid is needed to neutralize 25 mL of sodium hydroxide solution. The amount
of NaOH in 50 mL of the given sodium hydroxide soluton is: [JEE Mains 2019]
(a) 40 g (b) 20 g (c) 80 g (d) 10 g
49. For the following reaction, the mass of water produced from 445 g of C57H110O6 is
2C57 H110 O 6 (s)  163O 2 (g)  110H 2 OP(l) [JEE Mains 2019]

(a) 495 g (b) 490 g (c) 890 g (d) 445 g


50. The hardness of a water sample (in terms of equivalents of CaCO3) containing 10–3 M CaSO4 is:
(molar mass of CaSO4 = 136 g mol–1) [JEE Mains 2019]
(a) 100 ppm (b) 50 ppm (c) 10 ppm (d) 90 ppm

51. The mass percentage of nitrogen in histamine is ______. [JEE Mains 2020]

52. 10.30 mg of O2 is dissolved into a liter of sea water of density 1.03 g/mL. The concentration of O2
in ppm is__________. [JEE Mains 2020]

53. Ferrous sulphate heptahydrate is used to fortify foods with iron. The amount (in grams) of the salt
required to achieve 10 ppm of iron in 100 kg of wheat is _____. [JEE Mains 2020]
Atomic weight : Fe = 55.85 ; S = 32.0 ; O = 16.00
54. 250 mL of 0.5 M NaOH was added to 500 mL of 1 M HCl. The number of unreacted HCl molecules
in the solution after complete reaction is _______ × 1021. (Nearest integer)
(NA = 6.022 × 1023) [JEE Mains 2021]

Amity Institute for Competitive Examinations : Phones: 24336143, 24336144, 25573111/12/13/14,95120-2431839/42 [ 24 ]


Basic Concept of Chemistry AICE (IIT-JEE)

55. To synthesise 1.0 mole of 2-methylpropan-2-ol from Ethylethanoate ________ equivalents of


CH3MgBr reagent will be required. (Integer value) [JEE Mains 2021]
56. 4g equimolar mixture of NaOH and Na2CO3 contains x g of NaOH and y g of Na2CO3. The value
of x is _______ g. (Nearest integer) [JEE Mains 2021]
57. Methylation of 10 g of benzene gave 9.2 g of toluene. Calculate the percentage yield of
toluene ______. (Nearest integer) [JEE Mains 2021]
58. If the concentration of glucose (C6H12O6) in blood is 0.72 g L–1, the molarity of glucose in
blood is __________ × 10–3 M. (Nearest integer)
[Given: Atomic mass of C = 12, H = 1, O = 16 u) [JEE Mains 2021]
59. The number of significant figures in 0.00340 is __________ . [JEE Mains 2021]
60. Sodium oxide reacts with water to produce sodium hydroxide. 20.0 g of sodium oxide is dissolved in
500 mL of water. Neglecting the change in volume, the concentration of the resulting NaOH solution
is ________ × 10–1 M. (Nearest integer) [Atomic mass : Na = 23.0, O = 16.0, H = 1.0]
[JEE Mains 2021]
61. The molarity of the solution prepared by dissolving 6.3 g of oxalic acid (H2C2O4.2H2O) in 250 mL
of water in mol L–1 is x × 10–2. The value of x is ________. (Nearest integer)
[Atomic mass : H : 1.0, C : 12.0, O : 16.0] [JEE Mains 2021]
62. The number of halogen/(s) forming halic (V) acid is __________. [JEE Mains 2021]
63. For a first order reaction, the ratio of the time for 75% completion of a reaction to the time for
50% completion is _______. (Integer answer) [JEE Mains 2021]



Amity Institute for Competitive Examinations : Phones: 24336143, 24336144, 25573111/12/13/14,95120-2431839/42 [ 25 ]


Basic Concept of Chemistry AICE (IIT-JEE)

CHAPTER TEST
SECTION - I: MULTIPLE CHOICE QUESTIONS (Single Correct)
1. A sample of ammonium phosphate (NH4)3PO4 contains 3.18 mol of H atoms. The number of mole
of O atoms in the sample is:
(a) 0.265 (b) 0.795 (c) 1.06 (d) 3.18

2. If 10 g of Ag reacts with 1 g of sulphur, the amount of Ag2S formed will be:


(a) 7.75 g (b) 0.775g (c) 11g (d) 10g

3. 1 mol of iron (Fe) reacts completely with 0.65 mol O2 to give a mixture of only FeO and Fe2O3. Mole
ratio of ferrous oxide to ferric oxide is:
(a) 3 : 2 (b) 4 : 3 (c) 20 : 13 (d) None of these

4. Mole fraction of ethyl alcohol in aqueous ethyl alcohol (C2H5OH) solution is 0.25. Hence percentage
of ethyl alcohol by weight is:
(a) 54% (b) 25% (c) 75% (d) 46%

5. XeF6 fluorinates I2 to IF7 and liberates Xenon (g). 210 mmol of XeF6 can yield a maximum of _____
mmol of IF7
(a) 420 (b) 180 (c) 210 (d) 245

SECTION - II: MULTIPLE CHOICE QUESTIONS (More than one correct)

6. Silver metal in ore is dissolved by potassium cyanide solution in the presence of air by the reaction

4Ag  8KCN  O 2  2H 2O 
 4K[Ag(CN) 2 ]  4KOH

(a) The amount of KCN required to dissolve 100 g of pure Ag is 120g


(b) The amount of oxygen used in this process is 0.742 g (for 100g pure Ag)
(c) The amount of oxygen used in this process is 7.40 g (for 100 g pure Ag)
(d) the volume of oxygen used at STP is 5.20 litres

7. If 27g of Carbon is mixed with 88g of Oxygen and is allowed to burn to produce CO2, then:
(a) Oxygen is the limiting reagent (b) Volume of CO2 gas produced at NTP is 50.4L
(c) C and O combine in mass ratio 3 : 8 (d) Volume of unreacted O2 at STP is 11.2 L

8.  A 3B2 (unbalanced)
A  B 

 A3 B2C2 (unbalanced)
A3 B2  C 

Above two reactions are carried out by taking 3 moles each of A and B and one mole of C. Then
which option is/are correct?
(a) 1 mole of A3B2C2 is formed (b) 1/2 mole of A3B2C2 is formed
(c) 1/2 mole of A3B2 is formed (d) 1/2 mole of A3B2 is left finally

Amity Institute for Competitive Examinations : Phones: 24336143, 24336144, 25573111/12/13/14,95120-2431839/42 [ 26 ]


Basic Concept of Chemistry AICE (IIT-JEE)

9. If 100 ml of 1 M H 2SO 4 solution is mixed with 100 ml of 9.8% (w/w) H 2SO 4 solution
(d = 1 g/ml) then:
(a) concentration of solution remains same (b) volume of solution become 200 ml
(c) mass of H2SO4 in the solution is 98 g (d) mass of H2SO4 in the solution is 19.6 g

10. Equal volume of 0.1 M NaCl and 0.1 M FeCl2 are mixed with no change in volume due to mixing.
Which of the following will be true for the final solution. (No precipitation occurs). Assume complete
dissociation of salts and neglect any hydrolysis.
(a) [Na+] = 0.05 M (b) [Fe2+] = 0.05 M (c) [Cl–] = 0.3 M (d) [Cl–] = 0.15 M

SECTION - III: INTEGER TYPE QUESTIONS

11. Consider the following reaction involved in the preparation of teflon polymer ( CF2  CF2
)n

XeF6  ( CH2  CH 2


)n 
 ( CF2  CF2
) n  HF  XeF4
Determine the moles of XeF6 required for preparation of 100g Teflon.

60%
12. Cl2  KOH   KCl  KClO  H 2O
50%
KClO   KCl  KClO3
80%
KClO3   KClO4  KCl

112 L Cl2 gas at STP is passed in 10 L KOH solution containing 1 mole of potassium hydroxide per
liter. Calculate the total moles of KCl produced, rounding it off to nearest whole number. (Yield of
chemical reactions are written above the arrow () of respective reaction.

13. A 2  2B2 
 A 2 B4
Calculate the total mass in the reaction mixture if 4 moles of A2 and 4 moles of B2 are taken in a
reaction container.

14. Calculate the Cl– concentration in mol per litre in solution which is obtained by mixing one mole each
of BaCl2, NaCl and HCl in 500 ml water.

15. How many moles of Zn(FeS2) can be made from 2 mole zinc, 3 mole iron and 5 mole sulphur?

16. A solution containing 0.1 mol of a metal chloride MClx requires 500 ml of 0.8 M AgNO3 solution for

complete reaction MCl x  xAgNO3 


 xAgCl  M(NO3 ) x . Then the value of x is

17. A 3 : 2 molar ratio mixture of FeO and Fe2O3 reacts with oxygen to produce a 2 : 3 molar ratio
mixture of FeO and Fe2O3. Find the mass (in g) of O2 gas required per mole of the initial mixture.

18. A piece of Al weighing 27g is reacted with 200 ml of H2SO4 (Specific gravity = 1.8 and 54.5% by
weight). After the metal is completely dissolved 73g HCl is added and solution is further diluted to
500 ml solution then find the concentration of H+ ion in mol/litre.

Amity Institute for Competitive Examinations : Phones: 24336143, 24336144, 25573111/12/13/14,95120-2431839/42 [ 27 ]


Basic Concept of Chemistry AICE (IIT-JEE)

SECTION - IV: MATRIX MATCH

19. Match the following:

Column - I Column - II
(A) 100 ml of 0.2 M AlCl3 solution +400 ml (p) Total concentration of cation(s) = 0.12 M
of 0.1 M HCl solution

(B) 50 ml of 0.4 M KCl + 50 ml H2O (q) [SO 24 ]  0.06M

(C) 30 ml of 0.2 M K2SO4 + 70 ml H2O (r) [SO 24 ]  2.5M

(D) 200 ml 24.5% (w/v) H2SO4 (s) [Cl–] = 0.2 M


(a) (A) – (p), (s); (B) – (s);(C) – (p), (q), (D) – (q)
(b) (A) – (r), (B) – (q) (C) – (s), (D) – (p)
(c) (A) – (q); (B) – (p), (C) – (r), (D) – (s)
(d) (A) – (r); (B) – (s); (C) – (p), (D) – (q)

SECTION - IV: COMPREHENSION TYPE QUESTIONS


Comprehension

According to the Avogadro’s law, equal number of moles of gases occupy the same volume at identical
condition of temperature and pressure. Even if we have a mixture of non-reacting gases then Avogadro’s law
is still obeyed by assuming mixture as a new gas.

Now let us assume air to consist of 80% by volume of Nitrogen (N2) and 20% by volume of oxygen (O2). If
at is taken at STP then its 1 mole would occupy 22.4 L. 1 mole of air would contain 0.8 mol of N2 and 0.2 mol

of O2 hence the mole fractions of N2 and O2 are given by X N2  0.8, X O2  0.2

20. Volume occupied by air at NTP containing exactly 11.2 g of Nitrogen:


(a) 22.4 L (b) 8.96 L (c) 11.2 L (d) 2.24 L

21. If air is treated as a solution of O2 and N2 then % W/W of oxygen is

10 200 700 350


(a) (b) (c) (d)
9 9 9 9
22. Density of air at NTP is

9 2
(a) 1 g/L (b) g/L (c) g/L (d) can’t be determined
7 7



Amity Institute for Competitive Examinations : Phones: 24336143, 24336144, 25573111/12/13/14,95120-2431839/42 [ 28 ]


Basic Concept of Chemistry AICE (IIT-JEE)

ANSWERS
MULTIPLE CHOICE QUESTIONS (Single Correct )

1. (b) 2. (c) 3. (a) 4. (d) 5. (c)


6. (b) 7. (c) 8. (c) 9. (d) 10. (b)
11. (b) 12. (a) 13. (a) 14. (a) 15. (a)
INTEGER TYPE QUESTIONS

16. (3) 17. (1) 18. (4) 19. (8) 20. (2)
21. (4) 22. (6) 23. (4) 24. (4) 25. (3)
MULTIPLE CHOICE QUESTIONS (More than one Correct )

26. (c,d) 27. (b,c,d) 28. (a,b,d) 29. (a,c) 30. (a,c)
31. (b,c) 32. (b,c,d) 33. (a,c,d) 34. (a,c) 35. (a, b)
MATRIX MATCH TYPE QUESTIONS

36. A-(s); B-(q); C-(p); D-(r) 37. A-(s); B-(r); C-(p); D-(q)
COMPREHENSION TYPE QUESTIONS
38. (a) 39. (c) 40. (a) 41. (b) 42. (c)
43. (a)
PREVIOUS YEAR QUESTIONS
1. (d) 2. (c) 3. (a) 4. (b) 5. (c)
6. (a) 7. (8) 8. (4) 9. (8) 10. (b)
11. (b) 12. (a) 13. (b) 14. (c) 15. (b)
16. (a) 17. (c) 18. (a) 19. (d) 20. (a)
21. (b) 22. (a) 23. (c) 24. (a) 25. (c)
26. (b) 27. (b) 28. (a) 29. (d) 30. (c)
31. (c) 32. (c) 33. (a) 34. (c) 35. (b)
36. (a) 37. (b) 38. (c) 39. Data wrong 40. (6)
41. (c) 42. (b) 43. (a) 44. (a) 45. (a)
46. (a) 47. (a) 48. Bonus 49. (a) 50. (a)
51. 37.80 to 38.20 52. 10 53. 4.95 to 4.97 54. (226) 55. (2)
56. (1) 57. (78) 58. (4) 59. (3) 60. (13)
61. (20) 62. (3) 63. (2)

CHAPTER TEST
1. (c) 2. (a) 3. (b) 4. (d) 5. (b)
6. (a, c, d) 7. (b, c, d) 8. (b, d) 9. (a, b, d) 10. (a,b,d)
11. (4) 12. (4) 13. (2) 14. (8) 15. (2)
16. (4) 17. (2) 18. (6) 19. (a) 20. (c)
21. (b) 22. (b)

Amity Institute for Competitive Examinations : Phones: 24336143, 24336144, 25573111/12/13/14,95120-2431839/42 [ 29 ]


Basic Concept of Chemistry AICE (IIT-JEE)

HINTS AND SOLUTIONS


MULTIPLE CHOICE QUESTIONS (Single Correct)

1. (b) A  2B 
 C
1 mole 2 mole 1 mole
–––––––––––––––––––––––––––––––––––––
5 mole 8 mole
5 mole of A reacts with 10 moles of B
Therefore, B will be the limiting reagent
Moles of C produced = 4

 4x  y  y
2. (c) C x H y    O 2 
 xCO2  H 2O
 4  2

5 volume of hydrocarbon on complete combustion consumes 10 volume oxygen giving 5 volume


of CO2 means
x=1
4x  y
2 y=4
4
Hydrocarbon is CH4

1
3. (a) Zn  O2 
 ZnO
2
Equivalent of Zn = Equivalent of O

x  2 VO2

65 5.6
2 x  5.6
VO2  litre
65
4. (d) M1V1 = M2V2

WHCl  1000
5. (c) m  M  W
HCl solvent in gm

6. (b) N 2  3H 2 
 2NH3
x y 0
–a –3a +2a
––––––––––––––––––––––––––––––
x–a y – 3a 2a
––––––––––––––––––––––––––––––

2a = 2 mole x  a = 2 mole y  3a = 2 mole


a = 1 mole x = 3 mole y = 2 + 3 = 5 mole

Amity Institute for Competitive Examinations : Phones: 24336143, 24336144, 25573111/12/13/14,95120-2431839/42 [ 30 ]


Basic Concept of Chemistry AICE (IIT-JEE)

1
7. (c) 5.6 lit O2 = 1 gm equivalent of O2 = 8g O2 = mole O2
4
67200  0.33
8. (c) Weight of Fe in Haemoglobin =  221.76g
100
221.76
No. of moles of Fe =  4 mole
56
In 1 mole Haemoglobin 4 mole Fe is present.

1 Molecule of haemoglobin 4 atoms of Fe are present.

9. (d) 3BaCl2  2Na 3 PO4 


 Ba 3 (PO4 ) 2  6NaCl
3 mole 2 mole 1 mole
––––––––––––––––––––––––––––––––––
0.5 mole 0.2 mole
Na3PO4 is the limiting reagent.
 Moles of Ba3(PO4)2 produced = 0.1

10. (b) C3H 8  5O2 


 3CO2  4H2O
x lit 3 x litre
13
C4 H10  O2 
 4CO2  5H2O
2
(3 – x) lit (12 – 4x) lit
12 – 4x + 3x = 10
x = 2 lit

11. (b) CO 2  2NaOH 


 Na 2CO3  H 2
 20g NaOH reacts with 0.25 moles of CO2
 No. of moles of CO2 after oxidation of all CO into CO2 = 1 mole
 Therefore, weight of NaOH required is 80 g

12. (a) N1V1 + N2V2 = N3V3

13. (a) Molecular weight of virus = N0 × mass of one virus

14. (a) N1V1 + N2V2 = N3V3

15. (a) Zn  H 2SO 4 


 ZnSO 4  H 2 

Zn  2NaOH 
 Na 2 ZnO2  H 2 

INTEGER TYPE QUESTIONS

16. (3) Equivalent of acid = Equivalent of Ba(OH)2

Weight of polybasic acid  n-factor


 (N  V) Ba (OH)2
Molecular weight of polybasic acid

Amity Institute for Competitive Examinations : Phones: 24336143, 24336144, 25573111/12/13/14,95120-2431839/42 [ 31 ]


Basic Concept of Chemistry AICE (IIT-JEE)

Weight of H 2O 2 N  Equivalent weight


17. (1) % strength =  100 
Volume of solution 10

18. (4) CuSO4.5H2O is [Cu(H2O)4]SO4.H2O

19. (8) n-factor of salt is total no. of positive charge / Total no. of negative charge

6n  2 2n  2
20. (2) Cn H 2n 2  O2 
 nCO 2  H 2O
4 2
21. (2) Meq. of Na2CO3 = 25
Meq. of Acid ‘A’ = 25 = N × 10
N = 2.5 N
Meq. of Acid ‘B’ = 25 = N × 40
N = 0.625 N
NAVA + NB VB = NMVM
2.5 × VA + 0.625 (1 – VA) = 1 × 1
VA = 0.2
VB = 0.8
VB
4
VA

22. (4) No. of equivalent of oxygen = no. of equivalent of K2Cr2O7 in 1 M

23. (6) In ZnCr2Ox mole ratio of Zn, Cr and O is 1 : 2 : x


Zn Cr O
Moles 0.15 0.30 0.60
1 : 2 : 4
x=4

24. (4) [OH–] = N × 

0.08
α
2

0.08
% dissocaiton =  100  4%
2

25. (2) N = M × n-factor


n-factor for H3PO3 = 2
MULTIPLE CHOICE QUESTIONS (More than one Correct )

26. (c,d)

CO 2  2NaOH 
 Na 2 CO3  H 2 
1 mole 2 mole

Amity Institute for Competitive Examinations : Phones: 24336143, 24336144, 25573111/12/13/14,95120-2431839/42 [ 32 ]


Basic Concept of Chemistry AICE (IIT-JEE)

27. (a,b,c)

2VO  3Fe2O3 
 6FeO  V2 O5
28. (b,c,d)
Basicity of H3PO3 is two
2NaOH  H 3PO3   Na 2 HPO3  2H 2 O
29. (a,b,d)

SO 2 Cl2  H2O 
 H 2SO 4  2HCl
2.70 g
0.02 mole 0.02 mole 0.04 mole
30. (a,c)
3.42 g Al2(SO4)3 = 0.01 mole Al2(SO4)3
1.42 g Na2SO4 = 0.01 mole Na2SO4
31. (a,c)
Mass of carbon
% of C = 100
Molar mass
32. (a,b,c,d)
During neutralisation
Equivalent of acid = Equivalent of base
33. (a,b,c)
Volume strength 28
M   2.5M
11.2 11.2
W 2.5  34
%   100  8.5%
V 1000
2.5
x H 2O 2   0.2
2.5  10
2.5
m 1000  13.88
180
34. (a,b,c,d)
H3PO4  X(OH)2 
 XHPO4  2H2O
Basicity = 2 Acidity =2
35. (a,c,d)
1g molecule of V2O5 = 1 mole of V2O5

MATRIX MATCH TYPE QUESTIONS

36. A-(s); B-(q); C-(p); D-(r)


Moles of KOH = 4

(NH 4 )3 PO 4  3KOH 
 K 3PO 4  3NH 3  3H 2O
149 gm
1 mole 4 mole
Since (NH4)3PO4 is the limiting reagent
Volume of NH3 evolved = moles of NH3 × 22.4
= 3 × 22.4 = 67.2 lit

Amity Institute for Competitive Examinations : Phones: 24336143, 24336144, 25573111/12/13/14,95120-2431839/42 [ 33 ]


Basic Concept of Chemistry AICE (IIT-JEE)

Similarly volume of NH3 evolved can be calculated from other ammonium salt by the reaction with
KOH.

37. A-(s); B-(r); C-(p); D-(q)

Eq.wt. of Sulphite of X = E X  ESO23  40  40  80

Eq. wt. of Sulphite of Y = E Y  ESO32  80  40  120

Eq. wt. of Sulphite of X 80


  0.67
Eq. wt. of Sulphite of Y 120
Similarly the ratio of equivalent weight of X and Y of the other corresponding compounds can be
calculated.
COMPREHENSION TYPE QUESTIONS
38. (a)
Weight of HCl remain in solution = 18.25 – 2.75 = 15.50g

15.50 1000
N  0.58N
36.50  750

39. (c)
Equivalent of Ca(OH)2 = Equivalent of HCl
N1V1 = N2V2
0.2 × V = 10 × 0.1

1
V  5 ml
0.2

40. (a)
N = M × n-factor
41. (b)
WSolub le
% by weight = 100
WSolution
42. (c)
nNa 2S2O3
xNa 2S2O3 
nNa 2S2O3  nH2O

43. (a)

1000M
m
1000d  MM1

M = Molarity of solution
d = density of solution
M1 = Molar mass of solute



Amity Institute for Competitive Examinations : Phones: 24336143, 24336144, 25573111/12/13/14,95120-2431839/42 [ 34 ]


Chapter - 2
REDOX REACTIONS
2.1 Oxidation-Reduction
(1) Oxidation is a process which involves loss of electrons, i.e., de-electronation. It is an
endothermic process.
(2) Reduction is a process which involves gain of electrons, i.e., electronation. It is an exothermic
process.
Oxidation Reduction
M  M+n + ne– M+n + ne–  M
A–n  A + ne– A + ne–  A– n
M  n1 
 M  n 2  (n 2  n 1 )e M  n1  (n1  n 2 )e 
 M n2
(3) Oxidants are substances which :
(a) oxidize other substances
(b) are reduced themselves
(c) show electronation
(d) show a decrease in oxidation no. during a redox change.
(e) has higher oxidation no. in a conjugate pair redox reaction.
(4) Reductants are substances which
(a) reduce other substances
(b) are oxidized themselves
(c) show de-electronation
(d) show an increase in oxidation no. during a redox reaction
(e) have lower oxidation no. in a conjugate pair of redox reaction
(5) A redox change is one in which a reductant is oxidized to liberate electrons, which are then used
up by an oxidant to get itself reduced.

M1  M 1 n + ne– Oxidation
M 2 n + ne–  M2 Reduction
M1 + M 2 n  M 1 n + M2 Redox reaction

(6) A redox change occurs simultaneously.


Oxidation and Reduction can also be defined in many ways,
(i) Removal of hydrogen atoms is oxidation while addition of hydrogen atoms is reduction. For
example,

H2 + Cl2 2HCl; Cl2 is reduced to HCl


H2S + Cl2 2HCl + S; H2S is oxidised to S

(ii) Addition of oxygen is oxidation, while removal of oxygen is reduction. For example

C + O2 CO2 C is oxidised to CO2


CuO + H2 Cu + H2O CuO is reduced to Cu

Amity Institute for Competitive Examinations : Phones: 24336143, 24336144, 25573111/12/13/14,95120-2431839/42 [ 35 ]


Redox Reactions AICE (IIT-JEE)

(iii) Increase in positive valency of an element is oxidation, while decrease in positive valency of an
element is reduction.

FeCl2 FeCl3 change in valency of Fe2+  Fe3+

KMnO4 MnO2 change in valency of Mn+7  Mn+4

(iv) Addition of an electronegative element is oxidation while its removal is reduction. For example,

2Na + Cl2 2NaCl (oxidation of Na)

FeCl3 FeCl2 + 1/2 Cl2 (reduction of Fe)

(v) Addition of electropositive element is reduction while its removal is oxidation. For example,

HgCl2 + Hg Hg2Cl2 (reduction of HgCl2)

K4[Fe(CN)6] K3[Fe(CN)6] (oxidation of Fe in K4 [Fe(CN)6])

(vi) According to Franklin, removal or loss of electrons (de-electronation) is oxidation, while addition
or gain of electrons (electronation) is reduction. For example

Na Na+ + e– (oxidation)

Cl + e– Cl (reduction)

(vii)Increase in oxidation number is oxidation, while decrease in oxidation number is reduction. For
example,
+2 +4
C CO2 (change in O.N.; + 2  +4, hence oxidation)
+2 +1
CuI2 Cu2I2 (change in O.N.; + 2  + 1, hence reduction

Oxidation Number

(1) Oxidation No. of an element in a particular compound represents the no. of electrons lost or
gained by an element during its change from free state into that compound or Oxidation No. of
an element in a particular compound represent the extent of oxidation or reduction of an element
during its change from free state into that compound.

(2) Oxidation No. is given positive sign if electrons are lost. Oxidation No. is given negative sign if
electrons are gained.

(3) Oxidation No. represents real charge in case of ionic compounds. However, in covalent compounds
it represents imaginary charge.

Rules for Deriving Oxidation Number

Following rules have been arbitrarily adopted to decide oxidation no. of elements on the basis of their
periodic properties:

(1) In uncombined state or free state, oxidation no. of an element is zero.

(2) In combined state oxidation no. of:

Amity Institute for Competitive Examinations : Phones: 24336143, 24336144, 25573111/12/13/14,95120-2431839/42 [ 36 ]


Redox Reactions AICE (IIT-JEE)

(a) F is always –1.

1
(b) O is –2 in oxides; In peroxides it is –1, in super oxides  . However in OF2 it is +2 and
2
O2F2 it is +1. In ozonides it is –1/3.

(c) H is +1. In ionic hydrides it is –1.

(d) Metals always show +ve. Oxidation number.

(e) Alkali metals (i.e., I A group – Li, Na, K, Rb, Cs and Fr) is always +1.

(f) Alkaline earth metals (i.e., II A group–Be, Mg, Ca, Sr, Ba and Ra) are always +2.

(3) The algebraic sum of all the oxidation no. of all the elements in a compound is equal to zero,
e.g., K2CrO4 (O.N. of K) × 2 + (O.N.of Cr) + (O.N. of O)  4 = 0

(4) The algebraic sum of oxidation no. of all the elements in an ion is equal to the net charge on the
ion, e.g., in SO42– O.N.of S + (O.N. O) × 4 = –2.

(5) Oxidation number can be an integer or fraction, (+)ve or (–)ve.

1. Oxidation number of Mn in KMnO4. Let the oxidation number of Mn be x. Now we know


that the oxidation numbers of K is +1 and that of O is –2.

K Mn O4
+1 +x +4  –2 = +1+x –8

Now the sum of oxidation numbers of all atoms in the formula of the compound must be zero, i.e.
+1 + x – 8 = 0

Hence the oxidation number of Mn in KMnO4 is +7.

2. Oxidation number of chromium in potassium dichromate. Writing the known oxidation


number of potassium and oxygen

K2 Cr2 O7 or K2 Cr2 O7
+2  1 +2x +7(–2) +2 +2x –14

Since sum of oxidation numbers = 0, i.e.

2 + 2x – 14 = 0
2x = 14 – 2
x = +6

3. Oxidation number of S in S8. Since in S8, sulphur is present in free state, its oxidation number
is zero.

Formal Charge

Formal charge on an atom in a compound can be defined as the calculated charge present on the
atom in a compound. Thus the formal charge present on nitrogen atom in NH4+ ion is +1, and that on
nitrogen and boron atoms in the co-ordination compound H3N  BF3 is +1 an –1 respectively. The
formal charge can be calculated as below.

Amity Institute for Competitive Examinations : Phones: 24336143, 24336144, 25573111/12/13/14,95120-2431839/42 [ 37 ]


Redox Reactions AICE (IIT-JEE)

1
Formal charge = z – u – s
2

Where z = Number of outer (valence shell) electrons in the atom.


u = Number of unshared electrons
s = Number of shared electrons around the atom in the compound

1
Thus the formal charge on nitrogen atom in NH4+ (z = 5, u = 0, s = 8) = 5 – 0 – 8=1
2
1
Similarly, formal charge on S in SO2 = 6 – 2 – 4=2
2

Balancing of Redox Equations by Oxidation Number Method


This method is based on the fact that the number of electrons gained during reduction must be equal
to the number of electrons lost during oxidation. Following steps must be followed while balancing
redox equations by this method.

(i) Write the skeleton equation (if not given, frame it) representing the chemical change.

(ii) With the help of oxidation number of elements, find out which atom is undergoing oxidation/
reduction, and write separate equations for the atom undergoing oxidation/reduction.

(iii) Add the respective electrons on the right for oxidation and on the left for reduction equation.
Note that the net charge on the left and right sides should be equal.

(iv) Multiply the oxidation and reduction reactions by suitable integers so that total electrons lost in
one reaction is equal to the total electrons gained by other reaction.

(v) Transfer the coefficients of the oxidising and reducing agents and their products as determined
in the above step to the concerned molecule or ion.

(vi) By inspection, supply the proper coefficient for the other formulae of substances not undergoing
oxidation and reduction to balance the equation.

Balancing of Redox equation by Ion-Electron Method


This method involves the following steps:

I Divide the complete equations into two half reactions:

(a) One representing oxidation.

(b) The other representing reduction.

II Balance the atoms in each half reaction separately according to the following steps:

(a) Balance all atoms other than oxygen and hydrogen.

(b) To balance oxygen and hydrogen.

(1) In Acidic Medium:

(i) Add H2O to oxygen deficient side to balance oxygen atoms.

(ii) Add 2H+ for each added H2O to hydrogen deficient side to balance H atoms.

Amity Institute for Competitive Examinations : Phones: 24336143, 24336144, 25573111/12/13/14,95120-2431839/42 [ 38 ]


Redox Reactions AICE (IIT-JEE)

(2) In Basic Medium:

(i) For each excess of oxygen, one H2O is added on the same side and 2OH– on the other side.

(ii) If H - is not balanced, then for each excess of H, one OH– is added on the same side and
one H2O on the other side.

Balancing of Molecular Reactions by Oxidation Number Method


1. First of all the oxidation number of all those atoms are calculated which are changing during the
reactions.

2. Oxidising and reducing agents are identified.

3. Total number of lost and total number of gained electrons are counted.

4. Now, cross-multiplication is done to find the co-efficients of the reactants and products.

Oxidising agent No. of gained electrons



Reducing agent No. of lost electrons
5. In case of alkaline reactions, balancing should be done from right hand side.

6. If the same atoms are present in more than one molecule either on L.H.S. or R.H.S., then
balancing should be done from the side having more molecules having same atom in different
oxidaion number.

Balance the following:

i) KMnO 4  HCl   KCl  MnCl 2  Cl 2  H 2O


ii) K 2 Cr2 O 7  HCl   KCl  CrCl3  Cl 2  H 2 O
iii) Al  NaOH  H 2 O   NaAlO 2  H 2
iv) MnO 4  Fe2   H    Mn 2   Fe3  H 2 O
v) Cr2 O 72   I   H   Cr 3  I 2  H 2 O
vi) MnO 4  C 2 O 42   H   Mn 2   CO 2  H 2 O

OH
vii) Br2   Br   BrO3

viii) Cu  NO3 
H
 Cu 2   NO 2  H 2 O

ix) Cu  NO3 
H
 Cu 2   NO 2  H 2 O
x) FeCl3  SnCl 4 
 FeCl 2  SnCl 4

Illustration 1: Balance the following reaction


NO3– + H2S HSO4– + NH4+ in acidic medium

Solution: Here NO3– is undergoing reduction to NH4 and H2S undergoes oxidation to HSO4–
(i) NO3–  NH4+
(ii) H2S  HSO4–
by balancing each half reaction, we get
(iii) NO3– + 8e– + 10 H+ NH4 + 3H2O
(iv) H2S + 4H2O HSO4– + 8e– + 9H
by combining the equation, we get
NO3– + H2S + H2O + H+ NH4+ + HSO4–

Amity Institute for Competitive Examinations : Phones: 24336143, 24336144, 25573111/12/13/14,95120-2431839/42 [ 39 ]


Redox Reactions AICE (IIT-JEE)

Illustration 2: Balance the following reaction


NO3– + Al Al3+ + NH4in basic medium

Solution: Here NO3– is undergoing reduction and Al3+ is undergoing oxidation


(i) NO3– NH4+
(ii) Al Al3+
by balancing each half reaction, we get
(iii) NO3– + 7H2O + 8e– NH4++ 10 OH–
(iv) Al Al3+ + 3e–
by multiplying equation (iii) by 3 and equation (iv) by 8, we get
(v) 3NO3– + 7H2O + 24e– 3NH4++ 30 OH–
(vi) 8Al 8Al3+ + 24e–
by combining these equations, we get
8Al + 3NO3– + 21H2O 8Al3+ + 3NH4++ 30 OH–

Illustration 3: Calculate the equivalent weight of each oxidant and reductant in the following reactions.
(a) H3PO2 PH3 + H3PO3
(b) KI + K2Cr2O7 Cr+3 + 3I2
(c) Na2S2O3 + I2  Na2S4O6 + 2NaI

Solution : (a) H3PO2  PH3 + H3PO3


P+ + 4e–  P3–
(P+ P3+ + 2e–) × 2
Net reaction 3P+  P3– + 2P+3
3H3PO2 PH3 + 2H3PO3
n-factor for H3PO2 = 4/3
M 3M
Equivalent weight of H3PO2 = 
4/3 4
(b) 2I– I2 + 2e–
equivalent weight of KI = M/1
= 166/1
= 166
6e– + (Cr+6)2 2Cr+3
equivalent weight of K2Cr2O7
= M/6
= 294/6 = 49
(c) 2(S+2)2  S45/2 + 2e–
M 158
equivalent weight of Na2S2O3 = = = 158
1 1

Amity Institute for Competitive Examinations : Phones: 24336143, 24336144, 25573111/12/13/14,95120-2431839/42 [ 40 ]


Redox Reactions AICE (IIT-JEE)

SAMPLE PROBLEMS 2.1 (MCQ)


Problem 1: Both oxidation and reduction takes place in:

(a) NaBr  HCl 


 NaCl  HBr (b) HBr  AgNO3 
 AgBr  HNO3

(c) H 2  Br2 
 2HBr (c) CaO  H 2SO 4 
 CaSO 4  H 2O

Solution: (c) Both oxidation and reduction are taking place in:

Oxidised (Reducing agent)

0 0 1  1
H 2  Br2 
 2HBr

Reduced (oxidising agent)

O
O O
Problem 2: CrO5 has structure as shown Cr
O O
The oxidation number of chromium in the above compound is
(a) 4 (b) 5
(c) 6 (d) 0
()
(1) O (1)
O O
Solution: (c) Cr
O (+6) O
(1) (1)

Problem 3: In the conversion of Br2 to BrO3 , the oxidation state of bromine changes from

(a) –5 to +5 (b) –5 to 0
(c) 0 to +5 (d) +5 to –5

Solution: (c) BrO3 x + 3 (–2) = –1

x=+5
Problem 4: The sum of the oxidation numbers of the atoms in C6H5CHO is
(a) –1 (b) 0
(c) +1 (d) +2

Solution: (b)

Problem 5: In Ba(H2PO2)2, the oxidation number of phosphorus is


(a) –1 (b) 0
(c) +1 (d) +3

Solution: (c) (H 2 PO2 )  2 (+1) + x + 2 (–2) = –1

x = +1

Amity Institute for Competitive Examinations : Phones: 24336143, 24336144, 25573111/12/13/14,95120-2431839/42 [ 41 ]


Redox Reactions AICE (IIT-JEE)

Problem 6: Which of the following is not a redox reaction?


(a) KCN  Fe(CN)2 
 K 4 [Fe(CN) 6 ]
(b) Rb  H 2O 
 RbOH  H 2
(c) H 2 O2 
 H2O  O
(d) CuI 2 
 CuI  I 2

Solution: (a) KCN  Fe(CN)2 


 K 4 [Fe(CN) 6 ]

This is not redox reaction

Problem 7: In which of the following oxidation state shows by the iodine?


(a) +1, –1, +3, +5 (b) +1, –1, +3, +5, +7
(c) +1, +3, +5 (d) +1, +3, +7

Solution: (b) Because the oxidation state of I is–1 to +7. So the minimum –1 and maximum + 7

Problem 8: aK 2Cr2O7  bKCl  cH 2SO4 


 xCrO 2 Cl2  yKHSO4  zH 2O
The above equation balances when
(a) a = 2, b = 4, c = 6 and x = 2, y = 6, z = 6
(b) a = 4, b = 2, c = 6 and x = 6, y = 2, z = 3
(c) a = 6, b = 4, c = 2 and x = 6, y = 3, z = 2
(d) a = 1, b = 4, c = 6 and x = 2, y = 6, z = 3

Solution: (d) K 2Cr2O7  4KCl  6H 2SO 4 


 2CrO 2 Cl2  6KHSO4  3H 2O

Problem 9: In the reaction:

As 2S5  xHNO3 
 5H 2SO4  yNO2  2H 2 AsO4  12H 2O

the value of x and y are


(a) 40, 40 (b) 10, 10
(c) 30, 30 (d) 20, 20

Solution: (a) As 2S5  40HNO3 


 5H 2SO4  40NO2  2H3AsO4  12H2 O

Problem 10: Which one of the following reactions involves disproportionation?


(a) 2H 2SO 4  Cu 
 CuSO4  2H2 O  SO 2
(b) As 2O3  3H 2S 
 As2S3  3H 2O
(c) 2KOH  Cl2   KCl  KOCl  H2 O
(d) Ca 3P2  6H 2 O  3Ca(OH) 2  2PH 3
Oxidation

(0) ( 1) ( 1) (0)


2KOH  Cl2 
 K Cl  KO Cl  H 2 O
Solution: (c)
Reduction

Amity Institute for Competitive Examinations : Phones: 24336143, 24336144, 25573111/12/13/14,95120-2431839/42 [ 42 ]


Redox Reactions AICE (IIT-JEE)

2.2 Volumetric Analysis


The volumetric analysis is an analytical method of determining the concentration of a substance in a
solution by adding exactly same number of equivalents of another substance present in a solution of
known concentration.

This is the basic principle of titration. Volumetric analysis is also known as titrimetric analysis.

The substance whose solution is employed to estimate the concentration of unknown solution is
called titrant and the substance whose concentration is to be estimated is called titrate.

The volumetric analysis is divided into following types:

(a) Simple titrations (Acid-Base or Neutralisation)

(b) Back titrations

(c) Double titrations

Simple Titrations

The aim of simple titration is to find the concentration of an unknown solution with the help of the
known concentration of another solution.

Let us take a solution of a substance ‘A’ of unknown concentration. We are provided with solution of
another substance ‘B’ whose concentration is known (N1). We take a certain known volume (V2
litre) of ‘A’ in a flask and start adding ‘B’ from burette to ‘A’ slowly till all the ‘A’ is consumed by ‘B’.
This can be known with the help of suitable indicator, which shows colour change after the complete
consumption of ‘A’. Let the volume of B consumed is V1 litre. According to the law of equivalents,
the number of equivalents of ‘A’ would be equal to the number of equivalents of ‘B’.

 N1V1 = N2V2, where N2 is the concentration of ‘A’.

Thus using this equation, the volume of A2 can be calculated.

There are four types of simple titrations, namely

(a) Acid-base titrations

(b) Redox titrations

Acid-Base Titrations

In this type of titration, the concentration of an acid in a solution is estimated by adding a solution of
standard base and vice versa. The equivalence point is detected by adding a few drops of a suitable
indicator to the solution whose concentration is to be estimated. An acid-base indicator gives different
colours with acids and bases. The choice of indicator in a particular titration depends on the pH-
range of the indicator and the pH change near the equivalence point. End point is indicated by the
change in colours of indicators, depending upon pH and structure of the indications.

Amity Institute for Competitive Examinations : Phones: 24336143, 24336144, 25573111/12/13/14,95120-2431839/42 [ 43 ]


Redox Reactions AICE (IIT-JEE)

(i) Strong acid-strong base Titration: In the titration of HCl Vs NaOH, the equivalence point
lies in the pH-range of 4-10. Thus, methyl red (pH-range 4.2 to 6.3), methyl orange (pH-range
3.1 to 4.4) and phenolphthalein (pH-range 8.3 - 10) are suitable indicators of such titrations.

(ii) Weak acid-strong base Titration: In the titration of CH3COOH and NaOH, the equivalence
point lies between 7.5 and 10. Thus, phenolphthalein is the suitable indicator.

(iii) Weak base-strong acid Titration: In the titration of NH4OH and HCl, the equivalence point
lies in the pH range of 4 to 6.5. Thus, methyl orange and methyl red are suitable indicators.

(iv) Weak acid-weak base Titration: In the titration of CH3COOH and NH4OH, the equivalence
point lies between 6.5 and 7.5 and the pH change is not sharp at the equivalence point. Thus, no
simple indicator can be employed to detect the equivalence point.

Redox Titrations
In a redox titration, an oxidant is estimated by adding reductant or vice-versa. For example, Fe2+ ions
can be estimated by titration against acidified KMnO4 solution when Fe2+ ions are oxidised to Fe3+
ions and KMnO4 is reduced to Mn2+ in the presence of acidic medium. KMnO4 functions as self-
indicator as its purple colour is discharged at the equivalence point.

MnO4– + 8H+ + 5e–  Mn2+ + 4H2O


Fe2+  Fe3+ + e–] × 5
______________________________________
MnO4– + 8H+ + 5Fe2+ Mn2+ + 5Fe3+ + 4H2O
(n = 5)

In addition to acidified KMnO4, acidified K2Cr2O7 can also be employed. Other redox titrations are
iodimetry, iodometry, etc.

(i) Iodimetry: This titration involves free iodine. Such direct estimation of iodine is called iodimetry.
This involves the titration of iodine solution with known sodium thiosulphate solution, whose
normality is N. Let the volume of sodium thiosulphate used be V litre.

I2 + 2Na2S2O3 2NaI + Na2S4O6


(n=2) (n=1)

Equivalents of I2 = Equivalents of Na2S2O3 used = N × V

N× V
Moles of I2 =
2
 N× V 
Mass of free I2 in the solution =   254  g
 2 
(ii) Iodometry: This is an indirect method of estimation of iodine. An oxidising agent is made to
react with excess of solid KI. The oxidising agent oxidises I– to I2. This liberated iodine is then
made to react with Na2S2O3 solution of normality N. Let the volume of thiosulphate solution
required be V litre.

2 Na 2S2 O3
Oxidising Agent (A) + KI  I2   2NaI + Na2S4O6 + reduced form of oxidizing
agent

Amity Institute for Competitive Examinations : Phones: 24336143, 24336144, 25573111/12/13/14,95120-2431839/42 [ 44 ]


Redox Reactions AICE (IIT-JEE)

Equivalents of ‘A’ = Equivalents of I2 = Equivalents of Na2S2O3 used = N× V


Equivalents of I2 liberated from KI = N × V
Equivalents of ‘A’ = N × V
Let the n-factor of ‘A’ in its reaction with KI be x, then

N× V
Mass of ‘A’ consumed =  M A (where MA is the molar mass of A)
x

Back Titrations

Let us assume that we have an impure solid substance ‘C’, weighing ‘w’ g and we are required to
calculate the percentage purity of ‘C’ in the sample. We are also provided with two solutions ‘A’ and
‘B’, where the concentration of ‘B’ is known (N1) and that of ‘A’ is unknown. For the back titration
to work, following conditions are to be satisfied:

(i) Compounds ‘A’, ‘B’ and ‘C’ should be such that ‘A’ and ‘B’ react with each other.

(ii) ‘A’ and pure ‘C’ also react with each other but the impurity present in ‘C’ does not react
with ‘A’.

(iii) Also the product of ‘A’ and ‘C’ should not react with ‘B’.

Now we take out certain volume of ‘A’ in a flask (the equivalents of ‘A’ taken should be equivalents
of pure ‘C’ in the sample) and perform a simple titration using ‘B’. Let us assume that the volume of
‘B’ used be V1 litre.

Equivalents of ‘B’ reacted with ‘A’ = N1V1

 Equivalents of ‘A’ initially = N1V1

In another flask, we again take same volume of ‘A’ but now ‘C’ is added to this flask. Pure part of
‘C’ reacts with ‘A’ and excess of ‘A’ is back titrated with ‘B’. Let the volume of ‘B’ consumed is V2
litre.

Equivalents of ‘B’ reacted with excess of ‘A’ = N1V2

 Equivalents of ‘A’ in excess = N1V2

Equivalents of ‘A’ reacted with pure ‘C’ = (N1V1 – N1V2)

Equivalents of pure ‘C’ = (N1V1– N1V2)

(N1V1  N1V2 )
Let the n-factor of ‘C’ in its reaction with ‘A’ be x, then the moles of pure ‘C’ =
x
(N1V1  N1V2 )
 Mass of pure ‘C’ = × molar mass of ‘C’
x
(N1V1  N1V2 ) Molar mass of 'C'
 Percentage purity of ‘C’ =  × 100
x w

Amity Institute for Competitive Examinations : Phones: 24336143, 24336144, 25573111/12/13/14,95120-2431839/42 [ 45 ]


Redox Reactions AICE (IIT-JEE)

Double Titrations
The purpose of double titration is determine the percentage composition of an alkali mixture or an
acid mixture. In the present case, we will find the percentage composition of an alkali mixture. Let us
consider a solid mixture of NaOH, Na2CO3 and some inert impurities, weighing ‘w’ g. We are
required to find the % composition of this alkali mixture. We are also given an acid reagent (HCl) of
known concentration M1 that can react with the alkali sample.

We first dissolve this mixture in water to make an alkaline solution and then we add two indicators,
(Indicators are substances that indicate colour change of solution when a reaction gets completed),
namely phenolphthalein and methyl orange to the solution. Now, we titrate this alkaline solution with
standard HCl.

NaOH is a strong base while Na2CO3 is a weak base. So it is obvious that NaOH reacts first with
HCl completely and Na2CO3 reacts only after complete NaOH is neutralized.

NaOH + HCl NaCl + H2O ...(i)

Once NaOH has reacted completely, then Na2CO3 starts reacting with HCl in two steps, shown as

Na2CO3 + HCl NaHCO3 + NaCl ...(ii)

NaHCO3 + HCl NaCl + CO2 + H2O ...(iii)

It is clear that when we add HCl to the alkaline solution, alkali is neutralized and the pH of the
solution decreases. Initially the pH decrease would be rapid as strong base (NaOH) is neutralized
completely. When Na2CO3 is converted to NaHCO3 completely, the solution is still weakly basic due
to the presence of NaHCO3 (which is weaker as compared to Na2CO3). At this point, phenolphthalein
changes colour since it requires this weakly basic solution to show its colour change. When HCl is
further added, the pH again decreases and when all the NaHCO3 reacts to form NaCl, CO2 and
H2O the solution becomes weakly acidic due to the presence of the weak acid (H2CO3). At this
point, methyl orange changes colour as it requires this weakly acidic solution to show its colour
change.

Thus is general, phenolophthalein shows colour change when the solution contains weakly
basic NaHCO3 along with other neutral substances while methyl orange shows colour change
when solution contains weakly acidic H2CO3 along with other neutral substances.

Illustration 1: 0.56 g of lime stone was treated with oxalic acid to give CaC2O4. The precipitate decolorized
45 ml of 0.2 N KMnO4 in acid medium. Calculate percentage of CaO in lime stone.

Lime stone oxalic acid KMnO


Solution : CaCO3     CaC2O4  4  decolorizes

 Redox changes :

For CaC2O4 C2 3  2C+4 + 2e

For KMnO4 5e + Mn+7  Mn+2


 Meq. of CaCO3 = Meq. of CaC2O4 = Meq. of KMnO4
 Meq. of CaCO3 = Meq. of CaO (since CaO is present in CaCO3)

Amity Institute for Competitive Examinations : Phones: 24336143, 24336144, 25573111/12/13/14,95120-2431839/42 [ 46 ]


Redox Reactions AICE (IIT-JEE)

 Meq. of CaO = Meq. of KMnO4

w
× 1000 = 45 × 0.2
56 / 2
wt. of CaO = 0.252 g

0.252
 percentage of CaO in lime stone = × 100 = 45%
0.56

Illustration 2: 1 g sample of AgNO3 is dissolved in 50 ml of water. It is titrated with 50 ml of KI solution.


The AgI precipitated is filtered off. Excess of KI in filtrate is titrated with M/10 KlO3 in
presence of 6M HCl till all I– converted into ICl. It requires 50 ml of M/10 KlO3 solution. 20
ml of the same stock solution of KI requires 30 ml of M/10 KIO3 under similar conditions.
Calculate % of AgNO3 in sample. The reaction is :
KIO3 + 2KI + 6HCl  3KCl + 3H2O

Solution : AgNO3 + KI  AgI + KNO3


1. Ag present in AgNO3 is removed as AgI by adding 50 ml KI of which 20 ml requires 30
ml of M/10 KIO3.
2. The solution contains KI unused. The unused KI is converted into ICl by KIO3.
 Meq. of KI in 20 ml = Meq. of KIO3 4e + I+5  I+1
1
= 30 × ×4 I–  I+1 + 2e
10
 Meq. of KI in 50 ml added to AgNO3  Eq. wt. of KI = M
2
30  4  50
= = 30
10  20

1
Now Meq. of KI left unused by AgNO3 = 50 × × 4 = 20
10
 Meq. of KI used for AgNO3 = 30 – 20  Mole ratio of AgNO3 and KI
 Meq. of AgNO3 = 10 reaction is 1 : 1 and thus if Eq.

w
 × 1000 = 10 wt. of KI is M/2, then Eq. wt.
170 / 2
w = 0.85 g of AgNO3 = M/2
0.85 100
 percentage of purity of AgNO3 in sample = = 85%.
1
Illustration 3: A 5.0 cm3 solution of H2O2 liberates 0.508 g of iodine from an acidified KI solution. Calculate
the strength of H2O2 solution in terms of volume strength at STP.
Solution : Meq. of H2O2 = Meq. of I2
(w/17) × 1000 = [0.508/(254/2)] × 1000  w = 0.068 g
H2O2  H2O + (1/2)O2
 34 g H2O2 gives 11.2 litre O2, at STP
 0.068 g gives (11.2 × 0.068)/34 = 22.4 ml O2
 Volume strength of H2O2 = 22.4/5 = 4.48%.

Amity Institute for Competitive Examinations : Phones: 24336143, 24336144, 25573111/12/13/14,95120-2431839/42 [ 47 ]


Redox Reactions AICE (IIT-JEE)

Illustration 4: A mixture of KMnO4 and K2Cr2O7 weighing 0.24 g on being treated with KI in acid solution
liberates just sufficient l2 to react with 60 ml of 0.1 N Na2S2O3. Find out percentage of Cr
and Mn in mixture.

Solution : The reactions are :


5e + Mn+7  Mn+2 and 2I–  I2 + 2e
6e + Cr2+6  2Cr+3
Let wt. of KMnO4 and K2Cr2O7 be a and b g respectively.
 Meq. of KMnO4 + Meq. of K2Cr2O7 = Meq. of I2 = Meq. of hypo = 60 × 0.1

a b
 × 1000 + × 1000 = 6 .....(1)
158 / 5 294 / 6
Also given a + b = 0.24 .....(2)
 a = 0.098 g i.e. wt. of KMnO4
b = 0.142 g i.e. wt. of K2Cr2O7

55 0.098
Also wt. of Mn in 0.098 g KMnO4 = = 0.034 g
158

52  2  0.142
wt. of Cr in 0.142 g K2Cr2O7 = = 0.050 g
294

0.034  100
 percentage of Mn in sample = = 14.17%
0.24

0.050 100
 percentage of Cr in sample = = 20.92%.
0.24
Illustration 5: 1 g of H2O2 solution containing X% H2O2 by weight requires X ml of KMnO4 for complete
oxidation in acid medium. Calculate normality of KMnO4 solution.

Solution : Redox changes are :


Mn+7 + 5e  Mn2+
O2–1  O20 + 2e

34
 Eq wt. of H2O2 =
2
Now Meq. of KMnO4 = Meq. of H2O2
X
N.(X) = × 1000
100  34/2
 N = 0.588.
Illustration 6: Find out the percentage of oxalate ion in a given sample of oxalate salt of which 0.3 g
dissolved in 100 ml of water required 90 ml of N/20 KMnO4 for complete oxidation.

Solution : Redox changes are :


5e + Mn+7  Mn+2
C2+3  2C+4 + 2e
 Meq. of oxalate ion = Meq. of KMnO4

Amity Institute for Competitive Examinations : Phones: 24336143, 24336144, 25573111/12/13/14,95120-2431839/42 [ 48 ]


Redox Reactions AICE (IIT-JEE)

w 1 Io n ic w t.
× 1000 = 90 × EC 2 
E 20 2O 4
2
w
88 × 1000 = 9
2 2

 w C O–2 = 0.198 g
2 4

 0.3 g C2O4–2 sample has oxalate ion = 0.198 g


0.198  100
 percentage of C2O4–2 in sample = = 66%.
0. 3
Illustration 7: A 1.1 g sample of copper ore is dissolved and the Cu+2 ion is treated with excess KI. The
liberated I2 requires 12.12 ml of 0.1 M Na2S2O3 solution for titration. What is % of copper
by mass in the ore?

Solution : Cu+2 + e– Cu+


2I–  I2 + 2e–
2S2O32– S4O6– + 2e–
Cu +2
Meq of Cu+2 = Meq. of liberated I2 =Meq of Na2S2O3 ×1000 = 1.212
63.6
WCu+2  0.077g

0.077
%Cu =  100 = 7%
1.10

SAMPLE PROBLEMS 2.2 (MCQ)


Problem 1: When 8.3 g copper sulphate reacts with excess of potassium iodide then the amount of
iodine liberated is:
(a) 42.3 g (b) 24.3 g
(c) 4.23 g (d) 2.43 g

Solution: (c) Copper sulphate and potassium iodide react as follows:

2CuSO4  5H2 O  KI 
 2Cu 2 I 2  2K 2SO4  I2  10H2 O
Mass of copper sulphate used = 2(63 +32 + 64 + 5 × 18) = 498g
Mass of I2 liberated = 2 × 127 = 254 g
 496 g of CuSO4 liberate I2 = 254

254
8.3 g of CuSO4 liberate I 2   8.3  4.23g
498

Problem 2: MnO 24 (1 mol) in neutral aqueous medium is disproportionate to:


(a) 2/3 mol of MnO4 and 1/3 mol MnO2
(b) 1/3 mol of MnO4 and 2/3 mol MnO4
(c) 1/3 mol of Mn2O7 and 1/3 mol MnO2
(d) 2./3 mol Mn2O7 and 1/3 mol MnO2

Amity Institute for Competitive Examinations : Phones: 24336143, 24336144, 25573111/12/13/14,95120-2431839/42 [ 49 ]


Redox Reactions AICE (IIT-JEE)

Solution: (a) The reaction of MnO 24 in aqueous medium takes place as below:

3MnO24  2H 2O 
 MnO2  2MnO4  4OH

Problem 3: Number of moles of K2Cr2O7 reduced by 1 mole of Sn2+ is


(a) 1/6 (b) 1/3
(c) 2/3 (d) 1

Solution: (b) Moles of K2Cr2O7 × n-factor = moles of Sn2+ × n-factor.


1 2 1
Moles of K 2Cr2O7  
6 3
Problem 4: 3.92 g of ferrous ammonium sulphate are dissolved in 100 ml water 20 ml of this solution
requires 16 ml of potassium permanganate during titration for complete oxidation. The weight
of KMnO4 present in one litre of the solution is
(a) 34.75 g (b) 12.38 g
(c) 1.238 g (d) 3.95 g

Solution: (d) Wt. of FeSO4.(NH4)2SO4.6H2O in 20 ml solution = 0.784 g


Molar mass of FeSO4.(NH4)2SO4.6H2O = 392
Molar mass of KMnO4 = 158
Equivalent of FeSO4.(NH4)2SO4.6H2O = Equivalent of KMnO4

Wt. FeSO 4 .(NH 4 ) 2SO 4 .6H 2O Weight of KMnO4



Equivalent weight Equivalent weight of KMnO 4

0.784 Wt. of KMnO 4


 5
392 158

Weight of KMnO4 in 16 ml solution = 0.0632 g


Weight of KMnO4 in 1 lit of solution = 3.95 g

Problem 5: MnO4 ions are reduced in acidic condition of Mn2+ ions whereas they are reduced in neutral
condition to MnO2. The oxidation of 25 ml of solution X containing Fe2+ ions required in
acidic condition 20 ml of a solution Y containing MnO4 ions. What volume of solution Y
would be required to oxidise 25 ml of solution X containing Fe2+ ions in neutral condition?
(a) 11.4 ml (b) 12.0 ml
(c) 33.3 ml (d) 35.0 ml

Solution: (c) In acidic medium n-factor of MnO4 = 5


In neutral medium n-factor of MnO4 = 3
In acidic medium
meq. of MnO4 = meq . of Fe2+
M1 × V1 × n-factor = M2 × V2 × n-factor
M1 × 20 × 5 = M2 × 25 × 1

Amity Institute for Competitive Examinations : Phones: 24336143, 24336144, 25573111/12/13/14,95120-2431839/42 [ 50 ]


Redox Reactions AICE (IIT-JEE)

M2 = 4 × M1
In neutral medium

meq. of MnO4 = meq . of Fe2+

M1 × V1 × n-factor = M2 × V2 × n-factor
M1 × V1 × 3 = 4 × M1 × 25 × 1
100
V1   33.3 ml
3
Problem 6: Volume V1 mL of 0.1 M K2Cr2O7 is needed for complete oxidation of 0.678 g N2H4 in acidic
medium. The volume of 0.3 M KMnO4 needed for same oxidation in acidic medium will be:
2 5
(a) V1 (b) V1
5 2
(c) 113 V1 (d) cannot be determined

Solution: (a) Equivalent of K2Cr2O7 = equivalent of N2H4


also equivalent of KMnO4 = equivalent of N2H4
So, equivalent of K2Cr2O7 = equivalent of KMnO4
0.1 × 6 × V1 = 0.3 × 5 × V2
so V2 = 2/5 V1
N
Problem 7: 20 mL of H2O2 after acidification with dilute H2SO4 required 30 mL of KMnO4 for
12
complete oxidation. The strength of H2O2 solution is: [Molar mass of H2O2 = 34]
(a) 2 g/L (b) 4 g/L
(c) 8 g/L (d) 6 g/L

Solution: (a) meq. of KMnO4 = meq. of H2O2


1 30 1
30   20  N  N   N
12 12  20 8
1
Strength in gm/litre = N × equivalent mass =  17  2.12 g / L
8
Problem 8: A mixture of 0.02 mole of KBrO3 and 0.01 mole of KBr was treated with excess of KI and
acidified. The volume of 0.1 M Na2S2O3 solution required to consume the liberated iodine
will be:
(a) 1000 mL (b) 1200 mL
(c) 1500 mL (d) 800 mL

Solution: (b) BrO3  6I  


 3I 2  Br 

moles of I2 = 3 × moles of KBrO3


 moles of I2 = 0.03 × 3 = 0.06
0.06 × 2 = 0.1 ×V
V = 1.2 L = 1200 mL

Amity Institute for Competitive Examinations : Phones: 24336143, 24336144, 25573111/12/13/14,95120-2431839/42 [ 51 ]


Redox Reactions AICE (IIT-JEE)

Problem 9: In the reaction CrO5  H 2SO4 


 Cr2 (SO 4 )3  H 2O  O 2 , one mole of CrO5 will liberate
how many moles of O2:
(a) 5/2 (b) 5/4
(c) 9/2 (d) 7/4

Solution: (d) 4CrO5  6H 2SO4 


 2Cr2 (SO4 )3  6H 2O  7O2

1 mole CrO5 produces 7/4 moles of O2



Amity Institute for Competitive Examinations : Phones: 24336143, 24336144, 25573111/12/13/14,95120-2431839/42 [ 52 ]


Redox Reactions AICE (IIT-JEE)

CHAPTER ASSIGNMENT
MULTIPLE CHOICE QUESTIONS (Single Correct)
1. The equivalent weight of FeSO 4 when it is oxidised by acidified KMnO 4 will be equal to
(M = molecular weight)
M FeSO 4 M FeSO 4
(a) M of FeSO4 (b) (c) 2 M FeSO4 (d)
2 4
2. The equivalent weight of K2Cr2O7 when it is converted to Cr3+ will be equal to
M K 2Cr2O 7 M K 2 Cr2 O7 M K 2 Cr2 O7
(a) M K 2 Cr2O 7 (b) (c) (d)
3 4 6
3. The amount of nitric acid required to oxidise 127 gms of I2 to I2O5 will be _____. Assume that during
the reaction HNO3 gets converted to NO2.
(a) 12.7 (b) 3.15 (c) 315 (d) 31.5

4. 10 ml of oxalic acid was completely oxidised by 20 ml of 0.02 M KMnO4. The normality of oxalic
acid solution is:
(a) 0.05 N (b) 0.1 N (c) 0.2 N (d) 0.025 N

5. 0.2 g of a sample of H2O2 required 10 ml of 1N KMnO4 in a titration in the presence of H2SO4.


Purity of H2O2 is
(a) 25% (b) 65% (c) 85% (d) None of these

6. The number of moles of KMnO4 that will be needed to react completely with one mole of ferrous
oxalate in acidic solution is
(a) 2/5 (b) 3/5 (c) 4/5 (d) 1

7. Select the compound in which chlorine is assigned the oxidation number +5


(a) HClO4 (b) HClO2 (c) HClO3 (d) HCl

8. White P reacts with caustic soda. The products are PH 3 and NaH2PO2. This reaction is an
example of
(a) oxidation (b) reduction (c) oxidation and reduction (d) neutralization

9. One mole of N2H4 loses ten moles of electrons to form a new compound Y. Assuming that all the
nitrogen appears in the new compound, what is the oxidation state of nitrogen in Y ? (There is no
change in the oxidation state of hydrogen)
(a) –1 (b) –3 (c) +3 (d) +5

10. The oxidation number of iron in potassium ferricyanide is


(a) +1 (b) +2 (c) +3 (d) +4

11. In the following equation value of X is


ClO3– + 6H+ + X  Cl– + 3H2O
(a) 4e– (b) 5e– (c) 6e– (d) 7e–

Amity Institute for Competitive Examinations : Phones: 24336143, 24336144, 25573111/12/13/14,95120-2431839/42 [ 53 ]


Redox Reactions AICE (IIT-JEE)

12. The brown ring complex compound is formulated as [Fe(H2O)5(NO)+]SO4. The oxidation state of
iron is
(a) 1 (b) 2 (c) 3 (d) 0

13. If three electrons are lost by a metal iron M3+, its final oxidation number would be
(a) 0 (b) +2 (c) +5 (d) +6

14. In the following reaction


3Br2 + 6CO32– + 3H2O  5Br– + 6HCO3– + BrO3–
(a) bromine is both reduced and oxidised (b) bromine is neither reduced nor oxidised
(c) bromine is oxidised and carbonate is reduced (d) bromine is reduced and water is oxidised

15. In the following reaction the value of ‘x’ is


H2O + SO32–  SO42– + 2H+ + x
(a) 4e– (b) 3e– (c) 2e– (d) 1e–

INTEGER TYPE QUESTIONS

16. The value of n in the reaction: Cr2O72– + ne + 14H+  2Cr3+ + 7H2O

17. If four mole of Br2 undergo a loss and gain of six mole electrons to form two new oxidation state of
Br. How much Br2 mole acts are reductant?

18. Total number of electrons involved per molecule oxidation of FeC2O4 to Fe3+ and CO2.

19. In the reaction P4 + NaOH  PH3 + NaH2PO2, mole ratio of NaH2PO2 and PH3 is

20. The ratio of oxygen atom having –2 and –1 oxidation numbers in S2O82– is

21. Five moles of Ferric oxalate are oxidised by how much mole of KMnO4 in acid medium?

22. The difference in oxidation number of two nitrogen atoms in NH4NO3 is

23. The difference in oxidation number of Cl atoms in CaOCl2 is.

24. 4 moles of Cl2 undergoes disproportionation involving six electrons in change. How much Cl2 molecules
are oxidised?

25. The difference in the oxidation number of the two types of sulphur atoms in Na2S4O6 is.

MULTIPLE CHOICE QUESTIONS (More than one Correct)

26. Which of the following are disproportionation redox changes?


(a) (NH4)2Cr2O7 N2 + + Cr2O3 + 4H2O
(b) 5H2O2 + 2ClO2 + 2OH– 2Cl–+ 5O2 + 6H2O
(c) 3ClO–ClO3– + Cl–

Dilution with
(d) 2HCuCl2 
water
Cu + Cu2++ 4Cl–+ 2H+

Amity Institute for Competitive Examinations : Phones: 24336143, 24336144, 25573111/12/13/14,95120-2431839/42 [ 54 ]


Redox Reactions AICE (IIT-JEE)

27. Which one are correct about the reaction? HgS + HCl + HNO3  H2HgCl4 + NO + S + H2O
(a) Hg is reduced (b) Sulphide is oxidised (c) N is reduced (d) HNO3 is oxidant

28. Which of the followings are disproportionation reactions?


(a) 2O3 3O2 (b) 4KClO3 3KClO4 + KCl
(c) 2H2O2 2H2O + O2 (d) 2KO2 + 2H2O 4KOH + 3O2

29. For the reaction, KO2 + H2O + CO2 KHCO3 + O2; the mechanism of reaction suggests that
(a) acid-base reaction (b) disproportionation reaction
(c) hydrolysis (d) redox change

30. Which of the following can be used as oxidant and reductant both?
(a) HNO2 (b) SO2 (c) O2 (d) CO

31. Which molecules represented by the bold atoms show their highest oxidation state?

(a) H2S2O8 (b) P4 O10 (c) F2 O (d) Mn 2 O 7

32. Which molecules represented by the bold atoms show their lowest oxidation state?

(a) F2 O (b) H 2 S (c) P H3 (d) N 2 H 4

33. Which one are not correct about CH2 = CCl2?


(a) Both carbon are in +2 oxidation state
(b) Both carbon are in –2 oxidation state
(c) One carbon has +2 and other has –2 oxidation state
(d) The average oxidation number of carbon is zero

34. Dichromate ion in acidic medium oxidises stannous ion as

xSn 2  yCr2O 27  zH  


 aSn 4  bCr 3  cH 2 O

(a) the value of x : y is 1 : 3 (b) The value of x + y + z is 18


(c) a : b is 3 : 2 (d) the value of z – c is 7

35. When a equimolar mixture of Cu2S and CuS is titrated with Ba(MnO4)2 in acidic medium, the final
product contains Cu2+, SO2 and Mn2+. If the molecular weight of Cu2S, CuS and Ba(MnO4)2 are M1
M2 and M3 respectively then
M1
(a) Eq. wt. of Cu2S
18
M
(b) Eq. wt. of CuS is 3
6
M3
(c) Eq. wt. of Ba(MnO4)2 is
5
(d) Cu2S and CuS both have same equivalents in mixture

Amity Institute for Competitive Examinations : Phones: 24336143, 24336144, 25573111/12/13/14,95120-2431839/42 [ 55 ]


Redox Reactions AICE (IIT-JEE)

MATRIX MATCH TYPE QUESTIONS


36. Match the following:

Column - I Column - II

(A) 4.1g H2SO3 (p) 200 mL of 0.5 N base is used for complete
neutralisation
(B) 4.9g H3PO4 (q) 200 millimoles of oxygen atoms
(C) 4.5 g oxalic acid (H2C2O4) (r) Central atom is in its highest oxidation number
(D) 5.3 Na2CO3 (s) May react with an oxidising agent

37. Column - I Column - II


2
(A) Sn  MnO 4 (acidic) (p) Amount of oxidant available decides the
3.5 mole 1.2 mole

number of electrons transfer



2 C 2 O 4  MnO (acidic)
(B) H8.4 4 (q) Amount of reductant available decides the
mole 3.6 mole

number of electrons transfer


2
(C) S O  3.6Imole
2 3
7.2 mole
2 (r) Number of electrons involved per mole of
oxidant > Number of electrons involved per
mole of reductant
2 2
Fe  Cr2 O (acidic)
(D) 9.2 7 (s) Number of electrons involved per mole of
mole 1.6 mole

oxidant < Number of electrons involved per


mole of reductant.

COMPREHENSION TYPE QUESTIONS


Comprehension - I

In the chemical change: aN2H4 + bBrO3–  aN2 + bBr– + 6H2O, answer the following questions:

38. The element oxidised and reduced in the reaction are respectively:
(a) N2H4, BrO3– (b) N, Br (c) H, Br (d) BrO3–, N2H4

39. The equivalent weight of N2H4 in the above reaction is


(a) 8 (b) 10.6 (c) 16 (d) 6.4

40. The equivalent weight of KBrO3 in the above reaction is


(a) 167 (b) 27.83 (c) 55.67 (d) 83.5

Comprehension - II

Molecular weight / Atomic weight


Equivalent weight =
n-factor
In general n-factor of acid/base is number of moves H+ / OH– furnished per mol of acid/base. In redox
reaction n-factor is number of moves of electron lost/gained per mole of reactant.

Amity Institute for Competitive Examinations : Phones: 24336143, 24336144, 25573111/12/13/14,95120-2431839/42 [ 56 ]


Redox Reactions AICE (IIT-JEE)

41. For the reaction:

H 3PO 2  NaOH 
 NaH 2 PO2  H2 O
What is the equivalent weight of H3PO2? (Mol. wt. is M)
M M 2M
(a) M (b) (c) (d)
2 3 3

42. For the reaction Fe0.95O 


 Fe2 O3 .
What is the equivalent weight of Fe0.95O? (Mol. wt. of Fe0.95O is M)
M M M
(a) (b) (c) (d) None of these
0.85 0.95 0.8075

43. In the reaction xVO  yFe2O3 


 FeO  V2O5 .
What is the value of x and y respectively?
(a) 1.1 (b) 2, 3 (c) 3, 2 (d) None of these



Amity Institute for Competitive Examinations : Phones: 24336143, 24336144, 25573111/12/13/14,95120-2431839/42 [ 57 ]


Redox Reactions AICE (IIT-JEE)

PREVIOUS YEAR QUESTIONS


Paragraph for Questions Nos. 1 to 3

Redox reactions play a vital role is chemistry and biology. The values of standard redox potential (E°) of two
half-cell reactions decide which way the reaction is expected to proceed. A simple example is a Daniel cell
in which zinc goes into solution and copper gets deposited. Given below are a set of half-cell reactions (acidic
medium) along with their E° (V with respect to normal hydrogen electrode) values.

I2 + 2e–  2I E° = 0.54
– –
Cl2 + 2e  2Cl E° = 1.36
3+ – 2+
Mn + e  Mn E° = 1.50
Fe3+ + e–  Fe2+ E° = 0.77
+ –
O2 + 4H + 4e  2H2O E° = 1.23

1. Among the following, identify the correct statement [IIT 2007]


2+
(a) Chloride ion is oxidised by O2 (b) Fe is oxidised by iodine
(c) Iodide ion is oxidised by chlorine (d) Mn2+ is oxidised by chlorine

2. While Fe3+ is stable, Mn3+ is not stable in acid solution because [IIT 2007]
2+ 3+
(a) O2 oxidises Mn to Mn (b) O2 oxidises both Mn to Mn & Fe2+ to Fe3+
2+ 3+

(c) Fe3+ oxidises H2O to O2 (d) Mn3+ oxidises H2O to O2

3. Sodium fusion extract, obtained from aniline, on treatment with iron (II) sulphate and H2SO4 in
presence of air gives a Prussian blue precipitate. The blue colour is due to the
formation of [IIT 2007]
(a) Fe4 [Fe(CN)6]3 (b) Fe3 [Fe(CN)6] 2 (c) Fe4 [Fe(CN)6] 2 (d) Fe3 [Fe(CN)6] 3

4. The spin only magnetic moment value (in Bohr magneton units) of Cr(CO)6 is [IIT 2009]
(a) 0 (b) 2.84 (c) 4.90 (d) 5.92

5. For the reduction of NO3 ion in an aqueous solution, E0 is + 0.96V. Values of E0 for some metal ions
are given below
V2+ (aq) + 2e– V E0 = –1.19 V
Fe3+ (aq) + 3e– Fe E0 = –0.04 V
Au3+ (aq) + 3e– Au E0 = +1.40 V
Hg2+ (aq) + 2e– Ag E0 = +0.86 V

The pair(s) of metals that is (are) oxidized by NO3 in aqueous solution is (are) [IIT 2009]

(a) V and Hg (b) Hg and Fe (c) Fe and Au (d) Fe and V

6. The difference in the oxidation numbers of the two types of sulphur atoms in Na2S4O6 is [IIT 2011]

7. Which ordering of compounds is according to the decreasing order of the oxidation state of nitrogen?
(a) HNO3, NO, NH4Cl, N2 (b) HNO3, NO, N2, NH4Cl
(c) HNO3, NH4Cl, NO, N2 (d) NO, HNO3, NH4Cl, N2 [IIT 2012]

Amity Institute for Competitive Examinations : Phones: 24336143, 24336144, 25573111/12/13/14,95120-2431839/42 [ 58 ]


Redox Reactions AICE (IIT-JEE)

8. For the reaction: I   ClO3  H 2SO4 


 Cl  HSO 4  I 2
The correct statement(s) in the balanced equation is/are [JEE-Advance 2014]

(a) Stoichiometric co-efficient of HSO 4 is 6 (b) Iodide is oxidized

(c) Sulphur is reduced (d) H2O is one of the products

9. In an alkaline condition KMnO4 reacts as follows: 2KMnO4 + 2KOH  2K2MnO4 + H2O + O.


Its equivalent weight is [DCE-2002]
(a) 31.6 (b) 52.7 (c) 49 (d) 158

10. The oxidation number of S in Na2S4O6 is [DCE-1999]


(a) –2.5 (b) 2.5 (c) –10 (d) +10

11. For the reaction: Na2 CO3 + 2HCl  NaCl + H2O + CO2
Equivalent weight of Na2CO3 is [DCE-2006]
(a) M/2 (b) M (c) 2M (d) M/4

12. Oxidation state of sulphur in Na2S2O3 and Na2S4O6 [DCE-2006]


(a) 4 and 6 (b) 3 and 5 (c) 2 and 2.5 (d) 6 and 6

13. When KMnO4 acts as an oxidising agent and ultimately forms [MnO4]–1, MnO2, Mn2O3, Mn+2
then the number of electrons transferred in each case respectively is [AIEEE-2002]
(a) 4, 3, 1, 5 (b) 1, 5, 3, 7 (c) 1, 3, 4, 5 (d) 3, 5, 7, 1

14. In context with the transition elements, which of the following statements is
incorrect? [AIEEE-2009]
(a) In the highest oxidation states, the transition metal show basic character and form cationic
complexes
(b) In the highest oxidation states of the first five transition elements (Sc to Mn), all the 4s and 3d
electrons are used for bonding.
(c) Once the d5 configuration is exceeded, the tendency to involve all the 3d electrons in bonding
decreases
(d) In addition to the normal oxidation states, the zero oxidation state is also shown by these elements
in complexes.

 2  z
15.  xMn 2  2 yCO 2  H 2 O
Consider the following reaction: xMnO4  yC2 O 4  zH 
2

The values of x, y and z in the reaction are, respectively: [JEE-Mains 2013]


(a) 2, 5 and 16 (b) 5, 2 and 8 (c) 5, 2 and 16 (d) 2, 5 and 8

16. In which of the following reactions H2O2 acts as a reducing agent? [JEE-Mains 2014]
+ – – +
(1) H2O2 + 2H + 2e  2H2O (2) H2O2 – 2e  O2 + 2H
(3) H2O2 + 2e–  2OH– (4) H2O2 + 2OH– – 2e–  O2 + 2H2O
(a) (1), (3) (b) (2), (4) (c) (1), (2) (d) (3), (4)

Amity Institute for Competitive Examinations : Phones: 24336143, 24336144, 25573111/12/13/14,95120-2431839/42 [ 59 ]


Redox Reactions AICE (IIT-JEE)

17. Hydrogen peroxide oxidises [Fe(CN)6]4– to [Fe(CN)6]3– in acidic medium but reduces [Fe(CN6]3– to
[Fe(CN)6]4– in alkaline medium. The other products formed are, respectively: [JEE Mains 2018]
(a) H2O and (H2O + OH–) (b) (H2O + O2) and H2O

(c) (H2O + O2) and (H2O + OH ) (d) H2O and (H2O + O2)

18. The oxidation states of Cr in [Cr(H 2O) 6]Cl3 , [Cr(C 6 H 6 )2 ], and K 2 [Cr(CN) 2 (O)2 (O 2 )(NH 3 )]
respectively are [JEE Mains 2018]
(a) +3, 0 and +4 (b) +3, +4 and +6 (c) +3, +2 and +4 (d) +3, 0 and +6

19. In the reaction of oxalate with permanganate in acidic medium, the number of electrons involved in
producing one molecule of CO2 is: [JEE Mains 2019]

(a) 10 (b) 2 (c) 1 (d) 5

20. The redox reaction among the following is: [JEE Mains 2020]
(a) Combination of dinitrogen with dixoygen at 200 K
(b) Formation of ozone from atmospheric oxygen in the presence of sunlight
(c) Reaction of H2SO4 with NaOH
(d) Reaction of [Co(H2O)6]Cl3 with AgNO3
21. The compound that cannot act both as oxidising and reducing agent is : [JEE Mains 2020]
(a) H 2 O 2 (b) H2 SO 3 (c) HNO 2 (d) H 3 PO 4

22. Oxidation number of potassium in K2O, K2O2 and KO2, respectively is: [JEE Mains 2020]
1
(a) +1, +4 and +2 (b) +1, +2 and +4 (c) +1, +1 and +1 (d) +2 , +1 and 
2
23. The species given below that does NOT show disproportionation reaction is: [JEE Mains 2021]

(a) BrO4 (b) BrO (c) BrO2 (d) BrO3

24. Identify the process in which change in the oxidation state is five: [JEE Mains 2021]

(a) (b)

(c) (d)

25. In which one of the following sets all species show disproportionation reaction ?
(a) (b)

(c) (d)
26. The denticity of an organic ligand, biuret is :
(a) 2 (b) 4 (c) 3 (d) 6



Amity Institute for Competitive Examinations : Phones: 24336143, 24336144, 25573111/12/13/14,95120-2431839/42 [ 60 ]


Redox Reactions AICE (IIT-JEE)

CHAPTER TEST
SECTION - I: MULTIPLE CHOICE QUESTIONS (Single Correct)
1. Volume V1 mL of 0.1 M K2Cr 2O7 is needed for complete oxidation of 0.678 g N2H4 in acidic
medium. The volume of 0.3 M KMnO4 needed for same oxidation in acidic medium will be:
2 5
(a) V1 (b) V1 (c) 113 V1 (d) cannot be determined
5 2
2. In which of the following reactions is there a change in the oxidation number of nitrogen atom:
(a) 2NO2 
 N 2O 4  NH 4  OH 
(b) NH 3  H 2 O 
(c) N 2O5  H 2O 
 2HNO3 (d) None of these

3. In which of the following reactions, hydrogen is acting as an oxidising agent?


(a) With iodine to give hydrogen iodide (b) With lithium to give lithium hydride
(c) With nitrogen to give ammonia (d) With sulphur to give hydrogen sulphide

4. In which of the following pairs, there is greatest difference in the oxidation number of the under lined
elements?
(a) NO2 and N2O4 (b) P2O5 and P4O10 (c) N2O and NO (d) SO2 and SO3

5. Nitric oxide acts as a reducing agent in the reaction


(a) 4NH3  5O 2 
 4NO  6H 2O  2NO3  6I   8H 
(b) 2NO  3I2  4H 2O 
(c) 2NO  H 2SO3 
 N 2O  H 2SO 4 (d) 2NO  H 2S 
 N 2 O  S  H 2O

SECTION - II: MULTIPLE CHOICE QUESTIONS (More than one correct)


6. 25 mL of 0.5 M H2O2 solution is added to 50 mL of 0.2 M KMnO4 in acidic solution. Which of the
following statements is false?
(a) 0.010 mole of oxygen gas is liberated (b) 0.005 mole of KMnO4 is left
(c) 0.030 g of oxygen gas is evolved (d) 0.0025 mole H2O2 does not react with KMnO4

7. Find the normality and the volume strength of the solution, which made by mixing of 5.6V and 11.2V
H2O2 solutions. Volume of both is 1 litre each:
(a) 8.4 V (b) 1.5 N (c) 4.8 V (d) 1 N

8. Which of the following reactions do not involve oxidation or reduction?


(a) 2Rb  2H 2O 
 2RbOH  H 2 (b) 2CuI 2 
 2CuI  I 2
(c) NH 4Cl  NaOH 
 NaCl  NH3  H2O (d) 4KCN  Fe(CN) 2 
 K 4 [Fe(CN)6 ]

9. Which of the following can act both as an oxidisnig as well as reducing agent?
(a) HNO2 (b) H2O2 (c) H2S (d) SO2

10. In the following reaction: Cr(OH)3  OH   IO3 


 CrO42  H 2O  I 
(a) IO3 is oxidising agent (b) Cr(OH)3 is oxidised

(c) 6e are being taken per iodine atom (d) None of these

Amity Institute for Competitive Examinations : Phones: 24336143, 24336144, 25573111/12/13/14,95120-2431839/42 [ 61 ]


Redox Reactions AICE (IIT-JEE)

SECTION - III: INTEGER TYPE QUESTIONS

11. When SO2 is passed through an acidified solution of potassium dichromate, the oxidation state of S in
the product will be

12. In the reaction xHI  yHNO3 


 NO  I 2  H 2 O the ratio of x and y is

13. K2Cr2O7 oxidises oxalic acid in acidic medium. The number of CO2 molecules produced as per the
balanced equation is

14. The oxidation number of phosphorous in Ba(H2PO2)2 is

15. The oxidation state of chromium in [Cr(PPh3)3(CO)3] is

16. Number of moles of Sn2+ oxidised by one mole of K2Cr2O7 in acidic medium is

17. The volume strength of 1.43N H2O2 solution is

18. In the ionic equation 2KBrO3  12H   10e  Br2  6H 2O  2K  . The equivalent weight of
M
KBrO3 will be (M = molecular weight of KBrO3), The value of x is
x

SECTION - IV: MATRIX MATCH


19. Match the following:

Column - I Column - II (Equivalent weight of reactant)


 NO3
(A) NH 3  (p) M

(B) Fe2S3 
 2FeSO4  SO 2 (q) M/5

(C) KMnO4 
 MnSO4 (r) M/8

(D) CuS 
 CuSO4 (s) M/20
(a) (A) – (r); (B) – (s); (C) – (q); (D) – (r)
(b) (A) – (p); (B) – (q); (C) – (r); (D) – (s)
(c) (A) – (r); (B) – (s); (C) – (q); (D) – (p)
(d) (A) – (q); (B) – (r); (C) – (s); (D) – (p)

SECTION - V: COMPREHENSION TYPE QUESTIONS


Comprehension

2.5 g sample of copper is dissolved in excess H2SO4 to prepare 100 ml of 0.02 M CuSO4 (aq). 10 ml of 0.02
M solution of CuSO4 (aq) is mixed with excess of KI to show the following changes.

CuSO4  2KI 
 K 2SO 4  CuI2

2CuI 2 
 Cu 2 I 2  I2

Amity Institute for Competitive Examinations : Phones: 24336143, 24336144, 25573111/12/13/14,95120-2431839/42 [ 62 ]


Redox Reactions AICE (IIT-JEE)

The liberated iodine is titrated with hypo (Na2S2O3) and requires V mL of 0.1 M hypo solution for its complete
reduction.

20. The volume (V) of hypo required is


(a) 2 ml (b ) 20 ml (c) 1 ml (d) 10 ml

21. Percentage purity of sample is


(a) 10.16 (b) 5.08 (c) 2.54 (d) 1.27

22. The amount of I2 liberated in the reaction of 10 ml of 0.02 M solution with KI (excess) is
(a) 0.051g (b) 0.0254g (c) 0.102g (d) 0.204g


Amity Institute for Competitive Examinations : Phones: 24336143, 24336144, 25573111/12/13/14,95120-2431839/42 [ 63 ]


Redox Reactions AICE (IIT-JEE)

ANSWERS
MULTIPLE CHOICE QUESTIONS (Single Correct )

1. (a) 2. (d) 3. (c) 4. (c) 5. (c)


6. (b) 7. (c) 8. (c) 9. (c) 10. (c)
11. (c) 12. (a) 13. (d) 14. (a) 15. (c)

INTEGER TYPE QUESTIONS

16. (6) 17. (1) 18. (3) 19. (3) 20. (3)
21. (6) 22. (8) 23. (2) 24. (1) 25. (5)

MULTIPLE CHOICE QUESTIONS (More than one Correct )

26. (c,d) 27. (b,c,d) 28. (b,c,d) 29. (a,b,c,d) 30. (a,b,c,d)
31. (a,b,d) 32. (b,c) 33. (a,b) 34. (d) 35. (a, b)

MATRIX MATCH TYPE QUESTIONS


36. (A) – (p), (s); (B) – (q), (r); (C) – (p), (q), (s); (D) – (r)
37. (A) – (p), (r); (B) – (q), (r); (C) – (p), (q), (r); (D) – (q), (r)
COMPREHENSION TYPE QUESTIONS
38. (b) 39. (a) 40. (b) 41. (a) 42. (a)
43. (b)

PREVIOUS YEAR QUESTIONS


1. (c) 2. (d) 3. (a) 4. (a) 5. (a,b,d)
6. (5) 7. (b) 8. (a,d) 9. (d) 10. (b)
11. (a) 12. (c) 13. (c) 14. (a) 15. (a)
16. (b) 17. (d) 18. (d) 19. (c) 20. (a)
21. (d) 22. (c) 23. (a) 24. (b) 25. (c)
26. (a)

CHAPTER TEST
1. (a) 2. (d) 3. (b) 4. (d) 5. (b)
6. (a, c, d) 7. (a, b) 8. (c, d) 9. (a, b, d) 10. (a, b, c)
11. (6) 12. (3) 13. (6) 14. (1) 15. (0)
16. (3) 17. (8) 18. (5) 19. (a) 20. (a)
21. (b) 22. (b)

Amity Institute for Competitive Examinations : Phones: 24336143, 24336144, 25573111/12/13/14,95120-2431839/42 [ 64 ]


Redox Reactions AICE (IIT-JEE)

HINTS AND SOLUTIONS


MULTIPLE CHOICE QUESTIONS (Single Correct)

1. (a) MnO 4  Fe 2 
H
 Mn 2  Fe3
M
E FeSO4 
1
2. (d) Cr2 O7  14H  
2
 2Cr 3  7H 2 O
M
E K 2Cr2O7 
6
3. (c) Equivalent of I2 = Equivalent of HNO3
127  10 x  1

254 63
x = 63 × 5 = 315g
4. (c) Meq. of oxalic acid = Meq. of KMnO4
(NV)Oxalic acid  (NV) KMnO4
N × 10 = 0.02 × 5 × 20
N = 0.2
5. (c) Equivalent of H2O2 = Equivalent of KMnO4
x 1 10

17 1000
0.17
x = 0.17g, % purity =  100  85%
0.2
6. (b) Fe2  CrO24  MnO4   Fe3  CO2  Mn 2
Equivalent of KMnO4 = Equivalent of FeCrO4
5 × moles of KMnO4 = 3 × mole sof FeCrO4
4 3 5
7. (c) H ClO 4 H ClO 2 H ClO 3 HCl 1
P4  NaOH   PH3  NaH 2 PO 2
Reduced
8. (c)
Oxidised
2 x
9. (c) N 2 H 4 
 2 N  10e 
2x – 10 = – 4
2x = +6, x = + 3
3
10. (c) Potasium ferricyanide is K 3 [Fe(CN) 6 ]

11. (c) ClO3  6H   6e 


 Cl  3H 2O
1
12. (a) [Fe(H 2 O)5 (NO)  ]SO 4
13. (d) M3   M 6  3e 
3Br2  6CO32  3H 2 O   5Br   6HCO3  BrO3
Reduced
14. (a)
Oxidised
2
15. (c) H 2 O  SO 32 
SO 4
 2H   2e 

Amity Institute for Competitive Examinations : Phones: 24336143, 24336144, 25573111/12/13/14,95120-2431839/42 [ 65 ]


Redox Reactions AICE (IIT-JEE)

INTEGER TYPE QUESTIONS


2  
16.  2Cr 3  7H 2O
(6) Cr2O  14H  6e 
7

17.  2Br 3  6Br 


(1) 4Br2 
18. (3) Fe2  C2O 24 
 Fe3  2CO2  3e
19. (3) P4  3NaOH  3H2 O 
 PH3  3NaH2 PO2
2 2
O   O
O O
20. (3) 2 S S 2
O - - O
2 O 2 O
21. (6) Equivalent or ferric oxalate = equivalent of KMnO4
6 × moles of Fe2(C2O4)3 = 5 × moles of KMnO4
3 5
22. (8) N H 4 N O3
2 2 1 1
23. (2) Ca(O Cl) Cl

24.  2Cl3  6Cl


(1) 4Cl2 
O O
+5 0 0 +5
Na O S S S S O Na
25. (5)
O O
MULTIPLE CHOICE QUESTIONS (More than one Correct)
26. (c,d)
3ClO  ClO3  Cl 2HCuCl2  Cu  Cu 2  4Cl  2O 
Oxidation Reduction
Reduction Oxidation
27. (b,c,d)
HgS  HCl  HNO3 
 H 2 HgCl4  NO  S  H 2O
Reduction
Oxidation
2KO 2  2H 2O 
 4KOH  3O2
28. (b,c,d) Reduction
Oxidation
29. (a,b,c,d) KO 2  H 2 O 2  CO 2 
 KHCO3  O2
The above reaction is all four type reaction.
30. (a,b,c,d) Species which is in intermediate oxidation state, acts as both oxidising agent as well as
reducing agent.
6 5 7 1
31. (a,b,d) H 2 S 2 O8 P 4 O10 M n 7O7 F2 O
3 2
32. (b,c) F2 O 2
H 2S 2
P H3 N2 H4
2 2
33. (a,b) C H 2  C Cl 2

Amity Institute for Competitive Examinations : Phones: 24336143, 24336144, 25573111/12/13/14,95120-2431839/42 [ 66 ]


Redox Reactions AICE (IIT-JEE)

34. (d) 3Sn 2  14H   Cr2 O72 


 3Sn 3  2Cr 3  7H 2O


35. (a, b) Cu 2  Cu   S2  MnO 4 
H
 Cu 2  SO 2  Mn 3

n-factor of Cu2S = 8
n-factor of CuS = 6
n-factor of Ba(MnO4)2 = 10

MATRIX MATCH TYPE QUESTIONS

36. (A) – (p), (s); (B) – (q), (r); (C) – (p), (q), (s); (D) – (r)
(A) 4.1 g H2SO3 = 0.1 gm equivalent H2SO4
= 150 milli moles of oxygen
= Oxidation state of S is +4
= Reacts with oxidising agent
(B) 4.9 g H3PO4 = 0.15 gm equivalent of H3PO4
= 200 mill moles of oxygen
= Oxidation state of P is + 5
= Do not reacts with oxidising agent
(C) 4.5 g oxalic aicd= 0.1 gm equivalent of oxalic acid
= 200 millimole sof oxygen
= Oxidation state of carbon is + 3
= Reacts with oxidising agent
(D) 5.3 g Na2CO3 = 0.1 gm equivalent of Na2CO3
= 15 millimoles of oxygen
= Oxidation state of C is + 4
= Do not reacts with oxidising agent

37. (A) – (p), (r); (B) – (q), (r); (C) – (p), (q), (r); (D) – (q), (r)
(A) MnO4  Sn 2 
 Mn 2  Sn 4
(B) MnO4  H 2C2 O4 
 Mn 2  CO2
(C) S2O32  I 2 
 S4O62  I 
(D) Cr2 O72  Fe2 
 Cr 3  Fe3

COMPREHENSION TYPE QUESTIONS


38. (b) N2H4 is oxidised and BrO3 is reduced

M
39. (a) E N2H 4 
4
M
40. (b) E KBrO3 
6

Amity Institute for Competitive Examinations : Phones: 24336143, 24336144, 25573111/12/13/14,95120-2431839/42 [ 67 ]


Redox Reactions AICE (IIT-JEE)

41. (a) Basicity of H3PO2 is one

42. (a) Fe0.95O 


 Fe2 O3

Fe2+ = 0.85 is Fe0.95O


M
E Fe0.95O 
0.85
43. (b) 2VO  3Fe2O3 
 6FeO  V2 O5


Amity Institute for Competitive Examinations : Phones: 24336143, 24336144, 25573111/12/13/14,95120-2431839/42 [ 68 ]


Chapter - 3
ATOMIC STRUCTURE
3.1 Introduction
Matter is constituted by very small particles which can not exist in free state in most of the cases
called atoms. These atoms are constituted further by some fundamental particles called electrons;
protons and neutrons. Present chapter deals with the structure of atom in terms of these fundamental
particles and influence of electronic arrangement on the properties of the molecules.

Nature of cathode rays


(i) They travel in straight lines away from the cathode with very high velocities ranging from
107 – 109 cm per second.

(ii) They produce a green glow when strike the wall beyond anode.

(iii) They produce heat energy when they collide with the matter. It shows that cathode rays possess
kinetic energy.

(iv) They are deflected by the electric and magnetic fields. When the rays are passed between two
electrically charged plates, these are deflected towards the positively charged plate.

(v) When a pin wheel is placed in their path, the blades of the wheel are set in motion this indicates
cathode rays consists of material particles which have mass and velocity.

(vi) When they fall on material having high atomic mass, new type of penetrating rays of very small
wavelengths are emitted which are called X-rays.

(vii)They affect the photographic plate.

(viii) They can penetrate through thin foils of solid materials and cause ionisation in gases through
which they pass.

(ix) The nature of the cathode rays is independent of:


(a) The nature of the cathode and.
(b) The gas in the discharge tube.

(x) The negatively charged particles which constitute cathode rays are called electrons.

Rest mass of electron: The rest mass of electron is found to be 9.1096 × 10–31 kg.

Mass of moving electron : The mass of moving electron is given by the relation.

rest mass of electron


Mass of moving electron = 2
v
1   
c
Where v is the velocity of the electron and c is the velocity of light. When v becomes equal to c mass
of moving electron becomes infinity.

Amity Institute for Competitive Examinations : Phones: 24336143, 24336144, 25573111/12/13/14,95120-2431839/42 [ 69 ]


Atomic Structure AICE (IIT-JEE)

Charge on electron : The charge on electron is found to be –1.6022 × 10–19 coulombs. Since the
electron has the smallest charge known, it is designated as unit negative charge.

e/m of electron : Charge to mass ratio of the electron is found to be –1.7588 × 108 C per g.

Nature of Anode Rays

(i) These rays travel in straight lines and cast shadow of the object placed in their path.

(ii) Like cathode rays, they also rotate the pin wheel placed in their path and also have heating
effect. Thus, the rays possess kinetic energy.

(iii) The rays produce fluorescence on zinc sulphide screen.

(iv) The rays are deflected by electric and magnetic field in the direction opposite to that of cathode
rays. These rays are attracted towards the negatively charged plate showing that they carry
positive charge.

(v) They can pass through thin metal foils.

(vi) They can produce ionisation in gases.

(vii)They are capable of producing physical and chemical changes.

(viii) Positive particles in these rays have e/m values much smaller than that of the electron.

(ix) e/m value is dependent on the nature of the gas taken in the discharge tube. Particles of highest
e/m are obtained when hydrogen is taken in the discharge take. These particles of highest e/m
are called proton.

Mass of proton: Mass of proton is found to be 1.672 × 10–27 kg or 1.0072 amu. (amu = atomic mass
unit)

Charge on proton : Charge on proton is same as charge on electron (i.e., +1.6022 × 10–19 coulombs)
in magnitude but opposite in sign.

e/m of proton: Charge to mass ratio of proton is found to be +9.579 × 104 coulombs per g. Which
is very small as compared to electron.

Rutherford’s -scattering experiment


Rutherford carried out a series of experiments using
-particles. A beam of -particles was directed against a
thin foil of gold, platinum, silver or copper. The foil was
surrounded by a circular fluorescent zinc sulphide screen.
Whenever an -particle struck the screen, it produced a
flash of light.

Amity Institute for Competitive Examinations : Phones: 24336143, 24336144, 25573111/12/13/14,95120-2431839/42 [ 70 ]


Atomic Structure AICE (IIT-JEE)

The following observations were made:


(i) Most of the -particles went straight without any deflection.
(ii) A few of them got deflected through small angles.
(iii) A very few of them (about one in 20,000) returned back towards its source.

Following conclusions were drawn from the above observations:

(i) Since most of the particles went straight through the metal foil undeflected, it indicates most of
space of an atom is empty.

(ii) A few of the -particles were deflected from their original paths through obtuse angles, it was
concluded that whole of the positive charge is concentrated and the space called nucleus. It is
supposed to be present in the centre of the atom.

(iii) A very few of the -particles suffered strong deflections or even rebound on their path due to
maximum repulsion and minimum Impact parameter.

Mosley’s Experiment and the concept of Atomic Number : Moseley studied the x-ray spectra
of 38 different elements, starting from aluminium to gold. He measured the frequency of principal
lines of a particular series (the -lines in the k-series) of the spectra. He observed that the frequency
of the particular spectral line was related with the serial number of the element in the periodic table
which he termed as atomic number (Z). He suggested the following relationship.

v  a ( Z  b)
(a) (b)

20 20
v/108 s–1

–1
v/10 s

15 15
8

10 10

5 5
20 40 60 80 100 10 20 30 40
Relative Atomic Mass (A) Atomic Number (Z)
Where v = frequency of X-rays, Z = atomic number, a and b are constants. When the values of
square root of the frequency were plotted against atomic numbers of the elements producing X-rays,
a straight line was obtained.

Chadwick’s Experiment and discovery of Neutron : Chadwick bombarded beryllium with a


stream of -particles. He observed that penetrating radiations were produced which were not affected
by electric and magnetic fields. These radiations consisted neutral particles, which were called
neutrons.
9 Be  4 He  12 C  1n
4 2 6 0
beryllium α- particle carbon neutron

Amity Institute for Competitive Examinations : Phones: 24336143, 24336144, 25573111/12/13/14,95120-2431839/42 [ 71 ]


Atomic Structure AICE (IIT-JEE)

The mass of the neutron was determined. It was 1.675 × 10–24 g, i.e., nearly equal to the mass of
proton.

Thus, a neutron is a sub-atomic particle which has a mass 1.675 × 10–24 g approximately 1
amu or nearly equal to the mass of proton or hydrogen atom and carrying no electrical
charge. The e/m of neutron is zero.

Rutherford’s Model of Atom : Rutherford proposed a model of the atom which is known as
nuclear atomic model. According to this model.

(i) An atom consists of heavy positively charged nucleus where all the protons and neutrons are
present. The magnitude of the positive charge on the nucleus is different for different atoms.

(ii) The volume of the nucleus is very small and is only a minute fraction of the total volume of the
atoms.

Diameter of atom 108 cm


 13  105.
Diameter of the nucleus 10 cm

Thus, the diameter of an atom is 100,000 times the diameter of the nucleus.

The radius of a nucleus is proportional to the cube root of the mass number.

Radius of the nucleus = 1.33 × 10–13 × A1/3 cm where A is the mass number.

(iii) There is an empty space around nucleus called extranuclear part. In this part electrons are
present. The number of electrons in an atom is always equal to number of protons present in the
nucleus. The volume of the atom is about 1015 times the volume of the nucleus.

Volume of the atom (10 8 ) 3 10 24


   1015
Volume of the nucleus (10 13 ) 3 10 39

(iv) Electrons revolve round the nucleus in closed orbits with high speeds. The centrifugal force
acting on the revolving electrons is being counter balanced by the force of attraction between
electrons and the nucleus.

This model was similar to solar system, the nucleus representing the sun and electrons the
planet. The electrons are, therefore, generally referred as planetary electrons.

Limitations of Rutherford model


Nucleus
(i) According to wave theory, a charged particle when moves under the
influence of attractive force, it loses energy continuously in the form of
electromagnetic radiations. Thus, the electron which moves in an attractive +
field (created by protons present in the nucleus), must emit radiations. As a
result the electron should lose energy at every turn and move closer and
closer to the nucleus and finally by following a spiral path it fall into the Electron

Amity Institute for Competitive Examinations : Phones: 24336143, 24336144, 25573111/12/13/14,95120-2431839/42 [ 72 ]


Atomic Structure AICE (IIT-JEE)

nucleus, thereby making the atom unstable. Since atom is quite stable, it means the electrons do
not fall into nucleus, and this model does not explain the stability of the atom.

(ii) If the electrons lose energy continuously, the observed spectra should be continuous but the
observed spectra consists of well defined lines of definite frequencies. Hence, the loss of energy
by the electrons is not continuous in an atom.

Electromagnetic radiations (wave theory): Electromagnetic radiation is generated by oscillations


of a charged body in a magnetic field or of a magnet in an electric field. These radiations or waves
have electric and magnetic fields associated with them and travel at right angle to these fields. The
following are, the important characteristics of electromagnetic radiations.

(i) These consists of electric and magnetic fields that oscillate in directions perpendicular to each
other and perpendicular to the direction in which the wave is travelling.

(ii) All electromagnetic radiations travel with same velocity, i.e., the velocity of light (3 × 108 ms–1 or
3 × 1010 cm s–1).

(iii) The electromagnetic radiations do not require any medium for transmission.

Wave
A wave is always characterised by the following five characteristics :

(i) Wavelength (ii) Frequency (iii) Velocity (iv) Wave number and

(v) Amplitude

(i) Wavelength : The distance between two consecutive crests or troughs is called the wavelength.
It is denoted by .
Crest
Wavelength

Vibrating
source Energy

Trough

(ii) Frequency : It is defined as the number of waves which pass through a point in one second. It
is denoted by the symbol, v and is measured in terms of cycles per second (cps) or Hertz (Hz)
units.

c
v
λ
or where;  = wavelength
c = velocity of height
v = frequency.

(iii) Velocity : It is defined as the distance covered by wave in one second. It is denoted by the letter
‘c’. All electromagnetic waves travel with same velocity.

Amity Institute for Competitive Examinations : Phones: 24336143, 24336144, 25573111/12/13/14,95120-2431839/42 [ 73 ]


Atomic Structure AICE (IIT–JEE)

Thus, a wave of higher frequency has a shorter wavelength while a wave of lower frequency
has a longer wavelength.

(iv) Wave number: This is reciprocal of wavelength, i.e., It is denoted by the symbol v

1
v Its unit is cm–1 or m–1.
λ
(v) Amplitude: It is defined as the height of the crest of depth of the trough of a wave. It is denoted
by letter ‘a’. It determines the intensity of the radiation.

The arrangement of various types of electromagnetic radiations in the order of increasing or


decreasing wavelengths or frequencies is known as electromagnetic spectrum.

Electromagnetic radiation Wavelength range (Å) Frequency range (Hz)

Cosmic rays 0.01 to zero 3 × 1020 to infinity

Gamma rays 0.1 to 0.01 3 × 1019 to 3 × 1020

X-rays 150 to 0.1 2 × 1016 to 3 × 1019

Ultra-violet radiation (UV) 3800 to 150 7.9 × 1014 to 2 × 1016

Visible radiation 7600 to 3800 3.95 × 1014 to 7.9 × 1014

Infra-red radiation (IR) 6 × 106 to 7600 5 × 1011 to 3.95 × 1014

Micra waves 3 × 109 to 6 × 106 1 × 109 to 5 × 1011

Radio waves 3 × 1013 to 3 × 109 1 × 105 to 1 × 109

SAMPLE PROBLEMS 3.1 (MCQ)


Problem 1: Isotopes have
(a) Same number of protons (b) Same number of neutrons
(c) Different number of electrons (d) Different atomic numbers

Solution: (a) Isotopes have same atomic number.

Problem 2: The ratio of mass of an electron to that of the mass of hydrogen atom is:
(a) 1837 : 1 (b) 1 : 2000
(c) 1 : 1837 (d) 1 : 3500

Solution: (c)

Problem 3: The number of neutrons present in deuterium is:


(a) 3 (b) 0
(c) 2 (d) 1

Solution: (d)

Amity Institute for Competitive Examinations : Phones: 24336143, 24336144, 25573111/12/13/14,95120-2431839/42 [ 74 ]


Atomic Structure AICE (IIT–JEE)

Problem 4: What happens when -rays strike a thin gold foil:


(a) Most of the -rays gets deflected through small angles
(b) Most of the rays do not pass through the foil
(c) Most of the rays passes through without any deviation
(d) Most of the rays gets deflected back

Solution: (c)

Problem 5: Wave number is equal to:


c
(a) (b) λ  ν
λ
1
(c) (d) 
λ

1
Solution: (c) ν 
λ

Problem 6: The frequency of microwaves with wavelength 2 × 106 nm is


(a) 2.5 × 106 s–1 (b) 5 × 108 s–1
(c) 1.5 × 1011 s–1 (d) 3.2 × 107 s–1

c 3  108 ms 1
Solution: (c) ν   3
 1.5 1011 s1
λ 2  10 m
Problem 7: The diameter of an atom is ______times the diameter of the nucleus
(a) 105 (b) 103
(c) 104 (d) 102

diameter of atom
Solution: (a)  105
diameter of nucleus

Problem 8: The unit of frequency is/are


(a) Cycles per second (b) Hertz
(c) Metre per second (d) Both (a) and (b)

Solution: (d) Factual

Problem 9: The limitations of Rutherford’s -ray scattering experiment was


(a) It could not explain the stability of atom
(b) It could not explain the position of nucleus
(c) It could not explain the size of atom
(d) Both (a) and (c)

Solution: (a)
Problem 10: Unit of wavelength can be
(a) cm (b) nm
(c) pm (d) all of these

Solution: (d) Factual

Amity Institute for Competitive Examinations : Phones: 24336143, 24336144, 25573111/12/13/14,95120-2431839/42 [ 75 ]


Atomic Structure AICE (IIT–JEE)

3.2 Atomic Spectrum of Hydrogen


When an electric discharge is passed through hydrogen gas at low pressure, a bluish light is emitted.
When a ray of this light is passed through a prism, a discontinuous line spectrum of several isolated
sharp lines is obtained. The wavelengths of various lines lie in visible, ultraviolet and infra-red regions.
All these lines observed in the hydrogen spectrum can be classified into five series.

Spectral series Discovered by Appearing in

Lyman series Lyman Ultra-violet region


Balmer series Balmer Visible region
Paschen series Pashcen Infra-red region
Brackett series Brackett Infra-red region
Pfund series Pfund Infra-red region

Ritz presented a mathematical relation to find the wave number of various lines of H-spectrum.

1  1 1 
v R 2  2 
λ  n1 n 2 

where R is a universal constant, known as Rydberg constant. It’s value is 109,678 cm –1


(R = 1.097 × 107 m–1) n1 and n2 are integers (such that n2 > n1). For a given spectral series n1 remains
constant while n2 varies from line to line in the same series.

The value of n1 = 1, 2, 3, 4 and 5 for the Lyman, Balmer, Paschen, Brackett and Pfund series
respectively. n2 is greater than n1 by at least 1.

Values of n1 and n2 for various series

Spectral Series Value of n1 Value of n2

Lyman series 1 2, 3, 4, 5,.....


Balmer series 2 3, 4, 5, 6,.....
Pashcen series 3 4, 5, 6, 7,.....
Bracket series 4 5, 6, 7, 8,.....
Pfund series 4 6, 7, 8, 9,.....

Quantum theory of electromagnetic radiations: The wave theory successfully explains many
optical phenomenon such as reflection, refraction, diffraction, interference, polarisation etc., but fails
to explain phenomenon like black body radiation, photo-electric effect etc.

In order to explain black body radiation and photo-electric effect a new theory was proposed which
is known as quantum theory of radiation. According to this theory, a hot body emits energy not
continuously but discontinuously in the form of small packets of energy called quantum. The energy
associated with each quantum of a given radiation is proportional to the frequency of the emitted
radiation; E 

Amity Institute for Competitive Examinations : Phones: 24336143, 24336144, 25573111/12/13/14,95120-2431839/42 [ 76 ]


Atomic Structure AICE (IIT–JEE)

or E = hv where h is a constant known as Planck’s constant. It’s numerical value is 6.624 × 10–34 Js.
The energy emitted or absorbed by a body can be either one quantum or any whole number multiple
of hv, i.e., 2hv, 3hv, 4hv,...., nhv quanta of energy.

Thus energy emitted or absorbed = nhv, where n can have values 1, 2, 3, 4,...... Thus, the energy
emitted or absorbed is quantised.

Bohr’s Model of an atom

Postulates:

(i) The whole mass is concentrated at the nucleus and hence nucleus contain protons
and neutrons.

(ii) Negatively charged electrons are revolving around the nucleus in a same way as the planets are
revolving around the sun. The path of the electron is circular. The force of attraction between
the nucleus and the electron is equal to centrifugal force of the moving electron.

(iii) The electron can revolve only in those orbits whose angular momentum is an integral multiple of

h h
, i.e., mvr  n where m = mass of electron, v = velocity of electron, r = radius of orbit
2π 2π

and n = 1, 2, 3, .... number of orbit. Thus the angular momentum is quantized. The specified
orbits are called stationary orbits.

(iv) By the time, the electron remains in any one of the stationary orbits, it does to lose energy. Such
a state is called ground state.

(v) Each stationary orbit is associated with a definite amount of energy. The greater the distance of
the orbit from the nucleus, more shall be the energy associated with it. These orbits are also
called energy levels and are numbered as 1, 2, 3, 4, ..... or K, L, M, N,..... from nucleus outwards.

(vi) The emission or absorption of energy in the form of radiation can only occur when an electron
jumps from one stationary orbit to another.
E = Ehigh –Elow = hv

Electrons excited
by absorbing energy
(energy absorbed)

+
Nucleus
Energy radiated
when electrons fall
n=1
back
(K) n = 2 n=3
(L) n=4 (Energy emitted)
(M)
(N) n = 0
(O) n = 6
(P)

Amity Institute for Competitive Examinations : Phones: 24336143, 24336144, 25573111/12/13/14,95120-2431839/42 [ 77 ]


Atomic Structure AICE (IIT–JEE)

Energy is absorbed when electron jumps from inner to outer orbit and is emitted when it moves from
outer to an inner orbit.

When the electron moves from inner to outer orbit by absorbing definite amount of energy, the new
state of the electron is said to be excited state.

Radii of various orbits mv2


r Force of
Consider an electron of mass ‘m’ and charge ‘e’ revolving around attraction
v e
the nucleus of charge Ze (Z = atomic number). Let ‘v’ be the
r
tangential velocity of the revolving electron and ‘r’ the radius of the
orbit. The electrostatic force of attraction between the nucleus and + Ze
kZe 2
electron  2 .
r
1
where k is a constant. It is equal to 4πε ,  0 being absolute permittivity of medium. In SI units, the
0

1
numerical value of 4πε is equal to 9 × 109 Nm2/c2.
0

[In C.G.S. units, value of k is equal to 1]


As force of attraction = centrifugal force
kZe 2 mv 2
So, 
r2 r
kZe 2
v2  ...(i)
rm
According to one of the postulates,

h
Angular momentum  mvr  n
2
nh
or v ...(ii)
2mr
Putting the value of ‘v’ in eq. (i)

n 2h 2 kZe 2
2 2 2

4 m r mr

n 2h 2
or  kZe 2
4 2 mr
n 2h 2
or r ...(iii)
4 2 mkZe 2
n 2h 2
For hydrogen atom Z = 1, so, r 
4 2 mke 2
Now putting the values of h, m, e and k
n2
r = 0.529 × 10–8 × cm.
z
n2
and rn  0.529  Å for hydrogen like species.
Z

Amity Institute for Competitive Examinations : Phones: 24336143, 24336144, 25573111/12/13/14,95120-2431839/42 [ 78 ]


Atomic Structure AICE (IIT–JEE)

Velocity
We know
KZe 2
v2 = .....(i)
r.m
also we know

nh
v = 2r m .....(ii)

Dividing (i) by (ii) we get

v2 KZe2 2rm
= .
v r.m nh
2 KZe 2
v=
nh
2 Ke 2
v=
h
Z
v = 2.18 × 106 . m/sec.
n

Energy of an electron: Let the total energy of the electron be E. It is sum of kinetic energy and
potential energy.

E = kinetic energy + potential energy


1 kZe 2
 mv 2 
2 r
Putting the value of mv2 from eq. (i)
kZe 2 kZe 2 kZe 2
E  
2r r 2r
Putting the value of r from eq. (iii)

2 2 Z 2 k 2 me 4
E ...(iv)
n 2h 2
For hydrogen atom, Z = 1
2 2 k 2 me 4
so, E
n2h2
Putting the values of p, k, m, e and h
13.6
E   2 eV / atom
n
313.6
E   2 kcal/mole
n
1313
E   2 kJ / mole
n
E1
E n  2 for hydrogen atom
n
Z2
and E n  E1  for hydrogen like species.
n2

Amity Institute for Competitive Examinations : Phones: 24336143, 24336144, 25573111/12/13/14,95120-2431839/42 [ 79 ]


Atomic Structure AICE (IIT–JEE)

Interpretation of hydrogen spectrum


P
The electron in the hydrogen atom O
resides in the first orbit. When energy N Pashcen
series
is supplied, the electron moves to M
higher energy shells depending on the L
Balmer K Brackett
amount of energy absorbed. When this
series series
electron returns to any of the lower
energy level, it emits energy. Lyman
Pfund
series is obtained when the electron series
returns to the lowest energy state while
Balmer series is obtained when the Lyman
electron returns to second energy shell. series
Similarly, Pashcen, Brackett and Pfund
series are obtained when electron returns to the third, fourth and fifth energy shell from higher
energy shells respectively.

Maximum number of lines produced when an electron jumps from nth level to ground level. It is equal

n ( n  1)
to . If the electron comes back from energy level having energy E2 to energy level having
2

energy E1, then the difference may be expressed in terms of energy of photon as :
E2 – E1 = E = h
s or The frequency of the emitted photon is given by
E

h
Since, E can have only definite values depending on the energies of E2 and E1, v will have only fixed
values in an atom.

c E
or V  
 h
hc
or  
E

Since h and c are constants, E corresponds to definite energy, thus, each transition from one energy
level to another will produce a light of definite wavelength. This is actually observed as a line in the
spectrum of hydrogen atom.

Thus, the different spectral lines in the spectra of atoms correspond to different transitions of electrons
from higher energy levels to lower energy levels.

Let n1 and n2 be the two energy shells in hydrogen atom (n1 < n2).

2 2 me 4 k 2
Energy associated with n1 shell (E1)   n12 h 2

Amity Institute for Competitive Examinations : Phones: 24336143, 24336144, 25573111/12/13/14,95120-2431839/42 [ 80 ]


Atomic Structure AICE (IIT–JEE)

2 2 me 4 k 2
Energy associated with n2 shell (E2)  
n 22 h 2
2 2 me 4 k 2  2 2 me 4 k 2 
E 2  E1    
n 22 h 2  n12 h 2 
2 2 me 4 k 2  1 1
E   n2  n2 
h2  1 2
c
E  hv  h.

c 2 2 me 4 k 2  1 1
h.  2  2
 2
 h  n1 n 2 
1 2 me k  1
2 4 2 1
or   n2  n2 
 h 3c  1 2

1 1 1
   RH  2  2 
  n1 n2 
1 1
or   RH  c  2  2 
 n1 n2 
where, RH is Rydberg constant for hydrogen. Its value is 109, 679 cm–1. (RH = 1.098 × 107 m–1) for
H like atoms (Li2+, He+, etc.)

1 1
v  Z2 R H  1  2 
 n1 n2 

1 1
v  Z 2 RH c  1  2 
 n1 n2 

The following points support Bohr’s theory:

(i) The frequencies of the spectral lines calculated from Bohr’s equation are in the agreement with
the frequencies observed experimentally in hydrogen spectrum.

(ii) The value of Rydberg constant calculated from Bohr’s equation tallies with the determined
experimentally.

(iii) The spectra of hydrogen like species such as

He+, Li+2, Be+3, can be explained with the help of Bohr’s theory.

Limitations of Bohr’s theory


(i) It does not explain the spectra of multi-electron species (due to inter electronic repulsion)

(ii) When a high resolving power spectroscope is used, it is observed that a spectral line in the
hydrogen spectrum is a collection of several lines which are very close to one another. Bohr’s
theory does not explain the fine spectra of even hydrogen atom.

(iii) It does not explain the splitting of spectral lines into a group of lines under the influence of
magnetic field (Zeeman effect) and electric field (Stark effect).

(iv) It is not in agreement with Heisenberg’s uncertainty principle.

Amity Institute for Competitive Examinations : Phones: 24336143, 24336144, 25573111/12/13/14,95120-2431839/42 [ 81 ]


Atomic Structure AICE (IIT–JEE)

Photoelectric Effect: The emission of electrons from the surface of a metal when illuminated by
electromagnetic radiation of suitable wavelength of frequency is called photoelectric effect. For a
given material, there exists a minimum frequency or incident radiation which emits photoelectrons. If
the frequency is less than that no electron can be emitted from the metal surface, no matter how
intense the radiation may be. This minimum value of frequency is called threshold frequency. The
energy of the photoelectrons is proportional to the frequency of the incident radiation and the number
of photoelectrons emitted per second is proportional to the intensity of the incident radiation.

Accepting to Planck’s idea that radiation consists of photons, Einstein suggest that photon’s energy
is used in two ways:

(i) A part of energy is used to free the electron from the atom. This energy is known as photoelectric
work function. This is denoted by .

(ii) Rest of the energy is used in giving kinetic energy to the electron. Thus, energy of the photon,

hv    12 mv2 ...(i)

where, v is the velocity of the emitted electron .


When photon’s energy is equal to  the kinetic energy becomes zero. Under this condition, eq. (i)
reduces to
hv0 = 
where, v0 is the threshold frequency, from eq. (i)
1
2 mv2  hv 

Putting the value of 0,


1 mv 2  hv  hv 0
2

1 mv 2  h (v  v0 )
2

For a definite metal, the value of hv0 is constant

So, 1 mv 2  hv
2

or v2  v
Thus, increase of frequency of the incident radiation causes an increase in the velocity of electron
provided intensity of incident light is constant.

Wave nature of electrons (de-Broglie’s hypothesis): de-Broglie proposed that


electrons like light, behaves both as material particle and as a wave.

de-Broglie derived an expression for calculating the wavelength of the wave associated with the
electron. According to Planck’s equation
c
E  h  h. .....(i)

The energy of a photon on the basis of Einstein’s mass energy relationship is
E = mc2 .....(ii)
where c is the velocity of electron

Amity Institute for Competitive Examinations : Phones: 24336143, 24336144, 25573111/12/13/14,95120-2431839/42 [ 82 ]


Atomic Structure AICE (IIT–JEE)

Equating both the equations


c
h.  mc 2

h
or  .....(iii)
mc
The wavelength decreases if the value of mass (m) increases, de-Broglie equation is applicable in
the case of smaller particles like electron and has no significance for large particles.

SAMPLE PROBLEMS 3.2 (MCQ)


Problem 1: Bohr’s theory is not applicable to:
(a) Be2+ (b) H
+
(c) He (d) Li2+

Solution: (a) Bohr’s theory is applicable only to hydrogen and H-like species.

Problem 2: n1 for Balmer series of hydrogen spectrum where e– falls is:


(a) 3 (b) 2
(c) 4 (d) 1

Solution: (b) For Balmer series n1 = 2, n2 > 2

Problem 3: Radius of Bohr’s orbit of H-atom is


(a) 0.53Å (b) 0.26Å
(c) 1.25Å (d) 2.57Å

n2
Solution: (a) rn  0.53 
Z
Problem 4: Which of the following transitions in the H-atom will emit most energetic photon as per
Bohr’s theory?
(a) n = 5 to n = 3 (b) n = 6 to n = 5
(c) n = 2 to n = 1 (d) n = 6 to n = 1

Solution: (d) Difference of energy is maximum between orbit 6 and 1

Problem 5: The number of spectral lines observed when an electron falls from n = 5 to n = 1 in visible
region is
(a) 5 (b) 3
(c) 2 (d) 4

Solution: (b) 5  2, 4  2, 3  2 (3 lines)

Problem 6: The energy of the 3rd orbit of H-atom is


(a) –0.342 × 10–19J (b) –0.726 × 10–18 J
(c) –2.18 × 10–18 J (d) –2.42 × 10–19 J

Solution: (d) E n  R H  Z2 / n 2

Amity Institute for Competitive Examinations : Phones: 24336143, 24336144, 25573111/12/13/14,95120-2431839/42 [ 83 ]


Atomic Structure AICE (IIT–JEE)

Problem 7: The K.E. of photoelectrons depends upon:


(a) Intensity of incident rays (b) Frequency of incident rays
(c) Number of photons striking (d) Number of photoelectrons ejected

Solution: (b) Factual

Problem 8: The work function () of a metal having threshold frequency (0) 5.2 × 1014 s–1 is:
(a) 2.19 × 10–10 J (b) 1.23 × 10–22 J
(c) 3.44 × 10–19 J (d) 7.64 × 10–15 J

Solution: (c) Work function () = h0

Problem 9: The ratio of the radii of the three Bohr’s orbit is:
(a) 1 : 5 : 3 (b) 1 : 4 : 9
(c) 1 : 8 : 27 (d) 1 : 3 : 9

n2
Solution: (b) rn  0.529 
Z

Problem 10: A certain metal irradiated to light of frequency 3.2 × 1016 Hz emits photoelectrons with
twice K.E. as did photoelectrons when the same metal is irradiated by light of frequency
2 × 1016 Hz. The threshold frequency of metal is
(a) 1.2 × 1016 Hz (b) 4 × 1015 Hz
(c) 8 × 1015 Hz (d) 9 × 1014 Hz

Solution: (c) Threshold frequency (0) = h0

3.3 Bohr’s theory versus de-Broglie Equation


One of the postulates of Bohr’s theory is that angular momentum of an electron is an integral

h
multiple of . This postulate can be derived with the help of de-Broglie concept of wave nature of
2
electron.

Consider an electron moving in a circular orbit around nucleus. The wave would be associated with
the circular orbit. If the two ends of the electron wave meet to give regular series of crests and
troughs, the electron wave is said to be in phase, i.e., the circumference of Bohr’s orbit is equal to
whole number multiple of the wavelength of the electron wave.

so, 2r = n

2r
or  .....(i)
n

From de-Broglie Equation (In phase) (Out of phase)

h
 .....(ii)
mv
Amity Institute for Competitive Examinations : Phones: 24336143, 24336144, 25573111/12/13/14,95120-2431839/42 [ 84 ]
Atomic Structure AICE (IIT–JEE)

h 2r
Thus, 
mv n
h
or mvr  n. [v = velocity of electron and r = Radius of the orbit]
2

h
i.e., Angular momentum  n. ...(iii)
2
This proves that de-Broglie and Bohr-concepts are in perfect agreement with each other.

Heisenberg Uncertainty Principle: Werner Heisenberg presented a principle known as


Heisenberg uncertainty principle which states: “It is impossible to measure simultaneously the
exact position and exact momentum of a body as small as electron.”

The uncertainty of measurement in position, x, and the uncertainty of momentum, p or mv, are
related by Heisenberg’s relationship as

x.p  h/4
or x.mv  h/4 where, h is Planck’s constant.

For an electron of mass m (9.10 × 10–28 g), the product of uncertainty is quite large.

In the case of bigger particle (having a considerable mass), the value of the product of uncertainty is
negligible.

If the position is known quite accurately i.e., is very small, v becomes large and vice-versa. Thus,
uncertainty principle is important only in the case of smaller moving particles like electrons.

Quantum Numbers

The four quantum numbers are :

(i) Principal quantum number (ii) Azimuthal quantum number

(iii) Magnetic quantum number (iv) Spin quantum number

Each quantum number refers to a particular character of the electron in an atom. First three quantum
numbers have been deduced from Schrodinger wave equation.

Quantum or Wave mechanical model of atom: This new model of atom was put forward by
Schrodinger in 1920 by taking into account the de Broglie concept of dual nature of matter and
Heisenberg’s uncertainty principle. He described the motion of the electron in three dimensional
space in terms of a mathematical equation called Schrodinger wave equation viz.

 2  2  2 8 2 m
   (E – V)  = 0
x 2 y 2 z 2 h2

where  is the amplitude of the electron wave at a point with coordinates x, y, z, E = total energy and
V = potential energy of the electron y is also called wave function.  2 gives the probability of finding
the electron at (x, y, z). The acceptable solutions of the above equation for the energy E are called
eigen values and the corresponding wave function  are called eigen functions.

Amity Institute for Competitive Examinations : Phones: 24336143, 24336144, 25573111/12/13/14,95120-2431839/42 [ 85 ]


Atomic Structure AICE (IIT–JEE)

Schrodinger wave equation can be written as


 2 2  2  8 2 m
    
 x 2 y 2 z 2  (E – V)  = 0
  h2
8 2 m
or 2  (E – V) = 0
h2
2 2 2 2
where     is called Laplacian operator
x 2 y 2 z 2
The above equation can be rewritten as
8 2 m
 2  – (E – V) 
h2
 2 
 – h 2  V 
or  8 2 m  =E
 
or H = E

h2
where H  – 2
 2  V where H is called
8 m

In this operator, first term represents kinetic energy operator (T) and second term represents potential
energy operator (V). Hence Schrodinger wave equation can also be written as

(T + V)  = E 

Plots of Radial Wave Function (R), Radial Probability Density (R2) and Radial Probability
Function (4  r2R2): The probability of funding the electron in a spherical shell of thickness dr at
a distance r from the nucleus is equal to the product of the volume of shell of thickness dr at distance
r from the nucleus (4 r2dr) and the radial probability density (R2) per unit volume, i.e., equal to
4r2 dr R2 = P dr where P = 4r2R2 is called radial probability function.

2p
1s 2s
R R
R
(a) NODE
+
r r – r
2p
1s NODE 2s
R2 R
2
R
2

(b)

r r r

Amity Institute for Competitive Examinations : Phones: 24336143, 24336144, 25573111/12/13/14,95120-2431839/42 [ 86 ]


Atomic Structure AICE (IIT–JEE)

14
12 2p
10 1s 5 5
2s

4r2R2
8

4r2R2

4r2R2
4 4
6 3 3
4 2 2
(c) 2 1 1 2.1 Å
2.7 Å
0 0 0
2 4 6 8 2 4 6 8 2 4 6 8
r(Å) r(Å) r(Å)
0.529 Å
In the plots of radial probability versus distance from the nucleus, number of peaks, i.e. region of
maximum probability = n – l, e.g., 2s has two peaks 3s will have 3 peaks, 2p has one peak, 3p has
two peaks and so on.

Note that the nodal surface of 2s orbital exists at a distance of 2a0 from the nucleus where a0 is the
Bohr radius, i.e., 0.529 Å.

Principle quantum number: It describes the average distance of the electron from the nucleus. It
is designated as ‘n’. The energy of the electron and the volume of the electron cloud are dependent
on this quantum number. Theoretically it may have all integral values from 1 to  but only values
from 1 to 7 have been established in the atoms of known elements so far,

Higher the value of ‘n’:

(i) Greater is the distance of the shell from nucleus,


(ii) Greater is the magnitude of energy.

The maximum number of electron which can be present in a principal energy shell is equal to 2n2.

No energy shell in the atoms of known elements possess more than 32 electrons.

Azimuthal quantum number : This quantum number is also called secondary or subsidiary quantum
number. It is designated by symbol ‘l’. It describes spatial distribution of electron cloud and the
angular momentum of the electron. It actually tells about the shape of orbital occupied by the electron.
For any given value of principal quantum number ‘n’, the azimuthal quantum number ‘l’ can have all
integral values from 0 to (n – 1). Each value corresponds to an energy sub-shell or sublevel. The total
number of sublevels in each principal shell is numerically equal to the value of ‘n’.

The sub shells are designated by the letters s, p, d and f.

l=0 s-sub-shell; l=2 d-sub-shell


l=1 p-sub-shell; l=3 f-sub-shell

s, p, d and f are spectral terms and signify sharp, principal, diffuse and fundamental respectively.

The energies of the various sub-shells in the same shell are in order of s < p < d < f. Sub-shells
having equal l values but with different n values have similar shapes but their sizes increase as the
value of ‘n’ increases.

Magnetic quantum number: This quantum number is designated by the symbol ‘m’. To explain
splitting of spectral line into a number of closely spaced lines in the presence of magnetic field

Amity Institute for Competitive Examinations : Phones: 24336143, 24336144, 25573111/12/13/14,95120-2431839/42 [ 87 ]


Atomic Structure AICE (IIT–JEE)

(Zeeman effect), electron producing a single line has several possible space orientations for the
same angular momentum vector in a magnetic field, i.e., under the influence of magnetic field each
sub-shell is further sub-divided into orbitals. Magnetic quantum number describes the orientation or
distribution of electron cloud. For each value of ‘l’, the magnetic quantum number ‘m’ may assume
2l + 1 values ranging from –l to +l including zero. Each value of “m” represent on ORBITAL, which
contains maximum of two elements, with opposite spins, m for pz is zero.

Characteristics of orbitals
(i) All orbitals of the same shell in the absence of magnetic field possess same energy, i.e., they are
degenerate.
(ii) All orbitals of the same sub-shell differ in the direction of their space orientation.
(iii) Total number of orbitals in a main energy shell is equal to n2.
Spin quantum number: When spectral lines were observed by instruments of high resolving power,
each line was found consisting of a pair of lines. To account for these doublets, it was suggested that
electron while moving around the nucleus in an orbit, is also spinning about its own axis either in
clockwise or in anti-clockwise direction. There are two possible values either. These spins are also
shown by arrows; upwards () or downwards ().

Spin  12 ­ clockwise

Spin  12 ¯ anti-clockwise

The electrons with the same sign of spin quantum numbers are said to have parallel spins while those
having opposite signs of spin quantum numbers are said to have opposite spins or paired up spins.
N Magnetic field S

+1/2 –1/2

S N
Since, a spinning charge is associated with a magnetic field, an electron must have a magnetic
moment associated with it.
Pauli’s Exclusion Principle: The principle states that no two electrons in an atom can have
the same set of all the four quantum numbers. In other words, an orbital cannot have more than
two electrons.
Conclusion :
(i) The maximum capacity of a main energy shell is equal to 2n2 electrons.
(ii) The maximum capacity of a sub-shell is equal to 2(2l + 1) electrons.
(iii) Number of sub-shells in a main energy shell is equal to the value of n.
(iv) Number of orbitals in a main energy shell is equal to n2.
(v) One orbital cannot have more than two electrons. If two electrons are present, their spins should
be in opposite directions.

Amity Institute for Competitive Examinations : Phones: 24336143, 24336144, 25573111/12/13/14,95120-2431839/42 [ 88 ]


Atomic Structure AICE (IIT–JEE)

Aufbau’s Principle: This principle gives us a sequence in which various sub-shells are filled with
electrons. The sequence in which various sub-shells are filled up depends on the relative order of the
energy of sub-shells. The sub-shell with minimum energy is filled up first and when this
attains maximum quota of electrons, then next sub-shell of higher energy starts filling.

The sequence in which the various sub-shells are filled is the following:

1s, 2s, 2p, 3s, 3p, 4s, 3d, 4p, 5s, 4d, 5p, 6s, 4f, 5d, 6p, 7s, 5f, 6d, 7p

1s 2s 3s 4s 5s 7s

2p 3p 4p 5p 7p

3d 4d 5d

4f

The sequence in which various sub-shells are filled up can also be determined with the help of (n +
1) value for a given sub-shell. The sub-shell with lowest (n + l) value is filled up first. When
two or more sub-shells have same (n + l) value, the sub-shell with lowest value of ‘n’ is
filled up first.

Hund’s Rule of Maximum Multiplicity (Orbital Diagrams): It states that electrons are
distributed among the orbitals of a sub-shell in such a way as to give the maximum number
of unpaired electrons with parallel spins. Thus, the orbitals available in a sub-shell are first filled
singly before they begin to pair. This means that pairing of electrons occurs with the introduction of
second electron in s-orbital, the fourth electron in p-orbitals, sixth electron in d-orbitals and eighth
electron in f-orbitals.

The rule is based on the fact that electrons being of the same charge repel each other and hence try
to keep farther apart from each other as much as possible. The electrons thus, occupy different
orbitals of the sub-shell as to minimise the inter-electronic repulsion and increase the stability of the
atom. Orbitals tend to become half filled or completely filled since such an arrangement will be more
stable on account of symmetry.

All those atoms which consist of at least one of the orbitals singly occupied behave as paramagnetic
materials because these are weakly attracted to a magnetic field, while all those atoms in which all
the orbitals are doubly occupied behave as diamagnetic materials because they have no attraction
for magnetic field. However, these are slightly repelled by magnetic field due to induction.

Amity Institute for Competitive Examinations : Phones: 24336143, 24336144, 25573111/12/13/14,95120-2431839/42 [ 89 ]


Atomic Structure AICE (IIT–JEE)

SAMPLE PROBLEMS 3.3 (MCQ)


Problem 1: In which of the following orbitals, the sum of (n + l) value is lowest?
(a) 4s (b) 4p
(c) 5s (d) 3d

Solution: (a)

Problem 2: The maximum number of electrons that can be accommodated in d x 2  y 2 orbital is

(a) 5 (b) 2
(c) 1 (d) 3

Solution: (b) An orbital can accommodate only 2 electrons.

Problem 3: The maximum value of ‘m’ for a 3p orbital is


(a) +1 (b) +2
(c) +3 (d) +4

Solution: (a) For a p-orbital m = +1, 0, –1

Problem 4: The correct sequence of energy of orbitals in a multielectron species is:


(a) 4s < 4p < 3d (b) 4s < 3d < 4p
(c) 3d < 4s < 4p (d) 4p < 3d < 4s

Solution: (b) Use (n + l) to calculate the energy.

Problem 5: The number of electrons present in ‘L’ shell of phosphorus (Atomic number = 15) is
(a) 2 (b) 6
(c) 10 (d) 8

Solution: (d) K shell = 2 electrons


L shell = 8 electrons
M shell = 5 electrons

Problem 6: Which of the following ions contains maximum number of unpaired electrons?
(a) Fe2+ (b) Zn+
(c) Cu+ (d) Ti2+

Solution: (a) Write the electronic configuration of each ion.

Problem 7: In which of the following orbital the four lobes are present along the axis?

(a) d xy (b) d x 2  y 2

(c) d yz (d) d xz

Solution: (b)

Amity Institute for Competitive Examinations : Phones: 24336143, 24336144, 25573111/12/13/14,95120-2431839/42 [ 90 ]


Atomic Structure AICE (IIT–JEE)

Problem 8: The four quantum numbers of an electron present in valence shell of potassium atom are:
1 1
(a) 4,1,0, (b) 4,1,1,
2 2
1 1
(c) 4,0,0, (d) 4,0,1,
2 2
Solution: (c) Last electron of potassium is present in 4s orbital.

Problem 9: Maximum number of unpaired electrons present in a f-subshell is


(a) 3 (b) 5
(c) 9 (d) 7

Solution: (d) f-subshell has 7-orbitals.

Problem 10: Which of the following atom cannot have electrons in d-subshell?
(a) Si (b) Ca
(c) Cu (d) C

Solution: (d)

SOLVED EXAMPLES
Example 1 : Calculate the radius of the first Bohr orbit of a hydrogen atom. Given that h = 6.63 × 10–27 erg.
sec; m = 9 × 10–28 g and charge of an electron = 4.8 × 10 –10 esu.

Solution: We have, for hydrogen Z = 1, n = 1


h2
r1 
4 2 me 2
27 2
(6.63 10 )
 2 28 10 2
4  (3.14)  9  10  ( 4.8 10 )
= 0.53 × 10–8 cm = 0.53 Å.

Example 2 : Calculate the ionisation energy in eV of a hydrogen atom in the ground state.

Solution : Since ionisation energy is defined as the energy required to remove an electron from outermost
orbit of atom, we can mathematically say that since the hydrogen atom is in ground state,
ionisation energy will be the energy required to remove the electron from the first orbit to 
orbit (i.e., r1 to r).
Thus,
Ionisation energy = E – E1
= 0 – (2.18 × 10–11) erg.
We know that E1 = – 2.18 × 10–11 erg.
11
2.18  10
 Ionisation energy = 2.18 × 10 –11
erg.  12
 13.60 eV
1.602  10

Example 3 : Calculate the wavelength in angstrom of the photon that is emitted when an electron in Bohr
orbit n = 2 returns to the orbit n = 1 in the hydrogen atom. The ionisation potential of the
ground state of hydrogen atom is 2.17 × 10–11 erg per atom.

Amity Institute for Competitive Examinations : Phones: 24336143, 24336144, 25573111/12/13/14,95120-2431839/42 [ 91 ]


Atomic Structure AICE (IIT–JEE)

Solution : Since the hydrogen atom has only one orbit containing only one electron, the ionisation
potential of the ground state of the, hydrogen atom is the energy of the electron of the first
orbit, i.e.,
E1 = –2.17 × 10–11 erg.

E
Thus, E2  2
.....(Eqn. 5)
n

2.17  10 11
= – 2
(n = 2)
2
 Energy of the radiation emitted,
E = E2 – E1
2.17 1011
= 2
 (2.17 1011 )
2
= 1.627 × 10–11 erg
hc
We know E = h 

hc 11
Thus,  1.627 10

27 10
6.62  10  3  10 5
 11
 1.22  10 cm
1.627  10
= 1220Å (1 Å = 10–8 cm)

Example 4 : The vapour of Hg absorbs some electrons accelerated by a potential difference of 4.5 volt
as a result of which light is emitted. If the full energy of a single incident electron is supposed
to be converted into light emitted by single Hg atom, find the wave number (1/) of the light.

Solution: We know that electron volts = charge in coulombs × potential difference in volts

 Charge of an electron   Potential difference 


4.5 eV =   
 in coulomb   in volt 

= (1.6022 × 10–19) (4.5) joule


Now,

1 E
wave number = 
 h.c
19
(1.6022  10 ) (4.5)
= 34 8
(6.626  10 ) ( 2.998 10 )
= 3.63 × 106 metres–1.

Example 5 : 13.6 eV is needed for ionisation of a hydrogen atom. An electron in a hydrogen atom in its
ground state absorbs 1.50 times as much energy as the minimum required for it to escape
from the atom. What is the wavelength of the emitted electron? (me = 9.109 × 10–31 kg, e =
1.602 × 10–19 coulomb, h = 6.63 × 10–34 J.s.)

Amity Institute for Competitive Examinations : Phones: 24336143, 24336144, 25573111/12/13/14,95120-2431839/42 [ 92 ]


Atomic Structure AICE (IIT–JEE)

Solution : 1.5 times of 13.6 eV, i.e., 20.4 eV is absorbed by the hydrogen atom out of which 6.8 eV
(20.4 – 13.6) is converted to kinetic energy.
KE = 6.8 eV = 6.8 (1.602 × 10–19 coulomb) (1 volt) = 1.09 × 10–18 J

1 2
Now, KE  mv
2

KE 2(1.09 10 18 J)


or, v  2 
m (9.109 10 31 kg )
6
= 1.55 × 10 metres.
34
h 6.63 10
   = 37 6 = 4.70 × 10–10 meters.
mv (9.109 10 ) (1.55 10 )



Amity Institute for Competitive Examinations : Phones: 24336143, 24336144, 25573111/12/13/14,95120-2431839/42 [ 93 ]


Atomic Structure AICE (IIT–JEE)

CHAPTER ASSIGNMENT
MULTIPLE CHOICE QUESTIONS (Single Correct)
1. The radius of hydrogen atom in the ground state is 0.53 Å, the radius of Be3+ ion in the similar
state is
(a) 1.06 Å (b) 0.1325 Å (c) 0.18 Å (d) 0.53 Å

2. The wave number of first line of Balmer series of hydrogen is 152,000 cm–1. The wave number of
first Balmer line of Li2+ ion is
(a) 15,200 cm–1 (b) 60,800 cm–1 (c) 76,000 cm–1 (d) 1,368,000 cm–1

3. The wavelength of a spectral line for an electronic transition is not inversely related to
(a) The nuclear charge of the atom
(b) The difference in the energy of the energy-levels involved in the transition
(c) The velocity of the electron undergoing the transition
(d) Both (a) and (c)

4. If n and 1 are respectively the principal and azimuthal quantum numbers, then the expression for
calculating the total number of electrons in any energy level is
l nl l  n l l n l l nl
(a)  2(2l 1) (b)  2(2l  1) (c)  2( 2l  1) (d)  2( 2l 1)
l 0 l 1 l 0 l 0

5. If r1 is the radius of first orbit of hydrogen atom, then the radii of second, third and fourth orbits in
terms of r1 are
(a) r21, r31, r41 (b) 8r1, 27r1, 64r1 (c) 4r1, 9r1, 16r1 (d) 2r1, 6r1, 8r1

6. Magnetic quantum number signifies


(a) Orientation of orbitals in space (b) Size of orbitals
(c) Shape of orbitals (d) Stability of orbitals

7. Which of the following statement is false ?


(a) When an electron jumps from one orbital to another, energy is either emitted or absorbed
(b) Azimuthal quantum number determines the number of subshells
(c) No two electrons in an atom can have all the four quantum numbers alike
(d) Orbital of higher energy is filled up first before the filling of an orbital of lower energy

8. The kinetic energy of the electron emitted when light of frequency 3.5 × 1015 Hz falls on a metal
surface having threshold frequency 1.5 × 1015 Hz is (h = 6.6 × 10–34 Js)
(a) 1.32 × 10–18 J (b) 3.3 × 10–18 J (c) 6.6 × 10–19 J (d) 1.98 × 10–19 J

9. The kinetic energy of the photoelectrons does not depend upon :


(a) Intensity of incident radiation (b) Frequency of incident radiation
(c) Wavelength of incident radiation (d) Wave number of incident radiation

10. Which of the following set of quantum numbers is correct for the 19th electron of chromium?
(a) n = 3, l = 0, m = 0, s = 1/2 (b) n = 3, l = 2, m = –2, s = 1/2
(c) n = 4, l = 0, m = 0, s = 1/2 (d) n = 4, l = 1, m = –1, s = –1/2

Amity Institute for Competitive Examinations : Phones: 24336143, 24336144, 25573111/12/13/14,95120-2431839/42 [ 94 ]


Atomic Structure AICE (IIT–JEE)

11. The uncertainty in the position of a moving bullet of mass 10 gm is 10–5m. Calculate the uncertainty
in its velocity
(a) 5.2 × 10–28 m/sec (b) 3.0 × 10–28 m/sec (c) 5.2 × 10–22 m/sec (d) 3 × 10–22 m/sec

12. In a multi-electron atom, which of the following orbitals described by the three quantum number will
have the same energy in the absence of magnetic and electric fields ?
(i) n = 1, l = 0, m = 0 (ii) n = 2, l = 0, m = 0
(iii) n = 2, l = 1, m = 1 (iv) n = 3, l = 2, m = 0
(v) n = 3, l = 2, m = 0
(a) (i) and (ii) (b) (ii) and (iii) (c) (iii) and (iv) (d) (iv) and (v)

13. The wavelength of the radiation emitted, when in a hydrogen atom electron falls from infinity to
stationary state 1, would be (Rydberg constant = 1.097 × 107 m–1)
(a) 406 nm (b) 192 nm (c) 91 nm (d) 9.1 × 10–8 nm

14. In Bohr Series of lines of hydrogen spectrum, the third line from the red end corresponds to which
one of the following inter-orbit jumps of the electron for Bohr orbits in an atom of hydrogen
(a) 5  2 (b) 4  1 (c) 2  5 (d) 3  2

15. The de Broglie wavelength of a tennis ball of mass 60 g moving with a velocity of 10 metres per
second is approximately
(a) 10–31 metres (b) 10–16 metres (c) 10–25 metres (d) 10–33 metres
Planck’s constant, h = 6.63 × 10–34 Js

INTEGER TYPE QUESTIONS


16. The wavelengths of two photons are 2000 Å and 4000 Å respectively. What is the ratio of their
energies ?

17. The total spin resulting from a d7 configuration is x/2. Find the value of x.

18. There are three energy levels in an atom. How many spectral lines are possible in its emission
spectra?

19. The number of electrons in the valence shell of calcium is

20. For principle quantum number n = 4 the total number of orbitals having  = 3 is

21. Number of unpaired electrons in 1s2 2s2 2p3 is –

22. How many unpaired electrons are present in Ni2+ cation (atomic number = 28)?

23. The minimum value of spin multiplicity possible, when  = 3, is

24. With a certain exciting radiation of a particular frequency, to which hydrogen atoms are exposed, the
maximum number of spectral lines obtainable in the emission is 15. The uppermost energy level to
which the electron is excited is n =

25. The total number of electrons that can be accommodated in all the orbitals having principal quantum
number 2 and azimuthal quantum number 1 is :

Amity Institute for Competitive Examinations : Phones: 24336143, 24336144, 25573111/12/13/14,95120-2431839/42 [ 95 ]


Atomic Structure AICE (IIT–JEE)

MULTIPLE CHOICE QUESTIONS (More than one Correct)


26. For cathode rays the value of e/m
(a) Is independent of the nature of the cathode and the gas filled in the discharge tube
(b) Is constant
(c) Is –1.7588 × 108 coulombs/g
(d) Is lowest when hydrogen gas is filled in discharge tube.

27. Which of the following statement/s with regard to quantum number is/are correct?
(a) The azimuthal quantum number gives the contribution of energy due to angular momentum
towards the total energy of the electron
(b) The azimuthal quantum number gives the relative energies of subshells belonging to the same
shell
h
(c) The orbital angular momentum is given by azimuthal quantum number which is equal to (  1)
2
(d) The orbital angular momentum depends on the value of ‘n’

28. Choose the correct statement/s


(a) The nodal surface of 2s-orbital exists at a distance 1.058 Å from the nucleus.
(b) In the plots of radial probability versus distance from the nucleus, the number of peaks is equal
to (n – ).
(c) Wave function of electrons in atoms and molecules are called orbitals.
(d) The azimuthal quantum number determines the shape of an orbital.

29. Which of the following statement(s) regarding Bohr theory is/are correct ?
(a) Kinetic energy of an electron is half of the magnitude of its potential energy.
(b) Kinetic energy of an electron is negative of total energy of electron
(c) Energy of electron decreases with increase in the value of principal quantum number
(d) The ionization energy of H-atom in the first excited state is the negative of one fourth of the
energy of an electron in the ground state.

30. If the wave number of 1st line of Balmer series of H-atom is ‘x’ then –
108x
(a) wave number of 1st line of lyman series of He+ ion will be
5
36x
(b) wave number of 1st line of lyman series of He+ ion will be
5
5
(c) the wave length of 2nd line of lyman series of H-atom is
32x
32x
(d) the wave length of 2nd line of lyman series of H-atom is
5

31. In a hydrogen like sample two different types of photons A and B are produced by electronic transi-
tion. Photon B has it’s wavelength in infrared region. If photon A has more energy than B, then the
photon A may belong to the region.
(a) ultraviolet (b) visible (c) infrared (d) None of these

Amity Institute for Competitive Examinations : Phones: 24336143, 24336144, 25573111/12/13/14,95120-2431839/42 [ 96 ]


Atomic Structure AICE (IIT–JEE)

32. In a hydrogen like sample electron is in 2nd excited state, the binding energy of 4th state of this
sample is 13.6eV, then
(a) A 25 eV photon can set free the electron from the second excited state of this sample.
(b) 3 different types of photon will be observed if electrons make transition up to ground state from
the second excited state
(c) If 23 eV photon is used then K.E. of the ejected electron is 1eV.
(d) 2nd line of Balmer series of this sample has same energy value as 1st excitation energy of H
atoms
33. Identify the correct statement(s) –
(a) |  | is the probability of finding the electron in an orbital
(b) p-orbital is directional in nature
(c) d has dumb bell shape along x and y-axis
x2  y2

(d) d has dumb bell shape along x and y-axis


z2

34. Which of the following properties are proportional to the energy of electromagnetic radiation?
(a) frequency (b) wave number
(c) wavelength (d) number of photon
35. Reduction of the metal centre in aqueous permanganate ion involves
(a) 3 electrons in neutral medium (b) 5 electrons in neutral medium
(c) 3 electrons in alkaline medium (d) 5 electrons in acidic medium

MATRIX MATCH TYPE QUESTIONS


36. Match the column correctly –
Column I Column II
A. If P.E. = –13.6 eV, total energy will be (p) 21
B. Ionization energy of electron from 2nd shell of Na10+ (q) 10
C. Number of spectral lines when electron jumps from 7th to 3rd shell. (r) – 6.8
D. Number of spectral lines when electron come from 7th shell to 1st shell (s) 411.4
37. Match the column correctly –
Column I Column II

h
A.   (p)  2 is mathematical symbol of an orbital
2m(KE)

h  h 
B. x  p  (q)  (  1)  
4  2 

C. Schrodinger’s equation (r) de Broglie’s concept

D. orbital angular momentum (s) Simultaneous and accurate determination of


position and momentum of electron at a point
is not possible.

Amity Institute for Competitive Examinations : Phones: 24336143, 24336144, 25573111/12/13/14,95120-2431839/42 [ 97 ]


Atomic Structure AICE (IIT–JEE)

COMPREHENSION TYPE QUESTIONS


Comprehension - I

The behaviour of an electron in an atom is described mathematically by a wave function , or orbital . It turns
out that each wave function contains three variables called quantum numbers , which are represented as
n ,  and m. These quantum numbers describe the energy level of an orbital and define the shape and
orientation of the region in space where the electron will be found .

38. Which quantum number determines orientation of the electron ?


(a) Principal (b) Secondary (c) Magnetic (d) Spin

39. Radial nodes are maximum in :


(a) 4s (b) 4p (c) 3d (d) 5f

40. Total number of electrons in any energy level is :

 n   n 1   n 1   n 1
(a)  2 (2+ 1) (b)  2 (2 + 1) (c)  2 (2+ 1) (d)  2 (2 + 1)
 1  1  0  0

Comprehension - II

The hydrogen-like species Li2+ is in a spherically symmetric state S1 with one radial node. Upon absorbing
light the ion undergoes transition to a state S2. The state S2 has one radial node and its energy is equal to the
ground state energy of the hydrogen atom.

41. The state S1 is


(a) 1s (b) 2s (c) 2p (d) 3s
42. Energy of the state S1 in units of the hydrogen atom ground state energy is
(a) 0.75 (b) 1.50 (c) 2.25 (d) 4.50
43. The orbital angular momentum quantum number of the state S2 is
(a) 0 (b) 1 (c) 2 (d) 3



Amity Institute for Competitive Examinations : Phones: 24336143, 24336144, 25573111/12/13/14,95120-2431839/42 [ 98 ]


Atomic Structure AICE (IIT–JEE)

PREVIOUS YEAR QUESTIONS


1. Rutherford’s experiment on scattering of particles showed for the first time that the atom has
(a) electrons (b) protons (c) nucleus (d) neutrons [IIT 1981]

2. How many unpaired electrons are there in Ni2+? [IIT 1981]


(a) 0 (b) 2 (c) 4 (d) 8

3. The radius of atomic nucleus is of the order of [IIT 1985]


(a) 10–12 m (b) 10–8 m (c) 10–15 m (d) 10–10 m

4. Any p-orbital can accommodate up to [IIT 1983]


(a) four electrons (b) two electrons with parallel spin
(c) six electrons (d) two electrons with opposite spin

5. Rutherford’s scattering experiment is related to the size of the [IIT 1983]


(a) Nucleus (b) Atom (c) Electron (d) Neutron

6. The principal quantum number of an atom is related to the [IIT 1983]


(a) size of the orbital (b) spin angular momentum
(c) orbital angular momentum (d) orientation of the orbital in space

7. When alpha particles are sent through a thin metal foil, most of them go straight through the foil
because [IIT 1984, 94]
(a) alpha particles are much heavier than electron
(b) alpha particles are positively charged
(c) alpha particles move with high velocity
(d) most part of the atom is empty

8. The electron level which allows the hydrogen to absorb photons but not to emit is [IIT 1984]

(a) 1s (b) 2s (c) 2p (d) 3d

9. The increasing order of the values of e/m (charge/mass) is [IIT 1984]


(a) e, p, n, a (b) n, p, e, a (c) n, p, a, e (d) n, a, p, e

10. Which one of the following sets of quantum numbers represents an impossible arrangement?
(a) n = 3, l = 2, ml = –2, ms = 1/2 (b) n = 4, l = 0, ml = 0, ms = 1/2
(c) n = 3, l = 2, ml = –3, ms = 1/2 (d) n = 5, l = 3, ml = 0, ms = –1/2 [IIT 1986]

11. The orbital diagram in which Aufbau principle is violated is [IIT 1988]

(a)    (b)    

(c)     (d)    

Amity Institute for Competitive Examinations : Phones: 24336143, 24336144, 25573111/12/13/14,95120-2431839/42 [ 99 ]


Atomic Structure AICE (IIT–JEE)

12. Which one of the following is the smallest in size? [IIT 1989]
(a) N 3– (b) O 2– (c) F – (d) Na +

13. If the speed of electron in the Bohr’s first orbit of hydrogen atom be x, then speed of the electron in
the 3rd orbit is [IIT 1990]
(a) x/9 (b) x/3 (c) 3x (d) 9x

14. Which of the following relates to photons both as wave motion and as a stream of particles?
(a) Interference (b) E = mc2 (c) Diffraction (d) E = hv [IIT 1992]

15. Which combinations of quantum numbers n, l, m and s for the electron in an atom does not provide
a permissible solution of the wave equation? [IIT 1992]
(a) 3, 2, –2, 1/2 (b) 3, 3, 1, –1/2 (c) 3, 2, 1, 1/2 (d) 3, 1, 1, –1/2

16. The orbital angular momentum of an electron in 2s orbital is [IIT 1996]

1 h h h
(a)  · (b) zero (c) (d) 2·
2 2 2 2

17. An element M has an atomic mass 19 and atomic number 9, its ion is represented by [IIT 1996]
+ 2+ – 2–
(a) M (b) M (c) M (d) M

18. The energy of an electron in the first Bohr orbit of H atom is –13.6 eV. The possible energy value(s)
of the excited state(s) for electron in Bohr orbits of hydrogen is (are) [IIT 1998]
(a) –3.4 eV (b) –4.2 eV (c) –6.8 eV (d) +6.8 eV

19. The electrons, identified by quantum numbers n and l,


(i) n = 4, l = 1 (ii) n = 4, l = 0 (iii) n = 3, l = 2 (iv) n = 3, l = 1
can be placed in order of increasing energy, from the lowest to highest, as [IIT 1999]
(a) (iv) < (ii) < (iii) < (i) (b) (ii) < (iv) < (i) < (iii)
(c) (i) < (iii) < (ii) < (iv) (d) (iii) < (i) < (iv) < (ii)

20. The number of nodal planes in a px orbital is [IIT 2000]


(a) one (b) two (c) three (d) zero

21. The electronic configuration of an element is 1 s 2 2 s 2 2 p 6 3 s 2 3 p 6 3 d 5 4 s 1 . It


represents its [IIT 2000]
(a) excited state (b) ground state (c) cationic form (d) anionic form

22. Vn = PE; Kn = KE; En = TE; rn = radius [IIT 2006]

Vn
(i) =a A. 1
Kn
(ii) b = angular momentum in ground state in 1s orbital B. 0
(iii) rn = (En)c C. –1
1
(iv) = (Z)d D. –2
rn
a, b, c, d are related as
(a) (i) A; (ii) B; (iii) D; (iv) B (b) (i) D; (ii) B; (iii) C; (iv) A
(c) (i) D; (ii) C; (iii) B; (iv) A (d) (i) B; (ii) A; (iii) D; (iv) C

Amity Institute for Competitive Examinations : Phones: 24336143, 24336144, 25573111/12/13/14,95120-2431839/42 [ 100 ]
Atomic Structure AICE (IIT–JEE)

23. Match the entries in Column I with the correctly related quantum number(s) in column II. Indicate
your answer by darkening the appropriate bubbles of the 4 × 4 matrix given
in the ORS. [IIT 2008 (P-II)]
A. Orbital angular momentum of the electron (p) Principal quantum number
in a hydrogen-like atomic orbital
B. A hydrogen-like one-electron wave function (q) Azimuthal quantum number
obeying Pauli principle
C. Shape, size and orientation of hydrogen-like (r) Magnetic quantum number
atomic orbitals
D. Probability density of electron at the nucleus (s) Electron spin quantum number
in hydrogen-like atom

24. The maximum number of electrons that can have principal quantum number, n = 3, and spin quantum
number, ms – 1/2, is [IIT 2011]

25. The kinetic energy of an electron in the second Bohr orbit of a hydrogen atom is
[a0 is Bohr radius] [IIT 2012]

h2 h2 h2 h2
(a) (b) (c) (d)
42 ma02 162 ma02 322 ma02 64 2 ma02

26. The atomic masses of He and Ne are 4 and 20 a.m.u., respectively. The value of the
de Broglie wavelength of He gas at –73 °C is “M” times that of the de Broglie wavelength of Ne at
727°C. M is [JEE-Advance 2013]

1
27. In an atom, the total number of electrons having quantum numbers n = 4, |m1| = 1 and ms   is
2
[JEE-Advance 2014]

28. The energy required to dislodge electron from excited isolated H-atom.
IE1 = 13.6 eV is [DCE-2000]
(a) = 13.6 eV (b) > 13.6 eV (c) < 13.6 & > 3.4 eV (d)  3.4 eV

29. N2 atom has 3 unpaired electron, because of [DCE-1999]


(a) Hund’s Rule (b) Uncertainty Principle
(c) Pauli’s Exclusion Principle (d) Aufbau’s Rule

30. The set of quantum number not applicable for an electron in an atom is [DCE-1999]
(a) n = 1, l = 1, m = 1, m = +1/2 (b) n = 1, l = 0, m = 0, m = +1/2
(c) n = 1, l = 0, m = 0, m = –1/2 (d) n = 2, l = 0, m = 0, m = +1/2

31. Which of the following quantum numbers represent an impossible arrangement? [DCE-1996]
(a) n = 3, l = 2, m = –2, m = 1/2 (b) n = 4, l = 0, m = 0, m = 1/2
(c) n = 3, l = 2, m = –3, m = 1/2 (d) n = 5, l = 3, m = 0, m = –1/2

Amity Institute for Competitive Examinations : Phones: 24336143, 24336144, 25573111/12/13/14,95120-2431839/42 [ 101 ]
Atomic Structure AICE (IIT–JEE)

32. The momentum associated with a photon of frequency v is [DCE-1996]

hv hc uc h
(a) (b) (c) (d)
c v h uc

33. The quantum number values for last unpaired electrons of chlorine are [DCE-1995]
(a) n = 3, l = 0, m = 0, m = –1/2 (b) n = 3, l = 1, m = 2, m = +1/2
(c) n = 3, l = 1, m = +1, m = +1/2 (d) n = 3, l = 2, m = –1, m = –1/2

34. The idea that most of the mass of an atom is concentrated in a very small core, i.e., nucleus is
given by [DCE-1994]
(a) Amedo Avogadro (b) Rutherford (c) Bohr (d) Henery Mosley

35. Energy of photon of visible light is [DCE-2006]


(a) 1 eV (b) 1 MeV (c) 1 µeV (d) 1 KeV

36. Which one of the following species is diamagnetic in nature ? [AIEEE-2005]

(a) He2 (b) H 2 (c) H 2 (d) H –2

37. Which of the following sets of quantum numbers is correct for an electron in
4f orbital? [AIEEE-2004]

1 1
(a) n = 4, l = 3, m = +1, s = + (b) n = 4, l = 4, m = –4, s = –
2 2

1 1
(c) n = 4, l = 3, m = +4, s = + (d) n = 3, l = 2, m = +4, s = +
2 2

38. Consider the ground state of Cr atom (Z = 24). The numbers of electrons with the azimuthal quantum
numbers, l = 1 and 2 are, respectively [AIEEE-2004]
(a) 16 and 4 (b) 12 and 5 (c) 12 and 4 (d) 16 and 5

h
39. The orbital angular momentum for an electron revolving in an orbit is given by l (l  1) . This
2
momentum for an s-electron will be given by [AIEEE-2003]

h h 1 h
(a) zero (b) (c) 2 (d)  ·
2 2 2 2

40. In a hydrogen atom, if energy of an electron in ground state is 13.6 eV, then that in the 2nd excited
state is [AIEEE-2002]
(a) 1.51 eV (b) 3.4 eV (c) 6.04 eV (d) 13.6 eV

41. Uncertainty in position of a minute particle of mass 25 g in space is 10–5 m. What is the uncertainty
in its velocity (in ms–1)? (h = 6.6 × 10–34 Js) [AIEEE-2002]
(a) 2.1 × 10–34 (b) 0.5 × 10–34 (c) 2.1 × 10–28 (d) 0.5 × 10–23

Amity Institute for Competitive Examinations : Phones: 24336143, 24336144, 25573111/12/13/14,95120-2431839/42 [ 102 ]
Atomic Structure AICE (IIT–JEE)

42. According to Bohr’s theory, the angular momentum of an electron in 5th orbit is [AIEEE-2006]
(a) 2.5 h/  (b) 25 h/  (c) 1.0 h/  (d) 10 h/ 

43. Uncertainty in the position of an electron (mass = 9.1 × 10–31 kg) moving with a velocity 300 ms–1,
accurate upto 0.001%, will be [AIEEE-2006]
(a) 3.84 × 10–2 m (b) 19.2 × 10–2 m (c) 5.76 × 10–2 m (d) 1.92 × 10–2 m
(h = 6.663 × 10–34 Js)

44. Which of the following sets of quantum numbers represents the highest energy of an atom?
(a) n = 4, l = 0, m = 0, s = + 1/2 (b) n = 3, l = 0, m = 0, s = + 1/2
(c) n = 3, l = 1, m = 1, s = + 1/2 (d) n = 3, l = 2, m = 1, s = + 1/2 [AIEEE-2007]

45. In an atom, an electron is moving with a speed of 600 m/s with an accuracy of 0.005%. Certainty
with which the position of the electron can be located is (h = 6.6 × 10–34 kg m2s–1, mass of electron,
em = 9.1 × 10–31 kg): [AIEEE-2009]
(a) 5.10 × 10–3m (b) 1.92 × 10–3m (c) 3.84 × 10–3m (d) 1.52 × 10–4m

46. A gas absorbs a photon of 355 nm and emits at two wavelengths. If one of the emissions is at
680 nm, the other is at: [AIEEE-2011]
(a) 1035 nm (b) 325 nm (c) 743 nm (d) 518 nm

47. The outer electron configuration of Gd (Atomic No: 64) is: [AIEEE-2011]
(a) 4f 3 5d5 6s2 (b) 4f 8 5d0 6s2 (c) 4f 4 5d4 6s2 (d) 4f 7 5d1 6s2

48. The electrons identified by quantum numbers n and 1: [AIEEE-2012]


(A) n = 4, l = 1 (B) n = 4, l = 0 (C) n = 3, l = 2 (d) n = 3, l = 1
can be placed in order of increasing energy as:
(a) (C) < (D) < (B) < (A) (b) (D) < (B) < (C) < (A)
(c) (B) < (D) < (A) < (C) (d) (A) < (C) < (B) < (D)

2
18  Z 
49. Energy of an electron is given by E  2.178  10 J  2  . Wavelength of light required to excite
n 
an electron in an hydrogen atom from level n = 1 to n = 2 will be [JEE-Mains 2013]

(h = 6.62 × 10–34 Js and c = 3.0 × 108 ms–1)

(a) 6.500 × 10–7 m (b) 8.500 × 10–7m (c) 1.214 × 10–7 m (d) 2.816 × 10–7 m

50. The correct set of four quantum numbers for the valence electrons of rubidium atom (Z = 37) is
[JEE-Mains 2014]

1 1 1 1
(a) 5, 1, 1, + (b) 5, 0, 1, + (c) 5, 0, 0, + (d) 5, 1, 0, +
2 2 2 2

Amity Institute for Competitive Examinations : Phones: 24336143, 24336144, 25573111/12/13/14,95120-2431839/42 [ 103 ]
Atomic Structure AICE (IIT–JEE)

51. A stream of electrons from a heated filament was passed between two charged plates kept at a
potential difference V esu. If e and m are charge and mass of an electron, respectively, then the
value of h/ (where  is wavelength associated with electron wave) is
given by: [JEE Mains 2016]

(a) 2meV (b) meV (c) 2meV (d) meV

52. P is the probability of finding the 1s electron of hydrogen atom in a spherical shell of infinitesimal
thickness, dr, at a distance r from the nucleus. The volume of this shell is 4r2dr. The qualitative
sketch of the dependence of P on r is: [JEE Advanced 2016]

(a) (b)

(c) (d)

53. According to Molecular Orbital Theory, [JEE Advanced 2016]


2
(a) C 2
is expected to be diamagnetic
(b) O22 is expected to have a longer bond length than O2
(c) N 2 and N 2 have the same bond order
(d) He2 has the same energy as two isolated He atoms

54. The radius of the second Bohr orbit for hydrogen atom is
(Planck’s Const. h = 6.6262 × 10–34 Js; mass of electron = 9.1091 × 10–31 kg; charge of electron
e = 1.60210 × 10–19 C; permittivity of vacuum [JEE Mains 2017]
e0 = 8.854185 × 10–12 kg–1m–3A2)
(a) 1.65 Å (b) 4.76 Å (c) 0.529 Å (d) 2.12 Å

55. Which of the following species is not paramagnetic ? [JEE Mains 2017]
(a) NO (b) CO (c) O2 (d) B 2

56. The group having isoelectronic species is [JEE Mains 2017]


2– – + 2+ – – + 2– – 2+
(a) O , F , Na , Mg (b) O , F , Na, Mg (c) O , F , Na, Mg (d) O–, F–, Na+, Mg2+

57. Among H 2 , He 2 , Li 2 , Be 2 , B 2 , C 2 , N 2 , O 2 and F 2 , the number of diamagnetic


species is [JEE Advanced 2017]
(Atomic numbers: H = 1, He = 2, Li = 3, Be = 4, B = 5, C = 6, N = 7, O = 8, F = 9)

Amity Institute for Competitive Examinations : Phones: 24336143, 24336144, 25573111/12/13/14,95120-2431839/42 [ 104 ]
Atomic Structure AICE (IIT–JEE)

Answer Q.58, Q.59 and Q.60 by appropriately matching the information given in the three
columns of the following table.

The wave function  n, l , m1 is a mathematical function whose value depends upon spherical
polar coordinates (r) of the electron and characterized by the quantum numbers n, l and
m1. Here r distance from nucleus,  is colatitude and  is azimuth. In the mathematical
functions given in the Table, Z is atomic number and a0 is Bohr radius.
Column 1 Column 2 Column 3
(I) Is orbital 3
 Zr 
 Z  2  a0 
(i)  n, l ,m1    e
 a0 

(P)
(II) 2s orbital (ii) One radial node (Q) Probability density at
1
nucleus 
a03
(III) 2pz orbital 5
 Zr  (R) Probability density is
 Z  2  2 a  maximum at nucleus
(iii)  n, l ,m1    re  0  cos 
 a0 
(IV) 3dz2 orbital (iv) xy-plane is a nodal plane (S) Energy needed to excite
electron from n =2 state
27
to n = 4 state is
32
times the energy
needed to excite
electron from n = 2 state
to n = 6 state
58. For hydrogen atom, the only CORRECT combination is: [JEE Advanced 2017]
(a) (II) (i) (Q) (b) (I) (iv) (R) (c) (I) (i) (S) (d) (I) (i) (P)

59. For He+ ion, the only INCORRECT combination is: [JEE Advanced 2017]
(a) (I) (iii) (R) (b) (II) (ii) (Q) (c) (I) (i) (S) (d) (I) (i) (R)

60. For the given orbital in Column I, the only CORRECT combination for any hydrogen-like
species is: [JEE Advanced 2017]
(a) (IV) (iv) (R) (b) (III) (iii) (P) (c) (II) (ii) (P) (d) (I) (ii) (S)
61. The metal used for making X-ray tube window is: [JEE Mains 2019]
(a) Mg (b) Na (c) Ca (d) be
62. Which of the graphs shown below does not represent the relationship between incident light and the
electron ejected from metal surface? [JEE Mains 2019]

 
K.E. of e s No. of e s

(a) (b)
0 0
Energy of light Frequency of light

Amity Institute for Competitive Examinations : Phones: 24336143, 24336144, 25573111/12/13/14,95120-2431839/42 [ 105 ]
Atomic Structure AICE (IIT–JEE)

K.E. of es K.E. of es

(c) (d)
0 0
Frequency of light Intensity of light

63. Heat treatment of muscular pain involves radiation of wavelength of about 900 nm, Which spectral
line of H-atom is suitable for this purpose? [JEE Mains 2019]
[RH = 1 × 105 cm–1 h = 6.6 × 10–34 Js, c = 3 × 108 ms–1]
(a) Paschen, 5 3 (b) Paschen, 3 (c) Lyman, 1 (d) Balmer, 2

64. What is the work function of the metal if the light of wavelength 4000A generates photoelectrons of
velocity 6 × 105 ms–1 from it? [JEE Mains 2019]
(Mass of electron = 9 × 10–31 kg
Velocity of light = 3 × 108 ms–1
Planck’s constant = 6.626 × 10–34 Js
Charge of electron = 1.6 × 10–19 JeV–1)
(a) 0.9 eV (b) 4.0 eV (c) 2.1 eV (d) 3.1 eV

65. Which of the following combination of statements is true regarding the interpretation of the atomic
orbitals? [JEE Mains 2019 ]
(A) An electron in an orbital of high angular momentum stays away from the nucleus than an electron
(B) For a given value of the principal quantum number, the size of the orbit is inversely proportional
to the azimuthal quantum number.
h
(C) According to wave mechanics, the ground state angular momentum is h equal to
2
(D) The plot of  Vs r for various azimuthal quantum numbers, shows peak shifting towards higher
r value.
(a) (B), (C) (b) (A), (D) (c) (A), (B) (d) (A), (C)

66. When the first electron gain ethalpy (DegH) of oxygen is –141 kJ/mol, its second electron gain
enthalpy is: [JEE Mains 2019]
(a) almost the same as that of the first (b) negative, but less negative than the first
(c) a positive value (d) a more negative value than the first

67. The de-Broglie wavelength (l) associated with a photoelectron varies with the frequency(n) of the
incident radiation as, [0 is threshold frequency]: [JEE Mains 2019]
1 1 1 1
(a) (b)   (c)   (d)  
(   0 )3/ 2 (   0 )1/ 2 (   0 )1/ 4 (   0 )
68. If the de-Broglie wavelength of the electron in nth Bohr orbit in a hydrogenic atom is equal to 1.5 a0
(a0 is Bohr radius), then the value of n/z is: [JEE Mains 2019]
(a) 1.0 (b) 0.75 (c) 0.40 (d) 1.50

Amity Institute for Competitive Examinations : Phones: 24336143, 24336144, 25573111/12/13/14,95120-2431839/42 [ 106 ]
Atomic Structure AICE (IIT–JEE)

1
69. The number of orbitals associated with quantum numbers n = 5, ms = is: [JEE Mains 2020]
2
(a) 11 (b) 25 (c) 15 (d) 50
70. The de-Broglie wavelength of an electron in the 4th Bohr orbit is : [JEE Mains 2020]
(a) 8a 0 (b) 2a 0 (c) 4a 0 (d) 6a 0
71. The Azimuthal quantum number for the valence electrons of Ga+ ion is _______.
(Atomic number of Ga = 31) [JEE Mains 2020]
72. The value of magnetic quantum number of the outermost electron of Zn+ ion is _________.
[JEE Mains 2021]
73. Ge(Z = 32) in its ground state electronic configuration has x completely filled orbitals with ml= 0. The
value of x is _______. [JEE Mains 2021]
74. The number of hydrogen bonded water molecule(s) associated with stoichiometry
CuSO4.5H2O is _______. [JEE Mains 2021]



Amity Institute for Competitive Examinations : Phones: 24336143, 24336144, 25573111/12/13/14,95120-2431839/42 [ 107 ]
Atomic Structure AICE (IIT–JEE)

CHAPTER TEST
SECTION - I: MULTIPLE CHOICE QUESTIONS (Single Correct)
1. For the energy levels in an atom, which one of the following statement is correct ?
(a) There are six principal electron energy levels observed in the atoms that exist
(b) The second principal energy level has five sub-energy levels and contain a maximum of ten
electrons
(c) The principal energy level (N) can have a maximum of 64 electrons
(d) The 4s sub-energy level has lower energy than 3d sub-energy level

2. The quantum number not obtained from the Schrodinger’s wave equation is
(a) n (b) l (c) m (d) s

3. Which among the following graphs explains the photoelectric effect

v

(a) (b) (c) (d) v

KE of emitted
KE of emitted 
electrons
electrons
4. Which sublevel is filled in atoms after sublevel 5 s?
(a) 5p (b) 5d (c) 4p (d) 4d

5. The velocity of an electron in the first orbit of the H-atom is


1
(a) 137 times that of velocity of light (b) times that of velocity of light
137
1
(c) 274 times that of velocity of light (d) times that of velocity of light
274

SECTION - II: MULTIPLE CHOICE QUESTIONS (More than one correct)


6. The correct statement is/are
(a) An electron in an orbit can absorb only one photon and that too equivalent in energy difference
between the two orbits where transition takes place.
(b) 3d sub-shell penetrates more towards nucleus than 4s.
(c) The energy change between two successive orbits increases with increasing value of n.
(d) The number of angular nodes is (n – l – 1)

7. Which of the following statement/s is/are correct?


(a) Number of electrons in Ne having their angular momentum equal to zero are four
(b) Number of waves made by an electron in an orbit is equal to the number of orbit.
(c) Each atom has at least one orbital symmetrical about the nucleus.
(d) An electron can be de-excited from 1s orbital.

8. Which of the following species does not contain unpaired electron/s?


(a) Al2+ (b) Cu+ (c) Sc3+ (d) Zn2+

Amity Institute for Competitive Examinations : Phones: 24336143, 24336144, 25573111/12/13/14,95120-2431839/42 [ 108 ]
Atomic Structure AICE (IIT–JEE)

9. Choose the correct statement/s:


(a) The number of spectral lines given when an electron drops from 5th to 2nd shell is 6.
(b) Only Lyman series is observed in emission and absorption spectra both.
(c) K.E. of emitted photoelectrons is independent of the wavelength of incident radiation.
(d) 3 shells are present in Na+ ion.

10. The incorrect statement/s is/are


(a) The energy of 3d-orbital is less than 4s orbital.
(b) 3d-orbitals are far away from nucleus than the 4s-orbital
(c) YZ represents a nodal plane in a 3px orbital
(d) Nodal plane in a 4dxy orbital is 3.

SECTION - III: INTEGER TYPE QUESTIONS


11. The value of Rydberg constant RH is ‘y’ × 105 cm–1. If He+ ions are known to have the wave length
difference beetween first (of the longest wavelength) lines of Balmer and Lyman series equal to
133.7 nm. Calculate y’,

12. The accelerating potential needed to produce an electron beam with an effective wavelength of
0.090 Å is 1.86 × 10x volts x is

13. The total number of ‘m’ values for all the orbitals in M-shell is

14. Number of unpaired electrons in V3+ ion is

15. Number of nodes in d z 2 orbital is

16. The transition of electron occurs in H-atom from 6th to 3rd orbit. The number of spectral lines given
out is

17. Total number of degenerate orbital in y ψ4,2,0 is

18. The wavelenth of nth line of Balmer series for an orbital is 4103 Å. The value of ‘n’ is

SECTION - IV: MATRIX MATCH


19. Match the following:
According to Bohr’s theory
En = Total energy Kn = Kinetic energy
Vn = Potential energy rn = Radius of nth orbit
Column – I Column – II
(A) Vn /Kn = ? (p) 0

(B) If radius of nth orbit  Enx; x = ? (q) –1

(C) Angular momentum in lowest orbital (r) –2


1
(D) n
 Z y; y  ? (s) 1
r
(a) (A) – (s); (B) – (r); (C) – (p); (D) – (q) (b) (A) – (p); (B) – (q); (C) – (r); (D) – (s)
(c) (A) – (r); (B) – (s); (C) – (q); (D) – (p) (d) (A) – (q); (B) – (p); (C) – (s); (D) – (r)

Amity Institute for Competitive Examinations : Phones: 24336143, 24336144, 25573111/12/13/14,95120-2431839/42 [ 109 ]
Atomic Structure AICE (IIT–JEE)

SECTION - V: COMPREHENSION TYPE QUESTIONS


Comprehension

Whenever an electron falls from a higher level of energy to lower level, equivalent amount of energy is given
out. The jump of electrons not only depends upon major energy shell but also on the nature of orbital.
According to Bohr’s theory E1 for H-atom is 2.17 × 10–18 J/atom.

20. The wavelength of H line of Balmer series for H-atom is


(a) 386 nm (b) 486 nm (c) 586 nm (d) 686 nm

21. The corresponding H - line of He+ has the frequency


(a) 18.64 × 1014 (b) 9.12 × 1014 (c) 4.56 × 1014 (d) 2.46 × 1015

22. The longest wavelength of light which can remove electron completely from 2nd orbit of H-atom is
(a) 4000Å (b) 3000Å (c) 5060Å (d) 3663Å



Amity Institute for Competitive Examinations : Phones: 24336143, 24336144, 25573111/12/13/14,95120-2431839/42 [ 110 ]
Atomic Structure AICE (IIT–JEE)

ANSWERS
MULTIPLE CHOICE QUESTIONS (Single Correct )

1. (b) 2. (d) 3. (c) 4. (a) 5. (c)


6. (a) 7. (d) 8. (a) 9. (a) 10. (c)
11. (a) 12. (d) 13. (c) 14. (a) 15. (d)
INTEGER TYPE QUESTIONS

16. (2) 17. (3) 18. (3) 19. (2) 20. (7)
21. (3) 22. (2) 23. (1) 24. (6) 25. (6)
MULTIPLE CHOICE QUESTIONS (More than one Correct )

26. (a,b,c) 27. (a,b,c) 28. (c,d) 29. (a,b,d) 30. (a,c)
31. (a,b,c) 32. (a,b) 33. (b,c) 34. (a,b,d) 35. (a,c,d)
MATRIX MATCH TYPE QUESTIONS

36. A-(r); B-(s); C-(q); D-(p) 37. A-(r); B-(s); C-(p); D-(q)
COMPREHENSION TYPE QUESTIONS

38. (c) 39. (a) 40. (d) 41. (b) 42. (c)
43. (b)
PREVIOUS YEAR QUESTIONS

1. (c) 2. (b) 3. (c) 4. (d) 5. (a)


6. (a) 7. (d) 8. (a) 9. (d) 10. (c)
11. (b) 12. (d) 13. (b) 14. (d) 15. (b)
16. (b) 17. (c) 18. (a) 19. (a) 20. (a)
21. (b) 22. (b) 23. A-(q); B-(p),(q),(r),(s); C-(p),(q),(r); D-(p),(q),(r)
24. (9) 25. (c) 26. (5) 27. (3) 28. (d)
29. (a) 30. (a) 31. (c) 32. (a) 33. (c)
34. (b) 35. (a) 36. (b) 37. (a) 38. (b)
39. (a) 40. (a) 41. (c) 42. (a) 43. (d)
44. (d) 45. (b) 46. (c) 47. (d) 48. (b)
49. (c) 50. (c) 51. (a) 52. (b) 53. (a, c)
54. (d) 55. (b) 56. (a) 57. (6) 58. (c)
59. (a) 60. (c) 61. (d) 62. (c) 63. (b)
64. (c) 65. (d) 66. (c) 67. (b) 68. (b)
69. (b) 70. (a) 71. (0) 72. (0) 73. (7)
74. (1)

Amity Institute for Competitive Examinations : Phones: 24336143, 24336144, 25573111/12/13/14,95120-2431839/42 [ 111 ]
Atomic Structure AICE (IIT–JEE)

CHAPTER TEST

1. (d) 2. (d) 3. (b) 4. (d) 5. (b)


6. (a, b) 7. (a, b, c) 8. (c, d) 9. (a, b) 10. (a, b,d)
11. (1) 12. (4) 13. (9) 14. (2) 15. (0)
16. (6) 17. (5) 18. (4) 19. (a) 20. (b)
21. (d) 22. (d)



Amity Institute for Competitive Examinations : Phones: 24336143, 24336144, 25573111/12/13/14,95120-2431839/42 [ 112 ]
Atomic Structure AICE (IIT–JEE)

HINTS AND SOLUTIONS


MULTIPLE CHOICE QUESTIONS (Single Correct)

n2
1. Use the formula rn  0.53 
z

1 2 1 1 
2. Wave number (  )    RH .z  n 2  n 2 
 1 2 

3. Use the relation given in answer-2


4. Factual

n2
5. Use rn  0.53 
z
6. Factual
7. The orbital having lower energy is always filled prior to the orbital having higher energy.

1 2
8. h(   0 )  K .E i.e., h(   0 )  mv
2
9. Factual
10. 19th elecron of Cr (atomic no. 24) enters into the 4s orbital.

h
11. x.m. v 
4
12. Both the orbitals belongs ot the same shell i.e., 3rd.

1 1 1
13. Use  RH .z 2  2  
 1  

14. Third line from the red end of visible region (VIBGYOR) means the transition is taking place from
n = 5 to n = 2.

h
15. Use the relation  
mv

INTEGER TYPE QUESTIONS

hc
16. Use the relation E 

17. x=3

n( n  1)
18.
2

19. General electronic configuration of 2nd group elements is ns2.

Amity Institute for Competitive Examinations : Phones: 24336143, 24336144, 25573111/12/13/14,95120-2431839/42 [ 113 ]
Atomic Structure AICE (IIT–JEE)

20. l = 3 means f-subshell which has 7 orbitals.

21. Electrons in 2p3 are arranged as   

22. Electronic arrangement in Ni2+ is [Ar]3d8 4s0 i.e.,    


3d 8 4s0

23. Spin multiplicity = (2S + 1)


wher S = ½ (no. of unpaired electrons in the shell)
If all electrons are paired S = 0 and hence spin multiplicity in this case is 1.

n( n  1)
24. Use where n is the uppermost energy level.
2
25. n = 2, l = 1 is a 2p orbital which can accomodate a total of 6 electrons.

MULTIPLE CHOICE QUESTIONS (More than one Correct)


26. Cathode rays consists of electrons which are fundamental particles.

27. Energy of a subshell is calculated using the formula (n + l).

28. Factual

29. I.E of hydrogen like species = – (energy of the orbit)

1 2 1 1 
30. Wave number (  )    RH z  n 2  n2 
 1 2 

31. Factual

32. The energy provided to the electron i.e., 25 eV is higher than its binding energy so the electron will
become free. 3 different type of photons will be observed from IInd excited state i.e., n = 3.

33. Factual

nhc
34. Energy of EM radiation ( E )  where n is the number of photons.

COMPREHENSION TYPE QUESTIONS
38. Magnetic quantum number (m) helps in determining the orientation of electron in a subshell.

39. Number of radial nodes is calculated using the relation (n – l –1)

40. Factual



Amity Institute for Competitive Examinations : Phones: 24336143, 24336144, 25573111/12/13/14,95120-2431839/42 [ 114 ]

You might also like